由买买提看人间百态

boards

本页内容为未名空间相应帖子的节选和存档,一周内的贴子最多显示50字,超过一周显示500字 访问原贴
WaterWorld版 - [转] 红蓝眼睛的逻辑陷阱
相关主题
[合集] [转] 红蓝眼睛的逻辑陷阱问个中文标点符号的嵌套问题
再贴几张一个外嫁后生的孩子的照片大部分人不懂反证法
娶妻三次, 三个女人致其性无能每周1评:[2] 方舟子 的 16点
中国北方人跟南方人基本上是两个种族,无法协调陕西出现红眼兽 (转载)
大家承认吧蓝眼睛就是比粽眼睛好看 (转载)在自己国家岛上杀死敌人侵略者应该不算犯罪,是英雄
葛军,男,秒杀了52万江苏考生。。来做最后两题吧动物园举办【我和红眼蝉】活动 (转载)
别做小学题了,来个初中的[合集] 内贾德第一个在钓岛上声援中国 一席话太解气了!
问个奥数题靠~ 受不了,宇宙大得让我头晕[讨论]
相关话题的讨论汇总
话题: 红眼话题: 眼睛话题: 岛上话题: 自杀话题: 知道
进入WaterWorld版参与讨论
1 2 下页 末页 (共2页)
l*3
发帖数: 2279
1
转子知乎: http://www.zhihu.com/question/21262930
一个关于数学归纳法的悖论问题:到底是第N天有N个红眼睛自杀,还是什么都不会发生?
此问题最早据说是澳大利亚的华裔数学神童陶哲轩在网上贴出来让大家思考,逗大家玩
儿的。但却是真的把我难住了,一直百思不得其解。在此求教方家。
题目是这样的。说一个岛上有100个人,其中有5个红眼睛,95个蓝眼睛。这个岛有三个
奇怪的宗教规则。
1. 他们不能照镜子,不能看自己眼睛的颜色。
2. 他们不能告诉别人对方的眼睛是什么颜色。
3. 一旦有人知道了自己是红眼睛,他就必须在当天夜里自杀。
某天,有个旅行者到了这个岛上。由于不知道这里的规矩,所以他在和全岛人一起狂欢
的时候,不留神就说了一句话:【你们这里有红眼睛的人。】
最后的问题是:假设这个岛上的人足够聪明,每个人都可以做出缜密的逻辑推理。请问
这个岛上将会发生什么?
此问题的第一个答案是用数学归纳法得出的:如果这个岛上有N个红眼睛,那么在旅行
者说这句话的第N天,他们全部都会自杀。具体到本题则是,在第5天,这个岛上的5个
红眼睛会全部自杀。
证明过程如下:
如果这个岛上只有1个红眼睛,其他人都是蓝眼睛。那么,当旅行者说了这句话之后,
此人立刻就会知道自己是红眼睛,他就会在当天自杀。即,当n取第一个值n0=1时,命
题成立。
假设当这个岛上有N个红眼睛的时候,在旅行者说了这句话之后的第N天,这些红眼睛会
全部自杀。
那么,当这个岛上有N+1个红眼睛的时候,在每个红眼睛看来,岛上都确定有N个红眼睛
,并等待着他们在第N天自杀。而在第N天,大家都没有自杀。所以一到第N+1天,每个
红眼睛都明白了这个岛上还有第N+1个红眼睛——他自己。于是大家都在第N+1天自杀了。
所以命题得证:如果这个岛上有N个红眼睛,那么在旅行者说这句话的第N天,他们全部
都会自杀。
如果上述证明还让人有疑惑的话,也可以改用穷举法来证明。
当岛上只有一个红眼睛的时候,在旅行者说完这句话的当天,他就会自杀。这个无疑。
当岛上有两个红眼睛的时候。在旅行者说完这句话的当天,这两个红眼睛都在等着对方
自杀,但对方却没有自杀。于是在第二天他们立刻明白了自己也是红眼睛,于是在第二
天一起自杀了。
以此往下推理,当岛上有三个红眼睛的时候。旅行者说完这句话,每个红眼睛都在等着
第二天另外两个红眼睛集体自杀,但他们没有自杀。所以到了第三天,大家都明白了自
己也是红眼睛,就一起自杀了。
如此类推下去。就得出了命题:如果岛上有N个红眼睛,那么在旅行者说完这句话后的
第N天,这个N个红眼睛会一起自杀。具体到本题就是,到了第五天,这五个红眼睛一起
自杀。
以上证明看起来非常美妙。
可是可是可是可是可是可是。
陶哲轩说,这个旅行者事实上讲了一句废话,没有带来任何新的信息。因为这岛上有95
个蓝眼睛,5个红眼睛。每个人都知道这岛上有红眼睛的人。无非是蓝眼睛的人看到有5
个红眼睛,红眼睛的人看到有4个红眼睛而已。旅行者说的那句【岛上有红眼睛的人】
,没有输入任何新的信息,他说的就是岛上的人每天都看到的景象。所以哪怕岛上的人
思维再缜密严谨,也不会有任何自杀的情况发生。
从这个角度来说,也对呀。
这到底是怎么一回事,我迷茫了。请求方家解答疑惑。
z****e
发帖数: 54598
2
陶哲轩说,这个旅行者事实上讲了一句废话,没有带来任何新的信息。因为这岛上有95
个蓝眼睛,5个红眼睛。每个人都知道这岛上有红眼睛的人。无非是蓝眼睛的人看到有5
个红眼睛,红眼睛的人看到有4个红眼睛而已。旅行者说的那句【岛上有红眼睛的人】
,没有输入任何新的信息,他说的就是岛上的人每天都看到的景象。所以哪怕岛上的人
思维再缜密严谨,也不会有任何自杀的情况发生。
红眼睛的人知道有4个红眼睛,但是他因为规矩之一的缘故
所以不能告诉其它四个红眼睛,这个窗户纸只能由旅行者来点破
所以如果旅行者不说,都不会死
一旦点破,那就是每个人根据自身的观察等n天
那这个n的数字可能是4也可能是5,对于红眼睛来说,因为他不知道是4还是5
所以一旦等了4天,其它四个红眼睛还不死,那就说明自己也是
所以第五天所有红眼睛都会自杀
所以旅行者那一句话不是废话,他不说有红眼睛
其它人不能说,哪怕看到也不能说,所以红眼睛不知道自己是
所以红眼睛不会死,保持着一种平衡,看到也没用
因为不能说,红眼睛也就不知道,不知道的话也就不会去自杀
至于别人自杀不自杀,那因为你没告诉他是红眼睛
所以红眼睛没有自杀的义务,不知道怎么做?
知道和点破是两回事,现实中估计旅行者会被抓起来打死
然后告诉后来的旅行者,这就是说出有红眼睛的下场
l*3
发帖数: 2279
3
旅行者只说了 "这个岛上有红眼睛的人"
这难道不是尽人皆知的事情吗? 所有人都知道这个岛上有红眼睛的人呀.

95
有5

【在 z****e 的大作中提到】
: 陶哲轩说,这个旅行者事实上讲了一句废话,没有带来任何新的信息。因为这岛上有95
: 个蓝眼睛,5个红眼睛。每个人都知道这岛上有红眼睛的人。无非是蓝眼睛的人看到有5
: 个红眼睛,红眼睛的人看到有4个红眼睛而已。旅行者说的那句【岛上有红眼睛的人】
: ,没有输入任何新的信息,他说的就是岛上的人每天都看到的景象。所以哪怕岛上的人
: 思维再缜密严谨,也不会有任何自杀的情况发生。
: 红眼睛的人知道有4个红眼睛,但是他因为规矩之一的缘故
: 所以不能告诉其它四个红眼睛,这个窗户纸只能由旅行者来点破
: 所以如果旅行者不说,都不会死
: 一旦点破,那就是每个人根据自身的观察等n天
: 那这个n的数字可能是4也可能是5,对于红眼睛来说,因为他不知道是4还是5

z****e
发帖数: 54598
4
知道和采信是两回事
旅行者说出来之后,有红眼睛这个事实,就可以被确认被其它四个红眼睛知道
并且红眼睛的人也确定其它四个红眼睛在等着看其它人自杀
这个条件改变了一切

【在 l*3 的大作中提到】
: 旅行者只说了 "这个岛上有红眼睛的人"
: 这难道不是尽人皆知的事情吗? 所有人都知道这个岛上有红眼睛的人呀.
:
: 95
: 有5

z****e
发帖数: 54598
5
这是常见的统计里面的condition的改变造成结果不同的情况
类似,有三个门,其中一个后面有只猪
主持人让你选,你选好后,主持人推开剩下两个中的一个,说这里没有猪……
剩下的故事都知道了
这个剧本里面,一开始红眼睛没有在等其它红眼睛自杀
但是旅行者一旦点破了,红眼睛就开始等其它红眼睛自杀了
这个条件被改变了,造成了结果的不同

【在 l*3 的大作中提到】
: 旅行者只说了 "这个岛上有红眼睛的人"
: 这难道不是尽人皆知的事情吗? 所有人都知道这个岛上有红眼睛的人呀.
:
: 95
: 有5

z****e
发帖数: 54598
6
删了之前的一个回帖,前面说得没错
z****e
发帖数: 54598
7
把原命题精简一下
当这个岛上只有3个人
你是其中一个,你看到其它两个红眼睛
那么前两天这两个都没自杀
问:第三天晚上你自杀不自杀?
l*3
发帖数: 2279
8
我在第一天晚上就去把他们杀了!!!

【在 z****e 的大作中提到】
: 把原命题精简一下
: 当这个岛上只有3个人
: 你是其中一个,你看到其它两个红眼睛
: 那么前两天这两个都没自杀
: 问:第三天晚上你自杀不自杀?

z****e
发帖数: 54598
9
我看可以哈哈

【在 l*3 的大作中提到】
: 我在第一天晚上就去把他们杀了!!!
c********u
发帖数: 1608
10
这是对逻辑好的人最好的警告,嘿嘿.笨点还能保命.

生?

【在 l*3 的大作中提到】
: 转子知乎: http://www.zhihu.com/question/21262930
: 一个关于数学归纳法的悖论问题:到底是第N天有N个红眼睛自杀,还是什么都不会发生?
: 此问题最早据说是澳大利亚的华裔数学神童陶哲轩在网上贴出来让大家思考,逗大家玩
: 儿的。但却是真的把我难住了,一直百思不得其解。在此求教方家。
: 题目是这样的。说一个岛上有100个人,其中有5个红眼睛,95个蓝眼睛。这个岛有三个
: 奇怪的宗教规则。
: 1. 他们不能照镜子,不能看自己眼睛的颜色。
: 2. 他们不能告诉别人对方的眼睛是什么颜色。
: 3. 一旦有人知道了自己是红眼睛,他就必须在当天夜里自杀。
: 某天,有个旅行者到了这个岛上。由于不知道这里的规矩,所以他在和全岛人一起狂欢

相关主题
葛军,男,秒杀了52万江苏考生。。来做最后两题吧问个中文标点符号的嵌套问题
别做小学题了,来个初中的大部分人不懂反证法
问个奥数题每周1评:[2] 方舟子 的 16点
进入WaterWorld版参与讨论
T*********I
发帖数: 10729
11
精辟

【在 z****e 的大作中提到】
: 把原命题精简一下
: 当这个岛上只有3个人
: 你是其中一个,你看到其它两个红眼睛
: 那么前两天这两个都没自杀
: 问:第三天晚上你自杀不自杀?

s*******e
发帖数: 1630
12
违反宗教规则的下场是处死吗?如果是,那么如果红眼睛里边有一个笨蛋或者不诚实的
人,到时候了还不自杀,其他人貌似也奈何不了他,因为总不能说:“你红眼睛的干嘛
不自杀?”否则自己也违规了
T*********I
发帖数: 10729
13
不对,还是得自杀。

【在 z****e 的大作中提到】
: 把原命题精简一下
: 当这个岛上只有3个人
: 你是其中一个,你看到其它两个红眼睛
: 那么前两天这两个都没自杀
: 问:第三天晚上你自杀不自杀?

q***3
发帖数: 5088
14
要是红眼睛们再多等一天,这个岛就是他们的了
m****a
发帖数: 1257
15
这有什么好迷惑的,那个旅行者提供的绝对不是没用的信息!
如果只有一个红眼,他说话之后当晚必定自杀。如果当晚没人自杀,
旅行者的话就相当于开启了一扇门,每过一天就相当于告诉岛上的人,
这里至少有n+1个红眼。
再简单不过的问题了,哪里来的什么悖论

生?

【在 l*3 的大作中提到】
: 转子知乎: http://www.zhihu.com/question/21262930
: 一个关于数学归纳法的悖论问题:到底是第N天有N个红眼睛自杀,还是什么都不会发生?
: 此问题最早据说是澳大利亚的华裔数学神童陶哲轩在网上贴出来让大家思考,逗大家玩
: 儿的。但却是真的把我难住了,一直百思不得其解。在此求教方家。
: 题目是这样的。说一个岛上有100个人,其中有5个红眼睛,95个蓝眼睛。这个岛有三个
: 奇怪的宗教规则。
: 1. 他们不能照镜子,不能看自己眼睛的颜色。
: 2. 他们不能告诉别人对方的眼睛是什么颜色。
: 3. 一旦有人知道了自己是红眼睛,他就必须在当天夜里自杀。
: 某天,有个旅行者到了这个岛上。由于不知道这里的规矩,所以他在和全岛人一起狂欢

d****n
发帖数: 12461
16
旅行者说的就是归纳法的第一步啊,所以绝对不是一句废话。

生?

【在 l*3 的大作中提到】
: 转子知乎: http://www.zhihu.com/question/21262930
: 一个关于数学归纳法的悖论问题:到底是第N天有N个红眼睛自杀,还是什么都不会发生?
: 此问题最早据说是澳大利亚的华裔数学神童陶哲轩在网上贴出来让大家思考,逗大家玩
: 儿的。但却是真的把我难住了,一直百思不得其解。在此求教方家。
: 题目是这样的。说一个岛上有100个人,其中有5个红眼睛,95个蓝眼睛。这个岛有三个
: 奇怪的宗教规则。
: 1. 他们不能照镜子,不能看自己眼睛的颜色。
: 2. 他们不能告诉别人对方的眼睛是什么颜色。
: 3. 一旦有人知道了自己是红眼睛,他就必须在当天夜里自杀。
: 某天,有个旅行者到了这个岛上。由于不知道这里的规矩,所以他在和全岛人一起狂欢

p**s
发帖数: 2707
17
确实不是废话,但是你没解释清楚,这句话带来的新信息是什么。
旅行者就算不说,有红眼睛这个事实,也可以被确认被其它四个红眼睛知道

【在 z****e 的大作中提到】
: 知道和采信是两回事
: 旅行者说出来之后,有红眼睛这个事实,就可以被确认被其它四个红眼睛知道
: 并且红眼睛的人也确定其它四个红眼睛在等着看其它人自杀
: 这个条件改变了一切

a******o
发帖数: 261
18
the version i've been given states that there are N blue eyes on the island,
and that N>=1. N>=1 is a must b/c w/o it N=1 case wouldn't work.
u******a
发帖数: 7843
19
这个题不能递归吧. 因为递归的第一部根本就不存在.
岛上只有两种人, 一种知道至少有4个红眼睛, 并且知道红眼睛的人知道至少有3个红眼
睛, 一种知道至少有5个红眼睛, 并且知道红眼睛的人知道至少有4个红眼睛. 推理应该
直接从这一步开始, 而不是"假设岛上只有1个红眼睛". 因为岛上所有人都知道这个假设
根本就不存在.


生?

【在 l*3 的大作中提到】
: 转子知乎: http://www.zhihu.com/question/21262930
: 一个关于数学归纳法的悖论问题:到底是第N天有N个红眼睛自杀,还是什么都不会发生?
: 此问题最早据说是澳大利亚的华裔数学神童陶哲轩在网上贴出来让大家思考,逗大家玩
: 儿的。但却是真的把我难住了,一直百思不得其解。在此求教方家。
: 题目是这样的。说一个岛上有100个人,其中有5个红眼睛,95个蓝眼睛。这个岛有三个
: 奇怪的宗教规则。
: 1. 他们不能照镜子,不能看自己眼睛的颜色。
: 2. 他们不能告诉别人对方的眼睛是什么颜色。
: 3. 一旦有人知道了自己是红眼睛,他就必须在当天夜里自杀。
: 某天,有个旅行者到了这个岛上。由于不知道这里的规矩,所以他在和全岛人一起狂欢

S*E
发帖数: 3662
20
情形一:只有一个红眼睛。有一个人不知道有红眼睛。
于是,旅行者带来了新信息。
情形二:有两个红眼睛。每个人都知道有红眼睛。
表面上,旅行者没有带来新信息。
但是从一个红眼人甲的角度看,他本来并不知道另一个
红眼人乙是否知道这个岛上有红眼睛。
而现在有了旅行者的话,那么甲就知道乙应该知道
岛上有红眼睛。这样旅行者就带来了新信息。
在红眼睛更多的情形下,就是反复嵌套“我知道”
“你知道”“他知道”。旅行者的话改变了第N次
嵌套的结果。
相关主题
陕西出现红眼兽 (转载)[合集] 内贾德第一个在钓岛上声援中国 一席话太解气了!
在自己国家岛上杀死敌人侵略者应该不算犯罪,是英雄靠~ 受不了,宇宙大得让我头晕[讨论]
动物园举办【我和红眼蝉】活动 (转载)人不能蒙着眼睛说瞎话
进入WaterWorld版参与讨论
u******a
发帖数: 7843
21
看了下原帖知乎上的解释, 貌似懂了...逻辑论真是高端啊.

【在 S*E 的大作中提到】
: 情形一:只有一个红眼睛。有一个人不知道有红眼睛。
: 于是,旅行者带来了新信息。
: 情形二:有两个红眼睛。每个人都知道有红眼睛。
: 表面上,旅行者没有带来新信息。
: 但是从一个红眼人甲的角度看,他本来并不知道另一个
: 红眼人乙是否知道这个岛上有红眼睛。
: 而现在有了旅行者的话,那么甲就知道乙应该知道
: 岛上有红眼睛。这样旅行者就带来了新信息。
: 在红眼睛更多的情形下,就是反复嵌套“我知道”
: “你知道”“他知道”。旅行者的话改变了第N次

z*****x
发帖数: 2370
22
以前,虽然大家都知道有红眼睛(n>1 case),可是不知道别人也知道。旅行者的作用是
导致大家都知道别人也知道,大家都知道大家都知道别人也知道。。。。ie,,成了公
共知识。
这样一来,红眼睛才能推断出来自己是红眼睛。
在 plus (祝阳阳小妞豆豆早日康复) 的大作中提到: 】
p**s
发帖数: 2707
23
正解
或者说,
1,每个人都知道岛上有红眼睛
2,每个人都知道1
3,每个人都知道2
。。。
在旅行者来之前,5是不成立的,他说了那句话,5,6,7,8,。。。就都成立了。
另外就是,他说那句话,必须是当着所有人,如果是跑去每家单独说,就没用了。

【在 S*E 的大作中提到】
: 情形一:只有一个红眼睛。有一个人不知道有红眼睛。
: 于是,旅行者带来了新信息。
: 情形二:有两个红眼睛。每个人都知道有红眼睛。
: 表面上,旅行者没有带来新信息。
: 但是从一个红眼人甲的角度看,他本来并不知道另一个
: 红眼人乙是否知道这个岛上有红眼睛。
: 而现在有了旅行者的话,那么甲就知道乙应该知道
: 岛上有红眼睛。这样旅行者就带来了新信息。
: 在红眼睛更多的情形下,就是反复嵌套“我知道”
: “你知道”“他知道”。旅行者的话改变了第N次

F*****n
发帖数: 1552
24
还没看完的时候我就想到了陶轩哲的观点。本来想回复的,看到最后,发现我跟他站到
了同一高度上:D
x******s
发帖数: 398
25
这个问的逻辑就不对. 旅行者上岛, 的确没有带来任何新信息.
实际上如果岛上有5个红眼睛, 95个蓝眼睛, 有确定的某一天作为起始的第一天,每
天人全都能互相见到而且他们真的够聪明, 那这五个人早就自杀完了, 根本等不到旅行
者上岛. 不可能平时相安无事直到某天有人上岛. 告诉你题目的时候,陷阱已经设下了.
x******s
发帖数: 398
26

当n>1的时候, 不可能不知道别人也知道 "有红眼睛存在" 这个事实的.
旅行者的作用是

【在 z*****x 的大作中提到】
: 以前,虽然大家都知道有红眼睛(n>1 case),可是不知道别人也知道。旅行者的作用是
: 导致大家都知道别人也知道,大家都知道大家都知道别人也知道。。。。ie,,成了公
: 共知识。
: 这样一来,红眼睛才能推断出来自己是红眼睛。
: 在 plus (祝阳阳小妞豆豆早日康复) 的大作中提到: 】

l*3
发帖数: 2279
27
你这个不对.
就拿2个人举例吧, 没有外来者, 永远不会有人自杀.

了.

【在 x******s 的大作中提到】
:
: 当n>1的时候, 不可能不知道别人也知道 "有红眼睛存在" 这个事实的.
: 旅行者的作用是

h*x
发帖数: 674
28
要回答原贴的问题,可以先回答这样一个问题:
在旅行者到来之前,岛上为什么没有人自杀?
因为虽然每个人都知道这个岛上有红眼睛的人,但每个人都不知道自己是不是红眼睛,
所以没人自杀。
那旅行者的话改变了什么?改变了时间边界条件。
旅行者的话让今天跟明天不再一样,让时间有了一个方向,让“当天夜里自杀”这个条
件有了意义。在旅行者到来之前,每一天跟前一天没有区别,跟后一天也没有区别,所
谓“当天夜里”根本没有限定。但从旅行者说出那句话的时候,开始有了个计时器,大
家开始等待“当天夜里”会发生什么样的事情。
学过物理的人都知道。对于某些微分方程会有通解,在这个通解下,不同的边界条件会
导出不同的解。在旅行者到来之间,这个岛上的时间边界条件就像periodic boundary
condition, 在旅行者到来之后,这个岛上的时间边界条件变成了finite boundary
condition,结果当然不一样。
c****p
发帖数: 6474
29
考虑这样一个情况:
岛上只有一个红眼睛,那么旅行者说的话就是有用的信息,红眼睛当天晚上就会自杀
进而考虑有两个红眼睛,如果没有旅行者,那么两个红眼睛都不会自杀。
旅行者说了以后,两个红眼睛会在第二天晚上都自杀。
所以说这句话是有信息量的。
他说话之前,每个人都知道有红眼睛,但是他们都不知道别人是不是知道有红眼睛。
他说话之后,每个人不但知道有红眼睛,而且每个人都知道其他任何一个人都知道有红
眼睛。

【在 l*3 的大作中提到】
: 旅行者只说了 "这个岛上有红眼睛的人"
: 这难道不是尽人皆知的事情吗? 所有人都知道这个岛上有红眼睛的人呀.
:
: 95
: 有5

p**********t
发帖数: 3680
30
如果都是红眼睛呢? 睡一觉起来每一个个体都认为自己是蓝眼睛,想在第二天看到自
杀的,但是没有看到。于是继续睡,继续每一天。
相关主题
神啊,我老公眼睛都直了,我这个恨啊!再贴几张一个外嫁后生的孩子的照片
携带iPad进入中国内地要交税 (ZT)娶妻三次, 三个女人致其性无能
[合集] [转] 红蓝眼睛的逻辑陷阱中国北方人跟南方人基本上是两个种族,无法协调
进入WaterWorld版参与讨论
D*a
发帖数: 6830
31
当有四个红眼睛的时候,他假设“假设只有三个人,那么三个人会假设只有两个人”,
这是两层假设了,只有“只有三个人”这个前提存在的条件下,三个人才会假设只有两
个人。如果有五个人,就是“假设只有四个人,这四个人会假设只有三个人,这三个人
会假设只有两个人”,这就是假设套假设套假设了。但是他不需要作出任何假设,他的
知识就可以满足游客给定的条件“岛上有红眼人”。
假设下雪的话,公交车就会晚点,假设公交车晚点的话,我就赶不上晚上的电影。那么
是不是我没赶上晚上的电影,就说明下雪了?
m*****n
发帖数: 204
32
这个说的透彻。换句话说,归纳法要证明的是:如果第K天没人自杀那么N > (K-1)
旅行者的话使K == 2的CASE成立。

【在 h*x 的大作中提到】
: 要回答原贴的问题,可以先回答这样一个问题:
: 在旅行者到来之前,岛上为什么没有人自杀?
: 因为虽然每个人都知道这个岛上有红眼睛的人,但每个人都不知道自己是不是红眼睛,
: 所以没人自杀。
: 那旅行者的话改变了什么?改变了时间边界条件。
: 旅行者的话让今天跟明天不再一样,让时间有了一个方向,让“当天夜里自杀”这个条
: 件有了意义。在旅行者到来之前,每一天跟前一天没有区别,跟后一天也没有区别,所
: 谓“当天夜里”根本没有限定。但从旅行者说出那句话的时候,开始有了个计时器,大
: 家开始等待“当天夜里”会发生什么样的事情。
: 学过物理的人都知道。对于某些微分方程会有通解,在这个通解下,不同的边界条件会

z****e
发帖数: 54598
33
什么不对,我在问问题
问自杀不自杀
我的答案也是自杀,所以五个红眼睛共存的局面压根不会出现
旅行者还没来,就都挂了

【在 T*********I 的大作中提到】
: 不对,还是得自杀。
z****e
发帖数: 54598
34
会自杀
因为都看到对方是红眼睛了
所以大家都知道了
所以就等着有人自杀了
如果不自杀的话,就说明自己是
所以要死,旅行者还没来就都挂了

【在 l*3 的大作中提到】
: 你这个不对.
: 就拿2个人举例吧, 没有外来者, 永远不会有人自杀.
:
: 了.

z****e
发帖数: 54598
35
岛上只有一个红眼睛,那么旅行者说的话就是有用的信息,红眼睛当天晚上就会自杀
进而考虑有两个红眼睛,如果没有旅行者,那么两个红眼睛都不会自杀。
n=2的时候,其实就会自杀了
所以条件是错的

【在 c****p 的大作中提到】
: 考虑这样一个情况:
: 岛上只有一个红眼睛,那么旅行者说的话就是有用的信息,红眼睛当天晚上就会自杀
: 进而考虑有两个红眼睛,如果没有旅行者,那么两个红眼睛都不会自杀。
: 旅行者说了以后,两个红眼睛会在第二天晚上都自杀。
: 所以说这句话是有信息量的。
: 他说话之前,每个人都知道有红眼睛,但是他们都不知道别人是不是知道有红眼睛。
: 他说话之后,每个人不但知道有红眼睛,而且每个人都知道其他任何一个人都知道有红
: 眼睛。

x******s
发帖数: 398
36
当然会. 只要有一个大家都知道的游戏开始的时间, 必然会N人N天死. 两个人的话,
甚至不需要同步.

【在 l*3 的大作中提到】
: 你这个不对.
: 就拿2个人举例吧, 没有外来者, 永远不会有人自杀.
:
: 了.

s******s
发帖数: 13035
37
前面好多人都说的是对的。开始的时候,大家都知道有红眼睛,但是并不是每个人都知
道 每个人都知道 每个人都知道 每个人都知道 每个人都知道有红眼睛。旅行者一句话
就把这个变成公共知识,每天让大家多一层认识。
换一种说法,那五个红眼睛,我给他们命名老大到老五。
1. 老五只看到四个红眼睛,他假设自己不是红眼,那么老五认为老四看到了三个红眼。
2. 老五认为老四假设自己不是红眼,而且前面说了,老五假设自己不是红眼,那么老
五认为“老四认为老三只看到了两个红眼”
以此类推,最后的结果是
老五认为
老四认为
老三认为
老二认为
老大没看到红眼。
这系列的基本假设就是,老五认为每人都认为自己不是红眼,这就是旅行者来这前的所
有红眼睛的想法。
旅行者来了以后,过一夜,老大没死
老五认为
老四认为
老三认为
老二会很奇怪,老大为什么不自杀,所以会怀疑自己
过了第二夜,
老五认为
老四认为
老三会很奇怪为什么老二没自杀
同样,过了四夜,到了第五天,老五就只有一个怀疑对象了,就是自己,只能死了

【在 D*a 的大作中提到】
: 当有四个红眼睛的时候,他假设“假设只有三个人,那么三个人会假设只有两个人”,
: 这是两层假设了,只有“只有三个人”这个前提存在的条件下,三个人才会假设只有两
: 个人。如果有五个人,就是“假设只有四个人,这四个人会假设只有三个人,这三个人
: 会假设只有两个人”,这就是假设套假设套假设了。但是他不需要作出任何假设,他的
: 知识就可以满足游客给定的条件“岛上有红眼人”。
: 假设下雪的话,公交车就会晚点,假设公交车晚点的话,我就赶不上晚上的电影。那么
: 是不是我没赶上晚上的电影,就说明下雪了?

s*********t
发帖数: 16647
38
对lz的疑问,这位回答得最清楚

【在 S*E 的大作中提到】
: 情形一:只有一个红眼睛。有一个人不知道有红眼睛。
: 于是,旅行者带来了新信息。
: 情形二:有两个红眼睛。每个人都知道有红眼睛。
: 表面上,旅行者没有带来新信息。
: 但是从一个红眼人甲的角度看,他本来并不知道另一个
: 红眼人乙是否知道这个岛上有红眼睛。
: 而现在有了旅行者的话,那么甲就知道乙应该知道
: 岛上有红眼睛。这样旅行者就带来了新信息。
: 在红眼睛更多的情形下,就是反复嵌套“我知道”
: “你知道”“他知道”。旅行者的话改变了第N次

c******n
发帖数: 455
39
楼上各位似乎忽略了第二条规矩,那就是他们彼此之间是不能交流的,即使你看到谁是
,你也不能告诉别人,题目给出的5个具体数据在这100个人中,从自认自己不是红眼的
角度来说,红眼的认为有4个,非红眼的认为有5个,从自认自己是红眼的角度来说,红
眼的认为有5个,非红眼的认为有6个,所以,结果应该是要么都没死,要么全死。

眼。

【在 s******s 的大作中提到】
: 前面好多人都说的是对的。开始的时候,大家都知道有红眼睛,但是并不是每个人都知
: 道 每个人都知道 每个人都知道 每个人都知道 每个人都知道有红眼睛。旅行者一句话
: 就把这个变成公共知识,每天让大家多一层认识。
: 换一种说法,那五个红眼睛,我给他们命名老大到老五。
: 1. 老五只看到四个红眼睛,他假设自己不是红眼,那么老五认为老四看到了三个红眼。
: 2. 老五认为老四假设自己不是红眼,而且前面说了,老五假设自己不是红眼,那么老
: 五认为“老四认为老三只看到了两个红眼”
: 以此类推,最后的结果是
: 老五认为
: 老四认为

g*****g
发帖数: 624
40
注意,这五人是等价的,当老五在这么认为的时候,老大,老二,老三,老四都在这么
想。

眼。

【在 s******s 的大作中提到】
: 前面好多人都说的是对的。开始的时候,大家都知道有红眼睛,但是并不是每个人都知
: 道 每个人都知道 每个人都知道 每个人都知道 每个人都知道有红眼睛。旅行者一句话
: 就把这个变成公共知识,每天让大家多一层认识。
: 换一种说法,那五个红眼睛,我给他们命名老大到老五。
: 1. 老五只看到四个红眼睛,他假设自己不是红眼,那么老五认为老四看到了三个红眼。
: 2. 老五认为老四假设自己不是红眼,而且前面说了,老五假设自己不是红眼,那么老
: 五认为“老四认为老三只看到了两个红眼”
: 以此类推,最后的结果是
: 老五认为
: 老四认为

相关主题
中国北方人跟南方人基本上是两个种族,无法协调别做小学题了,来个初中的
大家承认吧蓝眼睛就是比粽眼睛好看 (转载)问个奥数题
葛军,男,秒杀了52万江苏考生。。来做最后两题吧问个中文标点符号的嵌套问题
进入WaterWorld版参与讨论
g*****g
发帖数: 624
41
如果你的逻辑成立,那么他们应该一起在同一时间死

眼。

【在 s******s 的大作中提到】
: 前面好多人都说的是对的。开始的时候,大家都知道有红眼睛,但是并不是每个人都知
: 道 每个人都知道 每个人都知道 每个人都知道 每个人都知道有红眼睛。旅行者一句话
: 就把这个变成公共知识,每天让大家多一层认识。
: 换一种说法,那五个红眼睛,我给他们命名老大到老五。
: 1. 老五只看到四个红眼睛,他假设自己不是红眼,那么老五认为老四看到了三个红眼。
: 2. 老五认为老四假设自己不是红眼,而且前面说了,老五假设自己不是红眼,那么老
: 五认为“老四认为老三只看到了两个红眼”
: 以此类推,最后的结果是
: 老五认为
: 老四认为

s******8
发帖数: 4192
42
我觉得旅行者的话只对一个和两个红眼有效,其他数目都无效。其他数目下红眼早在旅
行者上来前就死光了。除非他们在旅行者上来前不思考。
先说一个红眼,旅行者上来前这个红眼看不到任何红眼,所以他不知道有红眼这个事实
,以为全部是蓝眼,当然不会自杀。
两个红眼A,B。A看到一个红眼,第一天过后,没有自杀。那么A会认为只有一个红眼B,
B不知道自己是红眼,所以不自杀。反之亦然。
三个红眼以上,那么所有人都应该知道有红眼,没有误解,不需要旅行者提醒,N天后N
红眼over。
s******s
发帖数: 13035
43
当然了,原本不就是这样。我说的老大到老五有没有特异性,完全是等价的啊

【在 g*****g 的大作中提到】
: 如果你的逻辑成立,那么他们应该一起在同一时间死
:
: 眼。

s******s
发帖数: 13035
44
请看我前面的长贴。没有旅行者,他们是能够自圆其说的。或者说,
虽然每个人都知道有红眼,但是只要红眼的人不认为自己是红眼,
那么红眼1会认为 “红眼2认为 “红眼3认为 “红眼4认为 “红眼5认为没有人有红眼
””””
那么旅行者做的,就是从追后一个认为一点点打破这些假设

B,
后N

【在 s******8 的大作中提到】
: 我觉得旅行者的话只对一个和两个红眼有效,其他数目都无效。其他数目下红眼早在旅
: 行者上来前就死光了。除非他们在旅行者上来前不思考。
: 先说一个红眼,旅行者上来前这个红眼看不到任何红眼,所以他不知道有红眼这个事实
: ,以为全部是蓝眼,当然不会自杀。
: 两个红眼A,B。A看到一个红眼,第一天过后,没有自杀。那么A会认为只有一个红眼B,
: B不知道自己是红眼,所以不自杀。反之亦然。
: 三个红眼以上,那么所有人都应该知道有红眼,没有误解,不需要旅行者提醒,N天后N
: 红眼over。

f******b
发帖数: 1148
45
呵呵我想明白了
假设红眼者数量为n
旅行者说完后
只要第N天晚上没人自杀,就证明了红眼人数大于等于N+1
而对于红眼者来说,而他自己也看到了n-1个红眼者。
当n-1=N的的时候就会告诉红眼者自己肯定是红眼
所以第二天,也就是N+1的晚上自杀。
N+1=n
所以结论是对的。

生?

【在 l*3 的大作中提到】
: 转子知乎: http://www.zhihu.com/question/21262930
: 一个关于数学归纳法的悖论问题:到底是第N天有N个红眼睛自杀,还是什么都不会发生?
: 此问题最早据说是澳大利亚的华裔数学神童陶哲轩在网上贴出来让大家思考,逗大家玩
: 儿的。但却是真的把我难住了,一直百思不得其解。在此求教方家。
: 题目是这样的。说一个岛上有100个人,其中有5个红眼睛,95个蓝眼睛。这个岛有三个
: 奇怪的宗教规则。
: 1. 他们不能照镜子,不能看自己眼睛的颜色。
: 2. 他们不能告诉别人对方的眼睛是什么颜色。
: 3. 一旦有人知道了自己是红眼睛,他就必须在当天夜里自杀。
: 某天,有个旅行者到了这个岛上。由于不知道这里的规矩,所以他在和全岛人一起狂欢

c********h
发帖数: 7827
46
对,对于递推的第一步来说旅行者带来的信息是新的。

【在 S*E 的大作中提到】
: 情形一:只有一个红眼睛。有一个人不知道有红眼睛。
: 于是,旅行者带来了新信息。
: 情形二:有两个红眼睛。每个人都知道有红眼睛。
: 表面上,旅行者没有带来新信息。
: 但是从一个红眼人甲的角度看,他本来并不知道另一个
: 红眼人乙是否知道这个岛上有红眼睛。
: 而现在有了旅行者的话,那么甲就知道乙应该知道
: 岛上有红眼睛。这样旅行者就带来了新信息。
: 在红眼睛更多的情形下,就是反复嵌套“我知道”
: “你知道”“他知道”。旅行者的话改变了第N次

t*d
发帖数: 1290
47
你们都想得太多了。
假设岛上只有一只红眼。旅行者没来之前,这个红眼人不知道这个岛上有没有红眼人。
旅行者来了以后,他知道了。所以旅行者提供了信息。证毕。

生?

【在 l*3 的大作中提到】
: 转子知乎: http://www.zhihu.com/question/21262930
: 一个关于数学归纳法的悖论问题:到底是第N天有N个红眼睛自杀,还是什么都不会发生?
: 此问题最早据说是澳大利亚的华裔数学神童陶哲轩在网上贴出来让大家思考,逗大家玩
: 儿的。但却是真的把我难住了,一直百思不得其解。在此求教方家。
: 题目是这样的。说一个岛上有100个人,其中有5个红眼睛,95个蓝眼睛。这个岛有三个
: 奇怪的宗教规则。
: 1. 他们不能照镜子,不能看自己眼睛的颜色。
: 2. 他们不能告诉别人对方的眼睛是什么颜色。
: 3. 一旦有人知道了自己是红眼睛,他就必须在当天夜里自杀。
: 某天,有个旅行者到了这个岛上。由于不知道这里的规矩,所以他在和全岛人一起狂欢

l*3
发帖数: 2279
48
那你给列举下两个红眼的情形下, 是什么原因迫使他们确信自己是红眼然后自杀了?
比如说第0天发生了什么, 第一天发生了什么.

,

【在 x******s 的大作中提到】
: 当然会. 只要有一个大家都知道的游戏开始的时间, 必然会N人N天死. 两个人的话,
: 甚至不需要同步.

h*x
发帖数: 674
49
旅行者带来的信息量就是定义了第一天。
他说话之前,每个人都知道有红眼睛,每个人也知道别人知道有红眼睛(因为有5个红
眼睛的人,所以别人必然知道岛上有红眼睛的人)。这个条件不变。每个人只是不知道
自己是不是红眼睛。
从n=1天开始,每个人从自己看到的红眼睛人数和n的数值推断出自己是不是红眼睛,所
以n=1的定义是由旅游者带来的。
难道我28楼说的还不够清楚吗。。。

【在 c****p 的大作中提到】
: 考虑这样一个情况:
: 岛上只有一个红眼睛,那么旅行者说的话就是有用的信息,红眼睛当天晚上就会自杀
: 进而考虑有两个红眼睛,如果没有旅行者,那么两个红眼睛都不会自杀。
: 旅行者说了以后,两个红眼睛会在第二天晚上都自杀。
: 所以说这句话是有信息量的。
: 他说话之前,每个人都知道有红眼睛,但是他们都不知道别人是不是知道有红眼睛。
: 他说话之后,每个人不但知道有红眼睛,而且每个人都知道其他任何一个人都知道有红
: 眼睛。

h*x
发帖数: 674
50
一样的,第100天晚上集体自杀。
这个可以从只有一个蓝眼睛推起,其他红眼睛都看到一个蓝眼睛,所以他们至少可以睡
98天的安稳觉,这个蓝眼睛看到99个红眼睛,他可以睡99天安稳觉,那第99天晚上那99
个红眼睛自杀。
100个红眼睛就是大家一起等到99天,看看自己是不是那个蓝眼睛,但是发现没人自杀
。于是在第100天一起集体自杀。
也许你会问,如果100个都是红眼睛的话,那每个人都知道至少有99个人是红眼睛,是
不是可以跳过前98天的等待呢? 跳不过去的,因为每个人都不知道自己是不是蓝眼睛
,就不知道别人看到的是不是98个红眼睛和1个蓝眼睛,所以必须等到第99个晚上看有
没有人自杀。

【在 p**********t 的大作中提到】
: 如果都是红眼睛呢? 睡一觉起来每一个个体都认为自己是蓝眼睛,想在第二天看到自
: 杀的,但是没有看到。于是继续睡,继续每一天。

相关主题
大部分人不懂反证法在自己国家岛上杀死敌人侵略者应该不算犯罪,是英雄
每周1评:[2] 方舟子 的 16点动物园举办【我和红眼蝉】活动 (转载)
陕西出现红眼兽 (转载)[合集] 内贾德第一个在钓岛上声援中国 一席话太解气了!
进入WaterWorld版参与讨论
l*3
发帖数: 2279
51
你把三个红眼 (只有三个红眼, 并且没有外来人) 自杀的动机和心理历变过程给详细解
释一下...

B,
后N

【在 s******8 的大作中提到】
: 我觉得旅行者的话只对一个和两个红眼有效,其他数目都无效。其他数目下红眼早在旅
: 行者上来前就死光了。除非他们在旅行者上来前不思考。
: 先说一个红眼,旅行者上来前这个红眼看不到任何红眼,所以他不知道有红眼这个事实
: ,以为全部是蓝眼,当然不会自杀。
: 两个红眼A,B。A看到一个红眼,第一天过后,没有自杀。那么A会认为只有一个红眼B,
: B不知道自己是红眼,所以不自杀。反之亦然。
: 三个红眼以上,那么所有人都应该知道有红眼,没有误解,不需要旅行者提醒,N天后N
: 红眼over。

h*x
发帖数: 674
52
实在不知道你说啥,但我可以肯定地告诉你,只有一层不确定:不确定自己是不是红眼
睛。

【在 D*a 的大作中提到】
: 当有四个红眼睛的时候,他假设“假设只有三个人,那么三个人会假设只有两个人”,
: 这是两层假设了,只有“只有三个人”这个前提存在的条件下,三个人才会假设只有两
: 个人。如果有五个人,就是“假设只有四个人,这四个人会假设只有三个人,这三个人
: 会假设只有两个人”,这就是假设套假设套假设了。但是他不需要作出任何假设,他的
: 知识就可以满足游客给定的条件“岛上有红眼人”。
: 假设下雪的话,公交车就会晚点,假设公交车晚点的话,我就赶不上晚上的电影。那么
: 是不是我没赶上晚上的电影,就说明下雪了?

h*x
发帖数: 674
53
相信你看懂了我之前的解释。
你的k是第k天,N是N个红眼睛? 要证明的是:如果第K天没人自杀,那么N=k+1。
旅行者的话使任何N>=1的case成立。 并且旅行者的话定义了什么时候k=1.

【在 m*****n 的大作中提到】
: 这个说的透彻。换句话说,归纳法要证明的是:如果第K天没人自杀那么N > (K-1)
: 旅行者的话使K == 2的CASE成立。

h*x
发帖数: 674
54
如果有两个红眼睛,如果没有旅行者,当然不会自杀。
但如果有旅行者,这两个人就会在第二天晚上一起自杀。

【在 z****e 的大作中提到】
: 岛上只有一个红眼睛,那么旅行者说的话就是有用的信息,红眼睛当天晚上就会自杀
: 进而考虑有两个红眼睛,如果没有旅行者,那么两个红眼睛都不会自杀。
: n=2的时候,其实就会自杀了
: 所以条件是错的

h*x
发帖数: 674
55
当然每个人都知道 每个人都知道有红眼睛。 因为不止一个人有红眼睛。
旅行者的话没有改变大家的认识,旅行者只是给大家提供了这个岛上第三条宗教规则的
计时方法。

眼。

【在 s******s 的大作中提到】
: 前面好多人都说的是对的。开始的时候,大家都知道有红眼睛,但是并不是每个人都知
: 道 每个人都知道 每个人都知道 每个人都知道 每个人都知道有红眼睛。旅行者一句话
: 就把这个变成公共知识,每天让大家多一层认识。
: 换一种说法,那五个红眼睛,我给他们命名老大到老五。
: 1. 老五只看到四个红眼睛,他假设自己不是红眼,那么老五认为老四看到了三个红眼。
: 2. 老五认为老四假设自己不是红眼,而且前面说了,老五假设自己不是红眼,那么老
: 五认为“老四认为老三只看到了两个红眼”
: 以此类推,最后的结果是
: 老五认为
: 老四认为

h*x
发帖数: 674
56
谁忽略了? 我没忽略。。。
正因为不能交流,所以要由旅行者选定一个第一天,大家开始推断。

【在 c******n 的大作中提到】
: 楼上各位似乎忽略了第二条规矩,那就是他们彼此之间是不能交流的,即使你看到谁是
: ,你也不能告诉别人,题目给出的5个具体数据在这100个人中,从自认自己不是红眼的
: 角度来说,红眼的认为有4个,非红眼的认为有5个,从自认自己是红眼的角度来说,红
: 眼的认为有5个,非红眼的认为有6个,所以,结果应该是要么都没死,要么全死。
:
: 眼。

h*x
发帖数: 674
57
其他数目当然有效。 你要是不知道为什么其他数目没有在旅行者上来之前就死光了,
看我28楼回复。不重复了。

B,
后N

【在 s******8 的大作中提到】
: 我觉得旅行者的话只对一个和两个红眼有效,其他数目都无效。其他数目下红眼早在旅
: 行者上来前就死光了。除非他们在旅行者上来前不思考。
: 先说一个红眼,旅行者上来前这个红眼看不到任何红眼,所以他不知道有红眼这个事实
: ,以为全部是蓝眼,当然不会自杀。
: 两个红眼A,B。A看到一个红眼,第一天过后,没有自杀。那么A会认为只有一个红眼B,
: B不知道自己是红眼,所以不自杀。反之亦然。
: 三个红眼以上,那么所有人都应该知道有红眼,没有误解,不需要旅行者提醒,N天后N
: 红眼over。

h*x
发帖数: 674
58
如果你的第一步指第一天,那是对的。
如果你的第一步指岛上有一个红眼睛的情况,也不能算错,因为对于那个红眼睛,这个
信息是新的。但是题目已经是5个红眼睛了,所以不能解释原帖的问题。

【在 c********h 的大作中提到】
: 对,对于递推的第一步来说旅行者带来的信息是新的。
h*x
发帖数: 674
59
你的证明没错,但是跟原帖问题无关。

【在 t*d 的大作中提到】
: 你们都想得太多了。
: 假设岛上只有一只红眼。旅行者没来之前,这个红眼人不知道这个岛上有没有红眼人。
: 旅行者来了以后,他知道了。所以旅行者提供了信息。证毕。
:
: 生?

p*****y
发帖数: 1982
60
终于看明白了。
只有n=1时,旅行者直接带了信息。旅行者来之前那小子只知道红眼睛数量R为0或1,说
了这句话之后知道r只能为1,去死。
n=2时,旅行者说的话没有信息,因为两个红眼人都知道R=1或2。但是旅行者说了话之
后第二天对方没死这个现象带了信息,说明R!=1,只能等于2,两个人去死。
n=3是,旅行者的话和第二天没死人都没有带来信息,因为R=2或3。第三天早上发现还
没死人,这下有信息了,R!=2,大家gogogo

生?

【在 l*3 的大作中提到】
: 转子知乎: http://www.zhihu.com/question/21262930
: 一个关于数学归纳法的悖论问题:到底是第N天有N个红眼睛自杀,还是什么都不会发生?
: 此问题最早据说是澳大利亚的华裔数学神童陶哲轩在网上贴出来让大家思考,逗大家玩
: 儿的。但却是真的把我难住了,一直百思不得其解。在此求教方家。
: 题目是这样的。说一个岛上有100个人,其中有5个红眼睛,95个蓝眼睛。这个岛有三个
: 奇怪的宗教规则。
: 1. 他们不能照镜子,不能看自己眼睛的颜色。
: 2. 他们不能告诉别人对方的眼睛是什么颜色。
: 3. 一旦有人知道了自己是红眼睛,他就必须在当天夜里自杀。
: 某天,有个旅行者到了这个岛上。由于不知道这里的规矩,所以他在和全岛人一起狂欢

相关主题
靠~ 受不了,宇宙大得让我头晕[讨论]携带iPad进入中国内地要交税 (ZT)
人不能蒙着眼睛说瞎话[合集] [转] 红蓝眼睛的逻辑陷阱
神啊,我老公眼睛都直了,我这个恨啊!再贴几张一个外嫁后生的孩子的照片
进入WaterWorld版参与讨论
h*x
发帖数: 674
61
恩,对的。 如果你能回答为什么在旅行者到达之前岛上没有人自杀(那会他们也知道
岛上有红眼睛的人),你就应该完全懂了。

【在 p*****y 的大作中提到】
: 终于看明白了。
: 只有n=1时,旅行者直接带了信息。旅行者来之前那小子只知道红眼睛数量R为0或1,说
: 了这句话之后知道r只能为1,去死。
: n=2时,旅行者说的话没有信息,因为两个红眼人都知道R=1或2。但是旅行者说了话之
: 后第二天对方没死这个现象带了信息,说明R!=1,只能等于2,两个人去死。
: n=3是,旅行者的话和第二天没死人都没有带来信息,因为R=2或3。第三天早上发现还
: 没死人,这下有信息了,R!=2,大家gogogo
:
: 生?

h*x
发帖数: 674
62
最后我想问一下: 楼主你不迷茫了吧?
p**s
发帖数: 2707
63
不是你说的不清楚,是你说的根本不对,他带来的信息不是定义了第一天。
假如以前没有红眼要自杀的规定,旅行者来了以后,他当众宣布从当天开始实行这条自
杀的规定,这样也是定义了第一天,只要他不当众说,你们之间有红眼,这5个人还是
永远不会自杀。

【在 h*x 的大作中提到】
: 旅行者带来的信息量就是定义了第一天。
: 他说话之前,每个人都知道有红眼睛,每个人也知道别人知道有红眼睛(因为有5个红
: 眼睛的人,所以别人必然知道岛上有红眼睛的人)。这个条件不变。每个人只是不知道
: 自己是不是红眼睛。
: 从n=1天开始,每个人从自己看到的红眼睛人数和n的数值推断出自己是不是红眼睛,所
: 以n=1的定义是由旅游者带来的。
: 难道我28楼说的还不够清楚吗。。。

h*x
发帖数: 674
64
旅行者没有宣布任何规定,他只说了一句话:这个岛上有红眼睛的人。
那么你说,为什么在旅行者来之前,岛上的的规定没变,红眼睛的人数没变,却没有人
自杀?
在旅行者来之前,归纳法的第一天从何而来?

【在 p**s 的大作中提到】
: 不是你说的不清楚,是你说的根本不对,他带来的信息不是定义了第一天。
: 假如以前没有红眼要自杀的规定,旅行者来了以后,他当众宣布从当天开始实行这条自
: 杀的规定,这样也是定义了第一天,只要他不当众说,你们之间有红眼,这5个人还是
: 永远不会自杀。

h*x
发帖数: 674
65
你的第二段根本就没有考虑的必要。
“假如以前没有。。的规定”,为什么要这个假如? 以前有这个规定是前提条件,你
为什么要改前提条件? 你这样随意改动参数会让增加思考的难度,从而导致思路不清
晰。

【在 p**s 的大作中提到】
: 不是你说的不清楚,是你说的根本不对,他带来的信息不是定义了第一天。
: 假如以前没有红眼要自杀的规定,旅行者来了以后,他当众宣布从当天开始实行这条自
: 杀的规定,这样也是定义了第一天,只要他不当众说,你们之间有红眼,这5个人还是
: 永远不会自杀。

p**s
发帖数: 2707
66
没人自杀,是因为不是所有人都能确定所有人都知道所有人都知道所有人都知道所有人
都知道有红眼。

【在 h*x 的大作中提到】
: 旅行者没有宣布任何规定,他只说了一句话:这个岛上有红眼睛的人。
: 那么你说,为什么在旅行者来之前,岛上的的规定没变,红眼睛的人数没变,却没有人
: 自杀?
: 在旅行者来之前,归纳法的第一天从何而来?

p*****y
发帖数: 1982
67
你说的不对,知道规矩并不会自杀的。
假设两个红眼人被送上到了岛,上岛时候有个牌子上看到了这个规矩。然后两个人互相
看了下发现对方是红眼,心里想的是:这小子今晚该自杀了。
第二天起来发现对方还活蹦乱跳的,咋回事呢?想了一下,哦,他自己是红眼可他不知
道啊。没办法我又不能提醒他,就让他活着吧。
其实我觉得条件里应该说清楚不允许这些人交流。如果两个人里有个人说一句:“有人
该自杀阿”,这个局就破了。

【在 p**s 的大作中提到】
: 不是你说的不清楚,是你说的根本不对,他带来的信息不是定义了第一天。
: 假如以前没有红眼要自杀的规定,旅行者来了以后,他当众宣布从当天开始实行这条自
: 杀的规定,这样也是定义了第一天,只要他不当众说,你们之间有红眼,这5个人还是
: 永远不会自杀。

p**s
发帖数: 2707
68
为了说明你的“第一天”是错误的,如果你想不明白,那就让旅行者直接说,今天是第
一天,会不会有人自杀?

【在 h*x 的大作中提到】
: 你的第二段根本就没有考虑的必要。
: “假如以前没有。。的规定”,为什么要这个假如? 以前有这个规定是前提条件,你
: 为什么要改前提条件? 你这样随意改动参数会让增加思考的难度,从而导致思路不清
: 晰。

h*x
发帖数: 674
69
所有人都确定 所有人都知道有红眼。 因为有5个红眼。

【在 p**s 的大作中提到】
: 没人自杀,是因为不是所有人都能确定所有人都知道所有人都知道所有人都知道所有人
: 都知道有红眼。

p**s
发帖数: 2707
70
我没说知道规则会自杀,你这个例子正好是知道规则,也有第一天,但是不会自杀,正
好说明hyx的说法是错误的。

【在 p*****y 的大作中提到】
: 你说的不对,知道规矩并不会自杀的。
: 假设两个红眼人被送上到了岛,上岛时候有个牌子上看到了这个规矩。然后两个人互相
: 看了下发现对方是红眼,心里想的是:这小子今晚该自杀了。
: 第二天起来发现对方还活蹦乱跳的,咋回事呢?想了一下,哦,他自己是红眼可他不知
: 道啊。没办法我又不能提醒他,就让他活着吧。
: 其实我觉得条件里应该说清楚不允许这些人交流。如果两个人里有个人说一句:“有人
: 该自杀阿”,这个局就破了。

相关主题
再贴几张一个外嫁后生的孩子的照片大家承认吧蓝眼睛就是比粽眼睛好看 (转载)
娶妻三次, 三个女人致其性无能葛军,男,秒杀了52万江苏考生。。来做最后两题吧
中国北方人跟南方人基本上是两个种族,无法协调别做小学题了,来个初中的
进入WaterWorld版参与讨论
h*x
发帖数: 674
71
不会呀。 因为有5个红眼。 只有第5天这5个红眼才会一起自杀。
但是你这个第一天的概念是从哪里来的? 难道不是旅行者带来的?

【在 p**s 的大作中提到】
: 为了说明你的“第一天”是错误的,如果你想不明白,那就让旅行者直接说,今天是第
: 一天,会不会有人自杀?

p**s
发帖数: 2707
72
对,但是不是所有人都能确定所有人都知道所有人都知道所有人都知道所有人都知道有
红眼。

【在 h*x 的大作中提到】
: 所有人都确定 所有人都知道有红眼。 因为有5个红眼。
p**s
发帖数: 2707
73
错,如果旅行者只是宣布第一天,永远不会有人自杀。

【在 h*x 的大作中提到】
: 不会呀。 因为有5个红眼。 只有第5天这5个红眼才会一起自杀。
: 但是你这个第一天的概念是从哪里来的? 难道不是旅行者带来的?

h*x
发帖数: 674
74
你用这种方式表达有5个红眼的人是吧。
但是你忘了,蓝眼的人不知道是不是有6个红眼。
你自己带入一下吧。 旅行者说完这话以后,红眼的人不知道是有4个红眼,还是5个红
眼。 蓝眼的人不知道是有5个红眼,还是有6个红眼。 这个情况在旅行者说之前已经存
在。

【在 p**s 的大作中提到】
: 对,但是不是所有人都能确定所有人都知道所有人都知道所有人都知道所有人都知道有
: 红眼。

p**s
发帖数: 2707
75
这关蓝眼睛什么事。。。你完全可以考虑岛上只有5个红眼

【在 h*x 的大作中提到】
: 你用这种方式表达有5个红眼的人是吧。
: 但是你忘了,蓝眼的人不知道是不是有6个红眼。
: 你自己带入一下吧。 旅行者说完这话以后,红眼的人不知道是有4个红眼,还是5个红
: 眼。 蓝眼的人不知道是有5个红眼,还是有6个红眼。 这个情况在旅行者说之前已经存
: 在。

h*x
发帖数: 674
76
你这句话是对的。
但是你68楼说的是“如果让旅行者直接宣布今天是第一天”,并没有说“只是宣布第一
天啊”,我一直是在原题的条件下讨论问题的,也就是旅行者有说“岛上有红眼睛的人
”这句话。

【在 p**s 的大作中提到】
: 错,如果旅行者只是宣布第一天,永远不会有人自杀。
h*x
发帖数: 674
77
你代入一下吧。看看你是不是要增加一个“所有人都知道”。

【在 p**s 的大作中提到】
: 这关蓝眼睛什么事。。。你完全可以考虑岛上只有5个红眼
p*****y
发帖数: 1982
78
我觉得你这个套不对
对于5个红眼人的任意一个人A,他所知道的基本信息是
除了自己之外有95个蓝眼人,4个红眼人。情况1:如果自己是蓝眼人,就有一共4个红
眼人;情形2:自己是红眼人,一共有5个红眼人。
下面A试着去推其他一个红眼人的想法:
情形1自己是蓝眼,则其他红眼人(BCDE中任一个)能可看见3个红眼。该人也不能确定
自己的状态。他只能猜测有3个或4个红眼。
情形2自己是红眼,则其他红眼人可以看见4个红眼,由此可以猜测是4个或5个红眼。
BCDE的想法和A是一样的。只能推到这里了,不能套那么多。

【在 p**s 的大作中提到】
: 对,但是不是所有人都能确定所有人都知道所有人都知道所有人都知道所有人都知道有
: 红眼。

p**s
发帖数: 2707
79
因为你说旅行者带来的信息是定义了第一天,那就让他直接宣布第一天好了,可是这样
还是永远不会有人自杀的,而他说的“岛上有红眼睛的人”这句话,确实会让5个人在
第5天自杀,所以这句话带来的信息,并不是定义了第一天。

【在 h*x 的大作中提到】
: 你这句话是对的。
: 但是你68楼说的是“如果让旅行者直接宣布今天是第一天”,并没有说“只是宣布第一
: 天啊”,我一直是在原题的条件下讨论问题的,也就是旅行者有说“岛上有红眼睛的人
: ”这句话。

p*****y
发帖数: 1982
80
我看错了,不好意思太困了

【在 p**s 的大作中提到】
: 我没说知道规则会自杀,你这个例子正好是知道规则,也有第一天,但是不会自杀,正
: 好说明hyx的说法是错误的。

相关主题
问个奥数题每周1评:[2] 方舟子 的 16点
问个中文标点符号的嵌套问题陕西出现红眼兽 (转载)
大部分人不懂反证法在自己国家岛上杀死敌人侵略者应该不算犯罪,是英雄
进入WaterWorld版参与讨论
h*x
发帖数: 674
81
就是定义了第一天。注意,是旅行者那句话定义了第一天,不是旅行者定义了第一天。
同学,你在调整一个参数的时候,请fix其他参数,也就是旅行者说了“岛上有红眼的
人”这个条件不变。
至于他宣不宣布今天是第一天无所谓,整个归纳法就是从旅行者说那句话开始算起的,
也就是说,当旅行者说了那句话以后,岛上的人开始实行第三条宗教规则了。

【在 p**s 的大作中提到】
: 因为你说旅行者带来的信息是定义了第一天,那就让他直接宣布第一天好了,可是这样
: 还是永远不会有人自杀的,而他说的“岛上有红眼睛的人”这句话,确实会让5个人在
: 第5天自杀,所以这句话带来的信息,并不是定义了第一天。

p**s
发帖数: 2707
82
那他就直接说“今天是第一天”,这句话一样定义了第一天,第5天会不会有人自杀?
你就说会还是不会。

【在 h*x 的大作中提到】
: 就是定义了第一天。注意,是旅行者那句话定义了第一天,不是旅行者定义了第一天。
: 同学,你在调整一个参数的时候,请fix其他参数,也就是旅行者说了“岛上有红眼的
: 人”这个条件不变。
: 至于他宣不宣布今天是第一天无所谓,整个归纳法就是从旅行者说那句话开始算起的,
: 也就是说,当旅行者说了那句话以后,岛上的人开始实行第三条宗教规则了。

h*x
发帖数: 674
83
如果他说且只说“今天是第一天”,没有人会自杀,谁知道他这个第一天是什么。
如果他说且只说”从今天开始请大家执行岛上的三个规则,今天是第一天“,会有人自
杀。
如果他说且只说“岛上有红眼睛的人,从今天开始请大家开始执行岛上的三个规则,今
天是第一天”,会有人自杀。
如果他说且只说“岛上有红眼睛的人”,会有人自杀。

【在 p**s 的大作中提到】
: 那他就直接说“今天是第一天”,这句话一样定义了第一天,第5天会不会有人自杀?
: 你就说会还是不会。

p**s
发帖数: 2707
84

这个是错的,永远不会有人自杀。

【在 h*x 的大作中提到】
: 如果他说且只说“今天是第一天”,没有人会自杀,谁知道他这个第一天是什么。
: 如果他说且只说”从今天开始请大家执行岛上的三个规则,今天是第一天“,会有人自
: 杀。
: 如果他说且只说“岛上有红眼睛的人,从今天开始请大家开始执行岛上的三个规则,今
: 天是第一天”,会有人自杀。
: 如果他说且只说“岛上有红眼睛的人”,会有人自杀。

h*x
发帖数: 674
85
其实你这个问题挺好的。岛上的三个规则一直存在,只是岛上的人不知道从什么时候开
始执行这三个规则而已。所以我在28楼说,在这个旅行者来之前,岛上的人处于
periodic boundary condition,这个旅行者说了那句话之后,算执行那三个规则的第一
天。

【在 p**s 的大作中提到】
: 那他就直接说“今天是第一天”,这句话一样定义了第一天,第5天会不会有人自杀?
: 你就说会还是不会。

p**s
发帖数: 2707
86
那你说会不会有人自杀?

【在 h*x 的大作中提到】
: 其实你这个问题挺好的。岛上的三个规则一直存在,只是岛上的人不知道从什么时候开
: 始执行这三个规则而已。所以我在28楼说,在这个旅行者来之前,岛上的人处于
: periodic boundary condition,这个旅行者说了那句话之后,算执行那三个规则的第一
: 天。

h*x
发帖数: 674
87
很好,我们至少把分歧narrow到这个点。 不至于像前面那样到处飘忽。
如果旅行者说且只说”从今天开始请大家执行岛上的三个规则,今天是第一天“,会有
人自杀的。
因为红眼睛的看到了4个红眼,前4个晚上没有人自杀,到第5天,他明白了自己也是红
眼睛,所以第5天5个红眼一起自杀。
那你说说为什么永远不会有人自杀?

【在 p**s 的大作中提到】
: 那你说会不会有人自杀?
h*x
发帖数: 674
88
plus同学,现在距上个回帖已经15分钟了,我先去睡了,明天再来回。
l*3
发帖数: 2279
89
你这个有问题.
规则是一直在执行的. 规则是要求 "每个人严格知道自己是红眼后才在当晚自杀"
如果旅行者只说 "从今天开始执行规则, 今天是第一天", 那不会有任何影响, 因为在
这种情况下红眼无法严格知道自己是红眼.

【在 h*x 的大作中提到】
: 很好,我们至少把分歧narrow到这个点。 不至于像前面那样到处飘忽。
: 如果旅行者说且只说”从今天开始请大家执行岛上的三个规则,今天是第一天“,会有
: 人自杀的。
: 因为红眼睛的看到了4个红眼,前4个晚上没有人自杀,到第5天,他明白了自己也是红
: 眼睛,所以第5天5个红眼一起自杀。
: 那你说说为什么永远不会有人自杀?

l*3
发帖数: 2279
90
你先说清楚, 为什么旅行者不说 "岛上有红眼" 的情况下, 会有人自杀?

【在 h*x 的大作中提到】
: 很好,我们至少把分歧narrow到这个点。 不至于像前面那样到处飘忽。
: 如果旅行者说且只说”从今天开始请大家执行岛上的三个规则,今天是第一天“,会有
: 人自杀的。
: 因为红眼睛的看到了4个红眼,前4个晚上没有人自杀,到第5天,他明白了自己也是红
: 眼睛,所以第5天5个红眼一起自杀。
: 那你说说为什么永远不会有人自杀?

相关主题
动物园举办【我和红眼蝉】活动 (转载)人不能蒙着眼睛说瞎话
[合集] 内贾德第一个在钓岛上声援中国 一席话太解气了!神啊,我老公眼睛都直了,我这个恨啊!
靠~ 受不了,宇宙大得让我头晕[讨论]携带iPad进入中国内地要交税 (ZT)
进入WaterWorld版参与讨论
D*a
发帖数: 6830
91
所以就是我说的,整个推理就是假设套假设,就是你表示的,老五认为老四认为...
只有直接的知识回答不了的时候才需要引入假设来推理,但是红眼睛判断游客说的是对
是错,根本不需要用假设来判断。直接就可以判断对错的一句话,“我看到有红眼睛,
因此这句话是正确的”,一步推理可以得出的结论,为神马要引入假设来推理?

眼。

【在 s******s 的大作中提到】
: 前面好多人都说的是对的。开始的时候,大家都知道有红眼睛,但是并不是每个人都知
: 道 每个人都知道 每个人都知道 每个人都知道 每个人都知道有红眼睛。旅行者一句话
: 就把这个变成公共知识,每天让大家多一层认识。
: 换一种说法,那五个红眼睛,我给他们命名老大到老五。
: 1. 老五只看到四个红眼睛,他假设自己不是红眼,那么老五认为老四看到了三个红眼。
: 2. 老五认为老四假设自己不是红眼,而且前面说了,老五假设自己不是红眼,那么老
: 五认为“老四认为老三只看到了两个红眼”
: 以此类推,最后的结果是
: 老五认为
: 老四认为

p**********6
发帖数: 3408
92
这个题目其实不严密。因为首先,知道自己是红眼睛的必须在当天夜里自杀,但是题里
没说自杀就只有红眼睛这个原因。如果第五天晚上前有几个人因为其他原因默默自杀了
,其中有几个还是红眼睛,那第五天晚上剩下的红眼睛们也就没办法知道自己是红眼睛
了。
其次“岛上的人足够聪明,每个人都可以做出缜密的逻辑推理”这个条件很重要,而且
这点也得是公共知识,否则没人敢那样推理。这点题里也没说。
如果其实岛上100个人里有99个人的逻辑都很好,但有个人,他的逻辑就跟搞不懂这题
的人一样好,而且恰好只有他一个红眼睛,结果可能就会很惨,第二天晚上99个不是红
眼睛的人都自杀了,只剩下一个逻辑不好的红眼睛。所以有时逻辑不好也是一种福气。

生?

【在 l*3 的大作中提到】
: 转子知乎: http://www.zhihu.com/question/21262930
: 一个关于数学归纳法的悖论问题:到底是第N天有N个红眼睛自杀,还是什么都不会发生?
: 此问题最早据说是澳大利亚的华裔数学神童陶哲轩在网上贴出来让大家思考,逗大家玩
: 儿的。但却是真的把我难住了,一直百思不得其解。在此求教方家。
: 题目是这样的。说一个岛上有100个人,其中有5个红眼睛,95个蓝眼睛。这个岛有三个
: 奇怪的宗教规则。
: 1. 他们不能照镜子,不能看自己眼睛的颜色。
: 2. 他们不能告诉别人对方的眼睛是什么颜色。
: 3. 一旦有人知道了自己是红眼睛,他就必须在当天夜里自杀。
: 某天,有个旅行者到了这个岛上。由于不知道这里的规矩,所以他在和全岛人一起狂欢

s******8
发帖数: 4192
93
不能自圆其说。因为很简单,逻辑归纳是A推导出B,然后不是根据B再推出C.注意这里
“不是”。二
是A和B一起推出C。然后A,B,C推出D。
在你推出N=3结论时,把N=1的条件推翻了。所以N=3时的结论不成立,不能推出。后面
也就全部无效。

【在 s******s 的大作中提到】
: 请看我前面的长贴。没有旅行者,他们是能够自圆其说的。或者说,
: 虽然每个人都知道有红眼,但是只要红眼的人不认为自己是红眼,
: 那么红眼1会认为 “红眼2认为 “红眼3认为 “红眼4认为 “红眼5认为没有人有红眼
: ””””
: 那么旅行者做的,就是从追后一个认为一点点打破这些假设
:
: B,
: 后N

s******8
发帖数: 4192
94
当三个红眼在岛上第一天(上岛或者出生)睁开眼睛,发现岛上有红眼,然后仔细阅读
了游戏规则。
第三天晚上三个全部自杀。
前面都把旅行者出现作为第一天。实际上第三个红眼出现的时候才是真正的第一天,不
管他是和第一第二个一起出现的,还是自己单独出现的,还是后以后几个一起出现的。
第三个红眼才是真正的“外来者”。

【在 l*3 的大作中提到】
: 你把三个红眼 (只有三个红眼, 并且没有外来人) 自杀的动机和心理历变过程给详细解
: 释一下...
:
: B,
: 后N

s******s
发帖数: 13035
95
你错了。
"每个人都知道 每个人都知道有红眼睛" 不等于
“每个人都知道 每个人都知道 每个人都知道 每个人都知道 每个人都知道有红眼睛”
你不能简化这个关系的。必须一天一层的解。
旅行者不是提供了第一天的计时。计时本身没有意义,因为你看我的分析,每个人都可
以自圆其说,就算计时也不会改变。旅行者是提供了新的信息,这个信息就是我说的“
每个人都知道 每个人都知道 每个人都知道 每个人都知道 每个人都知道有红眼睛”
。我知道,你会说,所有人都知道有红眼,所以没有新的信息。问题是,我这个长句必
须连起来理解,不能中间简化。你再仔细读一下我的详细解释就知道了。我说清了了为
什么原来是可以自圆其说的,也从另一个角度说明了为什么是新信息。

【在 h*x 的大作中提到】
: 当然每个人都知道 每个人都知道有红眼睛。 因为不止一个人有红眼睛。
: 旅行者的话没有改变大家的认识,旅行者只是给大家提供了这个岛上第三条宗教规则的
: 计时方法。
:
: 眼。

s******s
发帖数: 13035
96
"我看到有红眼睛,因此这句话是正确的”
和”老五以为,老四以为,...., 老大以为没有红眼“没有矛盾,就算大家都看到四
个红眼,也没有矛盾。这句长句必须一起理解,并不是老二以为老大认为没有红眼,而
是从老五开始的整个过程。
我为什么写这么长的一个推理。正是因为原题lz的推理简单明了,但是不够具体,所以
早成了confusion。我这是具体而繁琐的从动力学上解释了,为什么之前不自杀,而旅
行者又具体带来了什么新信息

【在 D*a 的大作中提到】
: 所以就是我说的,整个推理就是假设套假设,就是你表示的,老五认为老四认为...
: 只有直接的知识回答不了的时候才需要引入假设来推理,但是红眼睛判断游客说的是对
: 是错,根本不需要用假设来判断。直接就可以判断对错的一句话,“我看到有红眼睛,
: 因此这句话是正确的”,一步推理可以得出的结论,为神马要引入假设来推理?
:
: 眼。

s******s
发帖数: 13035
97
你根本就没看懂。你写一下,我N=3的结论是什么,怎么把N=1推翻了?

【在 s******8 的大作中提到】
: 不能自圆其说。因为很简单,逻辑归纳是A推导出B,然后不是根据B再推出C.注意这里
: “不是”。二
: 是A和B一起推出C。然后A,B,C推出D。
: 在你推出N=3结论时,把N=1的条件推翻了。所以N=3时的结论不成立,不能推出。后面
: 也就全部无效。

l*3
发帖数: 2279
98
good one...

【在 p**********6 的大作中提到】
: 这个题目其实不严密。因为首先,知道自己是红眼睛的必须在当天夜里自杀,但是题里
: 没说自杀就只有红眼睛这个原因。如果第五天晚上前有几个人因为其他原因默默自杀了
: ,其中有几个还是红眼睛,那第五天晚上剩下的红眼睛们也就没办法知道自己是红眼睛
: 了。
: 其次“岛上的人足够聪明,每个人都可以做出缜密的逻辑推理”这个条件很重要,而且
: 这点也得是公共知识,否则没人敢那样推理。这点题里也没说。
: 如果其实岛上100个人里有99个人的逻辑都很好,但有个人,他的逻辑就跟搞不懂这题
: 的人一样好,而且恰好只有他一个红眼睛,结果可能就会很惨,第二天晚上99个不是红
: 眼睛的人都自杀了,只剩下一个逻辑不好的红眼睛。所以有时逻辑不好也是一种福气。
:

s******s
发帖数: 13035
99
还在纠结到底旅行者是定义了第一天的,还是按照我说的提供了新信息的。
仔细想想这个事情。
旅行者到了岛上,和所有人说岛上有红眼,但是不是和全岛人一起狂欢的时候大声说的
,而是狂欢和每个人跳舞的时候分别说的。这样同样定义了第一天,但是会有人自杀么
? 不会。
用只有两个红眼的例子作解释:之前之所以自杀,是因为B知道了A知道了有红眼,而A
没自杀,所以B第二天夜里自杀。现在你分别说有红眼,没有任何新信息,就算定义了
第一天,B也没有理由自杀,因为B不知道A知道了有红眼,所以不能根据A没自杀的事实
推理出任何东西。
所以区别明显就在当众说和分别说的区别上面,也就是我的”老五知道老四知道...."
的认识当中,而和所谓旅行者提供了第一天的定义毫无关系。

生?

【在 l*3 的大作中提到】
: 转子知乎: http://www.zhihu.com/question/21262930
: 一个关于数学归纳法的悖论问题:到底是第N天有N个红眼睛自杀,还是什么都不会发生?
: 此问题最早据说是澳大利亚的华裔数学神童陶哲轩在网上贴出来让大家思考,逗大家玩
: 儿的。但却是真的把我难住了,一直百思不得其解。在此求教方家。
: 题目是这样的。说一个岛上有100个人,其中有5个红眼睛,95个蓝眼睛。这个岛有三个
: 奇怪的宗教规则。
: 1. 他们不能照镜子,不能看自己眼睛的颜色。
: 2. 他们不能告诉别人对方的眼睛是什么颜色。
: 3. 一旦有人知道了自己是红眼睛,他就必须在当天夜里自杀。
: 某天,有个旅行者到了这个岛上。由于不知道这里的规矩,所以他在和全岛人一起狂欢

l*3
发帖数: 2279
100
你这没解释. 你只是吧 "三个红眼自杀" 这个事情又描述了一遍.
我是问你: 他们为什么自杀?什么样的心理活动和自我逻辑判断导致他们做出自杀的决
定?
他们如何判断出自己是红眼?

【在 s******8 的大作中提到】
: 当三个红眼在岛上第一天(上岛或者出生)睁开眼睛,发现岛上有红眼,然后仔细阅读
: 了游戏规则。
: 第三天晚上三个全部自杀。
: 前面都把旅行者出现作为第一天。实际上第三个红眼出现的时候才是真正的第一天,不
: 管他是和第一第二个一起出现的,还是自己单独出现的,还是后以后几个一起出现的。
: 第三个红眼才是真正的“外来者”。

相关主题
[合集] [转] 红蓝眼睛的逻辑陷阱中国北方人跟南方人基本上是两个种族,无法协调
再贴几张一个外嫁后生的孩子的照片大家承认吧蓝眼睛就是比粽眼睛好看 (转载)
娶妻三次, 三个女人致其性无能葛军,男,秒杀了52万江苏考生。。来做最后两题吧
进入WaterWorld版参与讨论
p**s
发帖数: 2707
101
错,这三个人永远不会自杀。除非有人告诉他们,岛上每一个人都知道“岛上每一个人
都知道有红眼”。
详细说,ABC三个红眼,C知道有红眼,因为他看到了AB
B也知道C知道有红眼,因为B能看到C看到A
但是A不知道"B知道C知道有红眼",因为A不知道自己是不是红眼。

【在 s******8 的大作中提到】
: 当三个红眼在岛上第一天(上岛或者出生)睁开眼睛,发现岛上有红眼,然后仔细阅读
: 了游戏规则。
: 第三天晚上三个全部自杀。
: 前面都把旅行者出现作为第一天。实际上第三个红眼出现的时候才是真正的第一天,不
: 管他是和第一第二个一起出现的,还是自己单独出现的,还是后以后几个一起出现的。
: 第三个红眼才是真正的“外来者”。

p**s
发帖数: 2707
102
这种逻辑题缺省都有所有人足够聪明,所有人足够理智的条件。你要觉得不严密,加上
好了,这不影响我们现在讨论。

【在 p**********6 的大作中提到】
: 这个题目其实不严密。因为首先,知道自己是红眼睛的必须在当天夜里自杀,但是题里
: 没说自杀就只有红眼睛这个原因。如果第五天晚上前有几个人因为其他原因默默自杀了
: ,其中有几个还是红眼睛,那第五天晚上剩下的红眼睛们也就没办法知道自己是红眼睛
: 了。
: 其次“岛上的人足够聪明,每个人都可以做出缜密的逻辑推理”这个条件很重要,而且
: 这点也得是公共知识,否则没人敢那样推理。这点题里也没说。
: 如果其实岛上100个人里有99个人的逻辑都很好,但有个人,他的逻辑就跟搞不懂这题
: 的人一样好,而且恰好只有他一个红眼睛,结果可能就会很惨,第二天晚上99个不是红
: 眼睛的人都自杀了,只剩下一个逻辑不好的红眼睛。所以有时逻辑不好也是一种福气。
:

s******8
发帖数: 4192
103
因为N=1的前提是这个红眼认为“这里没有红眼”。那么在N=3是,没有人会认为“没有
红眼”。N=1的前提不成立。N=3不能用N=1的结论,那么N=2推不出,N=3当然也推不出。

【在 s******s 的大作中提到】
: 你根本就没看懂。你写一下,我N=3的结论是什么,怎么把N=1推翻了?
s******8
发帖数: 4192
104
这么简单的逻辑。A当然知道"B知道C知道有红眼"。为什么不知道。因为有三个,最简
单的结论就是大家都知道“大家都知道”。

【在 p**s 的大作中提到】
: 错,这三个人永远不会自杀。除非有人告诉他们,岛上每一个人都知道“岛上每一个人
: 都知道有红眼”。
: 详细说,ABC三个红眼,C知道有红眼,因为他看到了AB
: B也知道C知道有红眼,因为B能看到C看到A
: 但是A不知道"B知道C知道有红眼",因为A不知道自己是不是红眼。

D*a
发帖数: 6830
105
你的推理就是我之前想过的,但是就是因为这种一环套一环的推理我再想过之后觉得是
有问题的,问题就是没有必要引入这种推理。做数学题的时候如果能正推,什么时候会
用假设去推?
你既然认为我说的跟你说的没有矛盾,那么我们两个是怎么推出来两个相反的结论的?
你并没有证明我的推理有错误啊。

【在 s******s 的大作中提到】
: "我看到有红眼睛,因此这句话是正确的”
: 和”老五以为,老四以为,...., 老大以为没有红眼“没有矛盾,就算大家都看到四
: 个红眼,也没有矛盾。这句长句必须一起理解,并不是老二以为老大认为没有红眼,而
: 是从老五开始的整个过程。
: 我为什么写这么长的一个推理。正是因为原题lz的推理简单明了,但是不够具体,所以
: 早成了confusion。我这是具体而繁琐的从动力学上解释了,为什么之前不自杀,而旅
: 行者又具体带来了什么新信息

l*3
发帖数: 2279
106
A知道B知道C知道有红眼 = 每个人都知道每个人都知道有红眼?
我怎么觉得你少了一层.

【在 s******8 的大作中提到】
: 这么简单的逻辑。A当然知道"B知道C知道有红眼"。为什么不知道。因为有三个,最简
: 单的结论就是大家都知道“大家都知道”。

D*a
发帖数: 6830
107
我觉得这个题在n=1,2,3的时候都是对的应该是没有异议了,但是再多了就有问题了
,n=4的时候每个人认为别的红眼都能看到两个红眼,也就是“每个人都知道每个人都
知道每个人都知道有红眼”,不需要再从n=1推。

A

【在 s******s 的大作中提到】
: 还在纠结到底旅行者是定义了第一天的,还是按照我说的提供了新信息的。
: 仔细想想这个事情。
: 旅行者到了岛上,和所有人说岛上有红眼,但是不是和全岛人一起狂欢的时候大声说的
: ,而是狂欢和每个人跳舞的时候分别说的。这样同样定义了第一天,但是会有人自杀么
: ? 不会。
: 用只有两个红眼的例子作解释:之前之所以自杀,是因为B知道了A知道了有红眼,而A
: 没自杀,所以B第二天夜里自杀。现在你分别说有红眼,没有任何新信息,就算定义了
: 第一天,B也没有理由自杀,因为B不知道A知道了有红眼,所以不能根据A没自杀的事实
: 推理出任何东西。
: 所以区别明显就在当众说和分别说的区别上面,也就是我的”老五知道老四知道...."

p**s
发帖数: 2707
108
这么简单的逻辑。A当然不知道"B知道C知道有红眼"。

【在 s******8 的大作中提到】
: 这么简单的逻辑。A当然知道"B知道C知道有红眼"。为什么不知道。因为有三个,最简
: 单的结论就是大家都知道“大家都知道”。

p**s
发帖数: 2707
109
是少了一层。

【在 l*3 的大作中提到】
: A知道B知道C知道有红眼 = 每个人都知道每个人都知道有红眼?
: 我怎么觉得你少了一层.

a****z
发帖数: 290
110
为什么很多人推到3就不推了? 怎么这么懒呢?
1-3个红眼命题是成立的. 4个红眼就不成立了. 不信的话谁推一个4个人然后自杀的,
我来挑出漏洞.

生?

【在 l*3 的大作中提到】
: 转子知乎: http://www.zhihu.com/question/21262930
: 一个关于数学归纳法的悖论问题:到底是第N天有N个红眼睛自杀,还是什么都不会发生?
: 此问题最早据说是澳大利亚的华裔数学神童陶哲轩在网上贴出来让大家思考,逗大家玩
: 儿的。但却是真的把我难住了,一直百思不得其解。在此求教方家。
: 题目是这样的。说一个岛上有100个人,其中有5个红眼睛,95个蓝眼睛。这个岛有三个
: 奇怪的宗教规则。
: 1. 他们不能照镜子,不能看自己眼睛的颜色。
: 2. 他们不能告诉别人对方的眼睛是什么颜色。
: 3. 一旦有人知道了自己是红眼睛,他就必须在当天夜里自杀。
: 某天,有个旅行者到了这个岛上。由于不知道这里的规矩,所以他在和全岛人一起狂欢

相关主题
葛军,男,秒杀了52万江苏考生。。来做最后两题吧问个中文标点符号的嵌套问题
别做小学题了,来个初中的大部分人不懂反证法
问个奥数题每周1评:[2] 方舟子 的 16点
进入WaterWorld版参与讨论
p**********6
发帖数: 3408
111
就算4个推得出来,他5个也推不出来,即便5个推得出来,6个他也推不出来,反正他总
会推不出来的。

【在 a****z 的大作中提到】
: 为什么很多人推到3就不推了? 怎么这么懒呢?
: 1-3个红眼命题是成立的. 4个红眼就不成立了. 不信的话谁推一个4个人然后自杀的,
: 我来挑出漏洞.
:
: 生?

l*3
发帖数: 2279
112
n=4时需要4层 "每个人都知道"
也就是 "每个人都知道每个人都知道每个人都知道每个人都知道有红眼"
如果没有外来人点破, 那至多只能推出3层 (也就是你说的), 而这是不够的.

【在 D*a 的大作中提到】
: 我觉得这个题在n=1,2,3的时候都是对的应该是没有异议了,但是再多了就有问题了
: ,n=4的时候每个人认为别的红眼都能看到两个红眼,也就是“每个人都知道每个人都
: 知道每个人都知道有红眼”,不需要再从n=1推。
:
: A

l**a
发帖数: 94
113
这是很老的逻辑题了,什么神童陶提出的,以前有一套四本数学大世界 的儿童读物就
有。 属于基本逻辑训练题目。
l*3
发帖数: 2279
114
来, 请找漏洞:
岛上有且仅有4个红眼人A,B,C,D, 外来人说了 "岛上有红眼" 之后. A开始思考:
如果我(即A)不是红眼, 那么岛上只有三个红眼BCD, 在这种情况下, 三天后BCD应该
都会自杀 (原因你已经说了, 3个红眼的命题是成立的)
结果三天后, BCD没有自杀. 于是第四天, A很无奈地自杀了. BCD同理.

【在 a****z 的大作中提到】
: 为什么很多人推到3就不推了? 怎么这么懒呢?
: 1-3个红眼命题是成立的. 4个红眼就不成立了. 不信的话谁推一个4个人然后自杀的,
: 我来挑出漏洞.
:
: 生?

c****p
发帖数: 6474
115
不止是计时开始。
想像这样一个情景:
第0天的时候,一股强大的力量把95个蓝和5个红放进了一个空无一人的村子。
没有旅行者的那句话的话,没有人会自杀。

【在 h*x 的大作中提到】
: 当然每个人都知道 每个人都知道有红眼睛。 因为不止一个人有红眼睛。
: 旅行者的话没有改变大家的认识,旅行者只是给大家提供了这个岛上第三条宗教规则的
: 计时方法。
:
: 眼。

a****z
发帖数: 290
116
所以说你懒啊。你要是不偷懒,那就从ABCD第一天的心理活动谈起. 哪有你这样穿越到
第三天分析的道理,这不是懒人是什么。
要不你稍微勤快一下?

【在 l*3 的大作中提到】
: 来, 请找漏洞:
: 岛上有且仅有4个红眼人A,B,C,D, 外来人说了 "岛上有红眼" 之后. A开始思考:
: 如果我(即A)不是红眼, 那么岛上只有三个红眼BCD, 在这种情况下, 三天后BCD应该
: 都会自杀 (原因你已经说了, 3个红眼的命题是成立的)
: 结果三天后, BCD没有自杀. 于是第四天, A很无奈地自杀了. BCD同理.

p**s
发帖数: 2707
117
你既然说前三天的分析没问题,他自然不用再把第一天到第三天都说一遍,你勤快可以
自己分析前三天,我们给你分析第四天。

【在 a****z 的大作中提到】
: 所以说你懒啊。你要是不偷懒,那就从ABCD第一天的心理活动谈起. 哪有你这样穿越到
: 第三天分析的道理,这不是懒人是什么。
: 要不你稍微勤快一下?

a****z
发帖数: 290
118
我没讲那么远。不要推五个,仔仔细细不要偷懒从第一天开始推出来四个成立就够了。

【在 p**********6 的大作中提到】
: 就算4个推得出来,他5个也推不出来,即便5个推得出来,6个他也推不出来,反正他总
: 会推不出来的。

a****z
发帖数: 290
119
我不用分析三天,第一天就够了。如果ABCD四个蓝眼睛,第一天A会想,我看到了三个
蓝眼睛,嗯,第一天我A可以拍拍胸脯对自己讲: "第一天铁定不会有人自杀". full
stop.

【在 p**s 的大作中提到】
: 你既然说前三天的分析没问题,他自然不用再把第一天到第三天都说一遍,你勤快可以
: 自己分析前三天,我们给你分析第四天。

l*3
发帖数: 2279
120
你不是说 "只有三个人, 外人一说, 三个人就会在第三天自杀" 这个没问题吗?
你都说没问题了, 你以为A比你还笨?
A第一天就想到这一点了, 然后等了三天, 结果悲剧的发现自己是红眼.

【在 a****z 的大作中提到】
: 所以说你懒啊。你要是不偷懒,那就从ABCD第一天的心理活动谈起. 哪有你这样穿越到
: 第三天分析的道理,这不是懒人是什么。
: 要不你稍微勤快一下?

相关主题
陕西出现红眼兽 (转载)[合集] 内贾德第一个在钓岛上声援中国 一席话太解气了!
在自己国家岛上杀死敌人侵略者应该不算犯罪,是英雄靠~ 受不了,宇宙大得让我头晕[讨论]
动物园举办【我和红眼蝉】活动 (转载)人不能蒙着眼睛说瞎话
进入WaterWorld版参与讨论
l*3
发帖数: 2279
121
你这么想也可以, 不过太麻烦了, 写起来长.
A足够聪明, 他可以这么想 "如果我是蓝眼, 那么只有三个红眼, 这三个红眼在听了外
人的话后第三天就会立马自杀"
你只要认同A所有的想法都没问题, 那么你就该认同 "A会在第四天自杀"

【在 a****z 的大作中提到】
: 我不用分析三天,第一天就够了。如果ABCD四个蓝眼睛,第一天A会想,我看到了三个
: 蓝眼睛,嗯,第一天我A可以拍拍胸脯对自己讲: "第一天铁定不会有人自杀". full
: stop.

a****z
发帖数: 290
122
, 现在我们讲的是四个人啊, 能套"只有三个人"条件下的结论吗?
别废话,四个人从第一天推。

【在 l*3 的大作中提到】
: 你不是说 "只有三个人, 外人一说, 三个人就会在第三天自杀" 这个没问题吗?
: 你都说没问题了, 你以为A比你还笨?
: A第一天就想到这一点了, 然后等了三天, 结果悲剧的发现自己是红眼.

l*3
发帖数: 2279
123
如果A是蓝眼, 那为什么不能套三个人的结论?

【在 a****z 的大作中提到】
: , 现在我们讲的是四个人啊, 能套"只有三个人"条件下的结论吗?
: 别废话,四个人从第一天推。

l*3
发帖数: 2279
124
关于不用这种 "归纳法" 的四个人的情况推演, 你可以看一下这个帖子:
http://www.zhihu.com/question/21262930
其中的第一个回答说的很详细.

【在 a****z 的大作中提到】
: , 现在我们讲的是四个人啊, 能套"只有三个人"条件下的结论吗?
: 别废话,四个人从第一天推。

a****z
发帖数: 290
125
废话,A是蓝眼,不就是BCD三个人红眼吗? 当然可以套三个人的结论。
我们不是讨论ABCD都是红眼吗。

【在 l*3 的大作中提到】
: 如果A是蓝眼, 那为什么不能套三个人的结论?
k*****e
发帖数: 1235
126
这个是正解!
旅行者带来的新信息是:我知道他知道他知道他知道他知道岛上有红眼睛。

【在 S*E 的大作中提到】
: 情形一:只有一个红眼睛。有一个人不知道有红眼睛。
: 于是,旅行者带来了新信息。
: 情形二:有两个红眼睛。每个人都知道有红眼睛。
: 表面上,旅行者没有带来新信息。
: 但是从一个红眼人甲的角度看,他本来并不知道另一个
: 红眼人乙是否知道这个岛上有红眼睛。
: 而现在有了旅行者的话,那么甲就知道乙应该知道
: 岛上有红眼睛。这样旅行者就带来了新信息。
: 在红眼睛更多的情形下,就是反复嵌套“我知道”
: “你知道”“他知道”。旅行者的话改变了第N次

k*****e
发帖数: 1235
127
你的逻辑真是不怎么样。

【在 z****e 的大作中提到】
: 会自杀
: 因为都看到对方是红眼睛了
: 所以大家都知道了
: 所以就等着有人自杀了
: 如果不自杀的话,就说明自己是
: 所以要死,旅行者还没来就都挂了

h*x
发帖数: 674
128
我就回你吧。
如果这个旅行者是跟每个人单独说的,确实事情没有任何改变,
但如果这个旅行者是跟所有人说了这么一句话,“从今天起我们开始执行岛上的三条宗
教规则,今天是第一天。” 你觉得会有人自杀吗?
我的答案是:会有人自杀。因为到第5天的时候红眼睛的人发现前天晚上没有人自杀,
于是知道自己也是红眼睛,于是第5天晚上所有红眼睛的人一起自杀。

A

【在 s******s 的大作中提到】
: 还在纠结到底旅行者是定义了第一天的,还是按照我说的提供了新信息的。
: 仔细想想这个事情。
: 旅行者到了岛上,和所有人说岛上有红眼,但是不是和全岛人一起狂欢的时候大声说的
: ,而是狂欢和每个人跳舞的时候分别说的。这样同样定义了第一天,但是会有人自杀么
: ? 不会。
: 用只有两个红眼的例子作解释:之前之所以自杀,是因为B知道了A知道了有红眼,而A
: 没自杀,所以B第二天夜里自杀。现在你分别说有红眼,没有任何新信息,就算定义了
: 第一天,B也没有理由自杀,因为B不知道A知道了有红眼,所以不能根据A没自杀的事实
: 推理出任何东西。
: 所以区别明显就在当众说和分别说的区别上面,也就是我的”老五知道老四知道...."

h*x
发帖数: 674
129
恩,这个假设也挺有意思的。
key point是,这100个人要达成共识,我们从哪天开始来执行这三个规则。

【在 c****p 的大作中提到】
: 不止是计时开始。
: 想像这样一个情景:
: 第0天的时候,一股强大的力量把95个蓝和5个红放进了一个空无一人的村子。
: 没有旅行者的那句话的话,没有人会自杀。

h*x
发帖数: 674
130
虽然在旅行者来之前这100个人也是在执行这三个规则的。
但是,在旅行者来之后改变了时间条件。
大家知道时间也是有方向的吧。定义时间方向的时候是以“事件”为参照物。在旅行者
来之前,这个时间的方向就像一个圆圈一样,没有头没有尾,对他们来讲,今天跟明天
有什么区别呢?反正我昨天看到这么多红眼睛,前天看见这么多红眼睛,大前天还是看
到这么多红眼睛。岛上的100个人就在这个光圈中执行三个规则,所有没有人自杀。
旅行者来之后,他提到红眼睛的事件使这个时间有了一个方向,不再是圆圈的,有了第
一天,第二天。。。所以每个人可以根据自己看到的红眼睛的数量和当天是第几天来推
断自己是不是红眼睛。

【在 h*x 的大作中提到】
: 恩,这个假设也挺有意思的。
: key point是,这100个人要达成共识,我们从哪天开始来执行这三个规则。

相关主题
神啊,我老公眼睛都直了,我这个恨啊!再贴几张一个外嫁后生的孩子的照片
携带iPad进入中国内地要交税 (ZT)娶妻三次, 三个女人致其性无能
[合集] [转] 红蓝眼睛的逻辑陷阱中国北方人跟南方人基本上是两个种族,无法协调
进入WaterWorld版参与讨论
a********6
发帖数: 21
131
re
p**s
发帖数: 2707
132
你的答案是错的。
如果想不清楚5个人,建议你想想3个人,或者2个人。

【在 h*x 的大作中提到】
: 我就回你吧。
: 如果这个旅行者是跟每个人单独说的,确实事情没有任何改变,
: 但如果这个旅行者是跟所有人说了这么一句话,“从今天起我们开始执行岛上的三条宗
: 教规则,今天是第一天。” 你觉得会有人自杀吗?
: 我的答案是:会有人自杀。因为到第5天的时候红眼睛的人发现前天晚上没有人自杀,
: 于是知道自己也是红眼睛,于是第5天晚上所有红眼睛的人一起自杀。
:
: A

p*****y
发帖数: 1982
133
你错了,不会有人自杀

【在 h*x 的大作中提到】
: 我就回你吧。
: 如果这个旅行者是跟每个人单独说的,确实事情没有任何改变,
: 但如果这个旅行者是跟所有人说了这么一句话,“从今天起我们开始执行岛上的三条宗
: 教规则,今天是第一天。” 你觉得会有人自杀吗?
: 我的答案是:会有人自杀。因为到第5天的时候红眼睛的人发现前天晚上没有人自杀,
: 于是知道自己也是红眼睛,于是第5天晚上所有红眼睛的人一起自杀。
:
: A

h*x
发帖数: 674
134
好吧,我看了你这个推断。旅行者把这个变成公共知识,这个说法是对的。
“这系列的基本假设就是,老五认为每人都认为自己不是红眼,这就是旅行者来这前的
所有红眼睛的想法。”怎么讲,你的推断是一个假设,这个假设成立的话,它只是所有
case中的一个可能性,但它不能完全解决这个问题。
你推断的过多了,老五凭什么认为每人都认为自己不是红眼? 事实上,这个岛上所有
人的心态应该是,不知道自己是不是红眼,就这么简单。
我觉得不能在“觉得别人觉得。。。”这个思路上做推断,在逻辑上,如果一个步骤既
不是其他步骤的充分条件,也不是其他步骤的必要条件,就没法推下去了。

眼。

【在 s******s 的大作中提到】
: 前面好多人都说的是对的。开始的时候,大家都知道有红眼睛,但是并不是每个人都知
: 道 每个人都知道 每个人都知道 每个人都知道 每个人都知道有红眼睛。旅行者一句话
: 就把这个变成公共知识,每天让大家多一层认识。
: 换一种说法,那五个红眼睛,我给他们命名老大到老五。
: 1. 老五只看到四个红眼睛,他假设自己不是红眼,那么老五认为老四看到了三个红眼。
: 2. 老五认为老四假设自己不是红眼,而且前面说了,老五假设自己不是红眼,那么老
: 五认为“老四认为老三只看到了两个红眼”
: 以此类推,最后的结果是
: 老五认为
: 老四认为

p*****y
发帖数: 1982
135
说明你没看懂
4个人的时候,推理是这样的。对于其中一个红眼人来说,他知道另外三个红眼人,但
是不知道自己的状态,所以有两种可能。
1.自己也是红眼人,一共4个红眼;
2.自己不是红眼人,三个红眼和一个蓝眼。一个重要的推理是,这样情况等效于只有三
个红眼人在岛上。
到第三天还没人死亡这个事实,排除了情形2,只能是情形1。

【在 a****z 的大作中提到】
: 为什么很多人推到3就不推了? 怎么这么懒呢?
: 1-3个红眼命题是成立的. 4个红眼就不成立了. 不信的话谁推一个4个人然后自杀的,
: 我来挑出漏洞.
:
: 生?

k*****e
发帖数: 1235
136
不知道自己是不是红眼的情况下,要做逻辑推理,必然就是做假设,
1、假设自己是蓝眼,看是否和事实矛盾。
2、假设自己是红眼,看是否和事实矛盾。
(1和2无先后关系)
如果1和2的假设都无法推导出矛盾,那么就维持不知道的状态,
那么就每天重复1和2的假设思考,直到1或者2的假设里出现矛盾为止。
所以说,不是“凭什么认为每人都认为自己不是红眼”,而是“每个人都会做同样的假
设思考”。

【在 h*x 的大作中提到】
: 好吧,我看了你这个推断。旅行者把这个变成公共知识,这个说法是对的。
: “这系列的基本假设就是,老五认为每人都认为自己不是红眼,这就是旅行者来这前的
: 所有红眼睛的想法。”怎么讲,你的推断是一个假设,这个假设成立的话,它只是所有
: case中的一个可能性,但它不能完全解决这个问题。
: 你推断的过多了,老五凭什么认为每人都认为自己不是红眼? 事实上,这个岛上所有
: 人的心态应该是,不知道自己是不是红眼,就这么简单。
: 我觉得不能在“觉得别人觉得。。。”这个思路上做推断,在逻辑上,如果一个步骤既
: 不是其他步骤的充分条件,也不是其他步骤的必要条件,就没法推下去了。
:
: 眼。

D*a
发帖数: 6830
137
我发懒没数写了几个。n=4的时候,每个人都看到三个,也就是每个人都知道另外的人
也能看到红眼,所以“岛上有红眼”就成立了,不需要继续从n=1推理。

【在 l*3 的大作中提到】
: n=4时需要4层 "每个人都知道"
: 也就是 "每个人都知道每个人都知道每个人都知道每个人都知道有红眼"
: 如果没有外来人点破, 那至多只能推出3层 (也就是你说的), 而这是不够的.

h*x
发帖数: 674
138
好吧,那我们就说说有3个红眼睛的情况。(如果只有2个红眼睛不行,必须由旅行者说
"岛上有红眼睛",不然红眼睛的人会不知道另一个红眼睛看到的是99个蓝眼睛,还是98
个蓝眼睛)。
again,当我说旅行者带来的信息是定义了第一天,指的是在95个蓝眼睛,5个红眼睛的
情况下。
下面我们就由你提议的3个红眼睛说起。旅行者说“从今天开始我们执行岛上的三个规
则,今天是第一天。” 对于红眼睛的人而言,他们看到了2个红眼睛,知道其他两个红
眼睛的人至少看到了一个红眼睛,也知道蓝眼睛的人至少看到了2个红眼睛。于是他们
知道这个岛上每个人都知道岛上有红眼睛的人存在。对蓝眼睛的人而言,他们看到了有
3个红眼睛,也知道这三个红眼睛的人每个人都至少看到2个红眼睛,于是他们知道这个
岛上每个人都知道岛上有红眼睛的人存在。于是他们知道了如何执行这三个宗教规则。
又,题目假设这100个人的逻辑都非常好,他们知道如何用归纳法推断自己是不是红眼
睛。对于红眼睛的人而言,他知道,如果另外两个红眼睛都只看到一个红眼睛,他们俩
就应该一起在第二天的晚上自杀。不会有人在第一天晚上自杀,因为另外两个红眼睛至
少看到了一个红眼睛,如果他们只看到一个红眼睛,那么他们都在等对方第一天晚上自
杀,结果第一天晚上没人自杀,于是在第二天其他两个红眼睛会知道自己也是红眼睛。
如果其他两个红眼睛只看到一个红眼睛,那么这两个红眼睛应该一起在第二天晚上自杀
。但是第二天晚上没人自杀,于是红眼睛就知道了其他两个红眼睛至少看到两个红眼睛
,鉴于他自己只看到了两个红眼睛,于是他知道只有三个红眼睛,其中包括自己,于是
这三个红眼睛一起在第三天晚上自杀。证明完毕。

【在 p**s 的大作中提到】
: 你的答案是错的。
: 如果想不清楚5个人,建议你想想3个人,或者2个人。

S*E
发帖数: 3662
139
又,题目假设这100个人的逻辑都非常好,他们知道如何用归纳法推断自己是不是红眼
睛。对于红眼睛的人而言,他知道,如果另外两个红眼睛都只看到一个红眼睛,他们俩
就应该一起在第二天的晚上自杀。
你的这个论断不成立。

98

【在 h*x 的大作中提到】
: 好吧,那我们就说说有3个红眼睛的情况。(如果只有2个红眼睛不行,必须由旅行者说
: "岛上有红眼睛",不然红眼睛的人会不知道另一个红眼睛看到的是99个蓝眼睛,还是98
: 个蓝眼睛)。
: again,当我说旅行者带来的信息是定义了第一天,指的是在95个蓝眼睛,5个红眼睛的
: 情况下。
: 下面我们就由你提议的3个红眼睛说起。旅行者说“从今天开始我们执行岛上的三个规
: 则,今天是第一天。” 对于红眼睛的人而言,他们看到了2个红眼睛,知道其他两个红
: 眼睛的人至少看到了一个红眼睛,也知道蓝眼睛的人至少看到了2个红眼睛。于是他们
: 知道这个岛上每个人都知道岛上有红眼睛的人存在。对蓝眼睛的人而言,他们看到了有
: 3个红眼睛,也知道这三个红眼睛的人每个人都至少看到2个红眼睛,于是他们知道这个

p*****y
发帖数: 1982
140
我知道你错在哪里了,
你在推3个红眼睛时引用了n=2个红眼睛的推理。如果旅行者只说开始,不说别的,2个
红眼人不会自杀的。因为n=2的推理一定要引用n=1的推理。
n=1的时候,大家都知道吧。

98

【在 h*x 的大作中提到】
: 好吧,那我们就说说有3个红眼睛的情况。(如果只有2个红眼睛不行,必须由旅行者说
: "岛上有红眼睛",不然红眼睛的人会不知道另一个红眼睛看到的是99个蓝眼睛,还是98
: 个蓝眼睛)。
: again,当我说旅行者带来的信息是定义了第一天,指的是在95个蓝眼睛,5个红眼睛的
: 情况下。
: 下面我们就由你提议的3个红眼睛说起。旅行者说“从今天开始我们执行岛上的三个规
: 则,今天是第一天。” 对于红眼睛的人而言,他们看到了2个红眼睛,知道其他两个红
: 眼睛的人至少看到了一个红眼睛,也知道蓝眼睛的人至少看到了2个红眼睛。于是他们
: 知道这个岛上每个人都知道岛上有红眼睛的人存在。对蓝眼睛的人而言,他们看到了有
: 3个红眼睛,也知道这三个红眼睛的人每个人都至少看到2个红眼睛,于是他们知道这个

相关主题
中国北方人跟南方人基本上是两个种族,无法协调别做小学题了,来个初中的
大家承认吧蓝眼睛就是比粽眼睛好看 (转载)问个奥数题
葛军,男,秒杀了52万江苏考生。。来做最后两题吧问个中文标点符号的嵌套问题
进入WaterWorld版参与讨论
p*****y
发帖数: 1982
141

98
|
就在这里,他们不会等对方自杀的,在旅行者只说了“开始”的情况下。因为他们知道
对方一定无法判断。

【在 h*x 的大作中提到】
: 好吧,那我们就说说有3个红眼睛的情况。(如果只有2个红眼睛不行,必须由旅行者说
: "岛上有红眼睛",不然红眼睛的人会不知道另一个红眼睛看到的是99个蓝眼睛,还是98
: 个蓝眼睛)。
: again,当我说旅行者带来的信息是定义了第一天,指的是在95个蓝眼睛,5个红眼睛的
: 情况下。
: 下面我们就由你提议的3个红眼睛说起。旅行者说“从今天开始我们执行岛上的三个规
: 则,今天是第一天。” 对于红眼睛的人而言,他们看到了2个红眼睛,知道其他两个红
: 眼睛的人至少看到了一个红眼睛,也知道蓝眼睛的人至少看到了2个红眼睛。于是他们
: 知道这个岛上每个人都知道岛上有红眼睛的人存在。对蓝眼睛的人而言,他们看到了有
: 3个红眼睛,也知道这三个红眼睛的人每个人都至少看到2个红眼睛,于是他们知道这个

h*x
发帖数: 674
142
我就知道你们会这么说。
这就是为什么我前面不觉得那位同学可以用 所有人知道所有人知道 的嵌套方法。
整个归纳法是应该存在于每个人脑中的,对于红眼睛的人而言,他只要等第二天晚上有
没有人自杀就行了。而不是在第二天晚上才来推断n=2的情况。
前面有人假设如果100个都是红眼睛呢,我的答案是他们会在第100天一起自杀。并且前
面99个晚上跳不过去。

【在 p*****y 的大作中提到】
: 我知道你错在哪里了,
: 你在推3个红眼睛时引用了n=2个红眼睛的推理。如果旅行者只说开始,不说别的,2个
: 红眼人不会自杀的。因为n=2的推理一定要引用n=1的推理。
: n=1的时候,大家都知道吧。
:
: 98

p*****y
发帖数: 1982
143
我不赞同那个知道知道的嵌套,那个应该是对的,但是容易让人糊涂。。
这里题的特点是所有人都是知道“有红眼”这个信息,但是“有红眼”这个条件无法触
发自杀,因为自杀的条件不是有红眼而是“确定自己有红眼”。想要确定自己是不是红
眼,一定要推理,而且只能从n=1的情况开始推理。除此之外别无他法。
n=1的情况下有红眼的这个公共信息恰好被屏蔽了,没法推,这时候旅行者的话就发挥
了作用。旅行者只说开始,会导致所有的推理无法进行。
换句话说,假设存在一种推理可以绕开n=1的情况而确定自己是不是红眼,则旅行者的
话就是废话。

【在 h*x 的大作中提到】
: 我就知道你们会这么说。
: 这就是为什么我前面不觉得那位同学可以用 所有人知道所有人知道 的嵌套方法。
: 整个归纳法是应该存在于每个人脑中的,对于红眼睛的人而言,他只要等第二天晚上有
: 没有人自杀就行了。而不是在第二天晚上才来推断n=2的情况。
: 前面有人假设如果100个都是红眼睛呢,我的答案是他们会在第100天一起自杀。并且前
: 面99个晚上跳不过去。

p**s
发帖数: 2707
144
如果他们只看到一个红眼睛,那么他们都在等对方第一天晚上自杀,结果第一天晚上没
人自杀,于是在第二天其他两个红眼睛会知道自己也是红眼睛。
这个不对,你这句话是红眼A的推理,在他的推理里,B不知道C是否知道有红眼

98

【在 h*x 的大作中提到】
: 好吧,那我们就说说有3个红眼睛的情况。(如果只有2个红眼睛不行,必须由旅行者说
: "岛上有红眼睛",不然红眼睛的人会不知道另一个红眼睛看到的是99个蓝眼睛,还是98
: 个蓝眼睛)。
: again,当我说旅行者带来的信息是定义了第一天,指的是在95个蓝眼睛,5个红眼睛的
: 情况下。
: 下面我们就由你提议的3个红眼睛说起。旅行者说“从今天开始我们执行岛上的三个规
: 则,今天是第一天。” 对于红眼睛的人而言,他们看到了2个红眼睛,知道其他两个红
: 眼睛的人至少看到了一个红眼睛,也知道蓝眼睛的人至少看到了2个红眼睛。于是他们
: 知道这个岛上每个人都知道岛上有红眼睛的人存在。对蓝眼睛的人而言,他们看到了有
: 3个红眼睛,也知道这三个红眼睛的人每个人都至少看到2个红眼睛,于是他们知道这个

S*E
发帖数: 3662
145
知道知道的嵌套是用来解释“悖论”的,不是用来证明推理的。

【在 p*****y 的大作中提到】
: 我不赞同那个知道知道的嵌套,那个应该是对的,但是容易让人糊涂。。
: 这里题的特点是所有人都是知道“有红眼”这个信息,但是“有红眼”这个条件无法触
: 发自杀,因为自杀的条件不是有红眼而是“确定自己有红眼”。想要确定自己是不是红
: 眼,一定要推理,而且只能从n=1的情况开始推理。除此之外别无他法。
: n=1的情况下有红眼的这个公共信息恰好被屏蔽了,没法推,这时候旅行者的话就发挥
: 了作用。旅行者只说开始,会导致所有的推理无法进行。
: 换句话说,假设存在一种推理可以绕开n=1的情况而确定自己是不是红眼,则旅行者的
: 话就是废话。

h*x
发帖数: 674
146
n=1的情况是红眼睛在推断自己只要等第二个晚上有没有人自杀的时候就已经用上了。
对于红眼睛而言,他看到了两个红眼睛,他也知道另外两个红眼睛都至少看到了一个红
眼睛。
别忘了,n=1的情况还假设了红眼睛的人不知道有红眼睛的存在。 但是这个假设岛上所
有人都用不上对吧,因为所有人都知道岛上有红眼睛的存在。
这个n=1的情况是岛上所有人在用归纳法推断自己应该在第几个晚上等有没有人自杀的
时候就已经用上了的。这是归纳法的第一步,不是岛上人行动的第一步。

【在 p*****y 的大作中提到】
: 我不赞同那个知道知道的嵌套,那个应该是对的,但是容易让人糊涂。。
: 这里题的特点是所有人都是知道“有红眼”这个信息,但是“有红眼”这个条件无法触
: 发自杀,因为自杀的条件不是有红眼而是“确定自己有红眼”。想要确定自己是不是红
: 眼,一定要推理,而且只能从n=1的情况开始推理。除此之外别无他法。
: n=1的情况下有红眼的这个公共信息恰好被屏蔽了,没法推,这时候旅行者的话就发挥
: 了作用。旅行者只说开始,会导致所有的推理无法进行。
: 换句话说,假设存在一种推理可以绕开n=1的情况而确定自己是不是红眼,则旅行者的
: 话就是废话。

s*******r
发帖数: 15
147
应该是100个人在各自老家被分别告知游戏规定(而且知道所有参与者都是逻辑人,只是
没见过其他人).然后被teleport到岛上,但是刚到了岛上都昏迷不醒,有的先醒,有的
过了几天醒,有的昏迷了几周再醒。然后来了这个多事的旅行者,用广播对所有人说岛
上有红眼。然后,有N个红眼,就在N天后一到自杀。
p*****y
发帖数: 1982
148
推n=1一定要屏蔽有红眼的这个信息,否则的话,你就是对的,哈哈。

【在 h*x 的大作中提到】
: n=1的情况是红眼睛在推断自己只要等第二个晚上有没有人自杀的时候就已经用上了。
: 对于红眼睛而言,他看到了两个红眼睛,他也知道另外两个红眼睛都至少看到了一个红
: 眼睛。
: 别忘了,n=1的情况还假设了红眼睛的人不知道有红眼睛的存在。 但是这个假设岛上所
: 有人都用不上对吧,因为所有人都知道岛上有红眼睛的存在。
: 这个n=1的情况是岛上所有人在用归纳法推断自己应该在第几个晚上等有没有人自杀的
: 时候就已经用上了的。这是归纳法的第一步,不是岛上人行动的第一步。

h*x
发帖数: 674
149
所以我说不要用那个嵌套,归纳法根本就不是用嵌套的思路。
嵌套是每一层都是另一层的子集,归纳法根本就没有那么多子集。
当我们在归纳法用n=1的时候,是说客观上岛上只有一个红眼,这时候是什么情况,而
不是A知不知道B不知道C是否知道有红眼人。
再回到岛上有三个红眼睛的情况。对于红眼睛而言,他知道这个岛上要么有2个红眼睛
,要么有3个红眼睛,所以他只要先用n=2的结论就行了。虽然他在得出n=2的结论时是
通过n=1的假设完成的。

【在 p**s 的大作中提到】
: 如果他们只看到一个红眼睛,那么他们都在等对方第一天晚上自杀,结果第一天晚上没
: 人自杀,于是在第二天其他两个红眼睛会知道自己也是红眼睛。
: 这个不对,你这句话是红眼A的推理,在他的推理里,B不知道C是否知道有红眼
:
: 98

h*x
发帖数: 674
150
n=1的假设是每个人脑子里在考虑n=2该怎么办的时候就已经用过的。
当岛上有3个红眼睛的时候,红眼睛要考虑的是n=2该怎么办,或n=3该怎么办。当他们
考虑n=2该怎么办的时候,用的是归纳法,而这个归纳法确实是建立在n=1的假设上完成
的。
我本来就是对的嘛,真是的。

【在 p*****y 的大作中提到】
: 推n=1一定要屏蔽有红眼的这个信息,否则的话,你就是对的,哈哈。
相关主题
大部分人不懂反证法在自己国家岛上杀死敌人侵略者应该不算犯罪,是英雄
每周1评:[2] 方舟子 的 16点动物园举办【我和红眼蝉】活动 (转载)
陕西出现红眼兽 (转载)[合集] 内贾德第一个在钓岛上声援中国 一席话太解气了!
进入WaterWorld版参与讨论
p**s
发帖数: 2707
151
如果旅行者是同一天悄悄告诉所有人岛上有红眼,N=1的情况,会有人自杀,N>=2时就
永远不会。
旅行者两种做法的差别,就是知道知道的嵌套。

【在 p*****y 的大作中提到】
: 我不赞同那个知道知道的嵌套,那个应该是对的,但是容易让人糊涂。。
: 这里题的特点是所有人都是知道“有红眼”这个信息,但是“有红眼”这个条件无法触
: 发自杀,因为自杀的条件不是有红眼而是“确定自己有红眼”。想要确定自己是不是红
: 眼,一定要推理,而且只能从n=1的情况开始推理。除此之外别无他法。
: n=1的情况下有红眼的这个公共信息恰好被屏蔽了,没法推,这时候旅行者的话就发挥
: 了作用。旅行者只说开始,会导致所有的推理无法进行。
: 换句话说,假设存在一种推理可以绕开n=1的情况而确定自己是不是红眼,则旅行者的
: 话就是废话。

h*x
发帖数: 674
152
所以我是从n=3开始说的呀,而题目是n=5.
从n=3往后,旅行者带来的信息就是定义了第一天。

【在 p*****y 的大作中提到】
: 推n=1一定要屏蔽有红眼的这个信息,否则的话,你就是对的,哈哈。
D*a
发帖数: 6830
153
n=5的时候已经是每个人看到n>1个红眼了,他为什么要引入n=1的假设?
一般来说一道题,比如说,直角三角形中,有如何如何的一些边和线段形成一些东西,
然后,请证明ab垂直于cd,那么假设法证明,是假设ab不垂直于cd,然后利用题目的条
件,推来推去推出矛盾。如果我上来先假设,假设题目里面三角形是锐角三角形,推来
推去,推出来ab不垂直于cd,就能证明直角三角形中ab和cd垂直吗?

【在 h*x 的大作中提到】
: n=1的假设是每个人脑子里在考虑n=2该怎么办的时候就已经用过的。
: 当岛上有3个红眼睛的时候,红眼睛要考虑的是n=2该怎么办,或n=3该怎么办。当他们
: 考虑n=2该怎么办的时候,用的是归纳法,而这个归纳法确实是建立在n=1的假设上完成
: 的。
: 我本来就是对的嘛,真是的。

i**********o
发帖数: 5993
154
岛上的人根本没守规矩。否则5个红眼睛的早死光了。轮不到旅行者来看。
p*****y
发帖数: 1982
155
我说的就是n=3的情况,其中一个小子在假设n=2的情况。人懒没说清楚。

所以我是从n=3开始说的呀,而题目是n=5.
从n=3往后,旅行者带来的信息就是定义了第一天。

【在 h*x 的大作中提到】
: 所以我是从n=3开始说的呀,而题目是n=5.
: 从n=3往后,旅行者带来的信息就是定义了第一天。

p*****y
发帖数: 1982
156
解释了,知道岛上有红眼(还不止一个),但是不能判断自己的状态导致无法自杀,很
有觉悟的

【在 i**********o 的大作中提到】
: 岛上的人根本没守规矩。否则5个红眼睛的早死光了。轮不到旅行者来看。
h*x
发帖数: 674
157
而且没有“他们知道n=2”一说。
n=2时,红眼睛不知道n=1还是n=2,蓝眼睛不知道n=2还是n=3.
这个时候,旅行者必须说岛上有红眼睛的人,因为不是所有人都知道所有人都知道岛上
有红眼睛的人。
n=3时,红眼睛不知道n=2还是n=3,蓝眼睛不知道n=3还是n=4.
这个时候,旅行者只要定义一下第一天就可以了,因为所有人都知道所有人都知道岛上
有红眼睛的人。所谓n=1的情况,是他们在推归纳法的时候用到的,岛上所有人都不需
要等第一天发生了什么。他们只要等第二天晚上有没有人自杀就行了。
就好比如果岛上所有人都是红眼睛,那么这100个人必须等第99天晚上发生了什么,前
面的98天是跳不过去的。

【在 p*****y 的大作中提到】
: 解释了,知道岛上有红眼(还不止一个),但是不能判断自己的状态导致无法自杀,很
: 有觉悟的

h*x
发帖数: 674
158
他们知道对方可以通过自己看到的判断。 所以不要用那个嵌套。
在用归纳法时,“自己看到的红眼睛的人数”+“今天是第几天”=充分条件。

【在 p*****y 的大作中提到】
: 我说的就是n=3的情况,其中一个小子在假设n=2的情况。人懒没说清楚。
:
: 所以我是从n=3开始说的呀,而题目是n=5.
: 从n=3往后,旅行者带来的信息就是定义了第一天。

p*****y
发帖数: 1982
159
解释n=2是针对你这段话来的:
“对于红眼睛的人而言,他知道,如果另外两个红眼睛都只看到一个红眼睛,他们俩
就应该一起在第二天的晚上自杀。不会有人在第一天晚上自杀,因为另外两个红眼睛至
少看到了一个红眼睛,如果他们只看到一个红眼睛,那么他们都在等对方第一天晚上自
杀”
我说的是他们不会等对方自杀,他们知道对方会因为无法判断而活下去。
不回了阿,变成语言问题了。

【在 h*x 的大作中提到】
: 而且没有“他们知道n=2”一说。
: n=2时,红眼睛不知道n=1还是n=2,蓝眼睛不知道n=2还是n=3.
: 这个时候,旅行者必须说岛上有红眼睛的人,因为不是所有人都知道所有人都知道岛上
: 有红眼睛的人。
: n=3时,红眼睛不知道n=2还是n=3,蓝眼睛不知道n=3还是n=4.
: 这个时候,旅行者只要定义一下第一天就可以了,因为所有人都知道所有人都知道岛上
: 有红眼睛的人。所谓n=1的情况,是他们在推归纳法的时候用到的,岛上所有人都不需
: 要等第一天发生了什么。他们只要等第二天晚上有没有人自杀就行了。
: 就好比如果岛上所有人都是红眼睛,那么这100个人必须等第99天晚上发生了什么,前
: 面的98天是跳不过去的。

h*x
发帖数: 674
160
归纳法就是从n=1开始假设的,从而得出我看到几个红眼睛,就在第几天晚上等有没有
人自杀就行了的结论。
比如我看到有3个红眼睛,我就等第三天晚上有没有人自杀就行了。如果有,那我就不
是红眼睛。如果没有,那我就是红眼睛,我就第四天晚上自杀好了。
但是我怎么知道第三天晚上对我很重要呢? 这是我从归纳法得出来的,而归纳法是从n
=1开始假设的。 但实际中,我不需要看第一个晚上发生了什么,因为我已经看到了3个
红眼睛。

【在 D*a 的大作中提到】
: n=5的时候已经是每个人看到n>1个红眼了,他为什么要引入n=1的假设?
: 一般来说一道题,比如说,直角三角形中,有如何如何的一些边和线段形成一些东西,
: 然后,请证明ab垂直于cd,那么假设法证明,是假设ab不垂直于cd,然后利用题目的条
: 件,推来推去推出矛盾。如果我上来先假设,假设题目里面三角形是锐角三角形,推来
: 推去,推出来ab不垂直于cd,就能证明直角三角形中ab和cd垂直吗?

相关主题
靠~ 受不了,宇宙大得让我头晕[讨论]携带iPad进入中国内地要交税 (ZT)
人不能蒙着眼睛说瞎话[合集] [转] 红蓝眼睛的逻辑陷阱
神啊,我老公眼睛都直了,我这个恨啊!再贴几张一个外嫁后生的孩子的照片
进入WaterWorld版参与讨论
D*a
发帖数: 6830
161
归纳法是证明不管n=几的时候某个式子都成立,而不是问已知n=几的时候那个式子成立
不成立,你说的这种推理,完全忽略了已知条件。求证,n=2的时候,n^2=n+n,需要从n
=1开始推吗?直接把n=2代进去就行了。

从n

【在 h*x 的大作中提到】
: 归纳法就是从n=1开始假设的,从而得出我看到几个红眼睛,就在第几天晚上等有没有
: 人自杀就行了的结论。
: 比如我看到有3个红眼睛,我就等第三天晚上有没有人自杀就行了。如果有,那我就不
: 是红眼睛。如果没有,那我就是红眼睛,我就第四天晚上自杀好了。
: 但是我怎么知道第三天晚上对我很重要呢? 这是我从归纳法得出来的,而归纳法是从n
: =1开始假设的。 但实际中,我不需要看第一个晚上发生了什么,因为我已经看到了3个
: 红眼睛。

h*x
发帖数: 674
162
你这个还是一层层的嵌套思维啊。归纳法中只嵌套两层。没有更多了。
红眼睛不需要判断对方能不能判断,只要每个人都知道其他人都知道这个岛上有红眼睛
的存在,那么每个人都可以根据自己看到的红眼睛人数判断自己该怎么做。

【在 p*****y 的大作中提到】
: 解释n=2是针对你这段话来的:
: “对于红眼睛的人而言,他知道,如果另外两个红眼睛都只看到一个红眼睛,他们俩
: 就应该一起在第二天的晚上自杀。不会有人在第一天晚上自杀,因为另外两个红眼睛至
: 少看到了一个红眼睛,如果他们只看到一个红眼睛,那么他们都在等对方第一天晚上自
: 杀”
: 我说的是他们不会等对方自杀,他们知道对方会因为无法判断而活下去。
: 不回了阿,变成语言问题了。

h*x
发帖数: 674
163
你这说的不就是我说的吗。。。我说当红眼睛看到有2个红眼睛的时候,就等第二天晚
上发生了什么就行了。

从n

【在 D*a 的大作中提到】
: 归纳法是证明不管n=几的时候某个式子都成立,而不是问已知n=几的时候那个式子成立
: 不成立,你说的这种推理,完全忽略了已知条件。求证,n=2的时候,n^2=n+n,需要从n
: =1开始推吗?直接把n=2代进去就行了。
:
: 从n

D*a
发帖数: 6830
164
我说的是,当n=5,每个红眼睛都知道其他的红眼睛能看到别的红眼睛,根本不需要推
理,就可以由现有知识推出来游客说的话是正确的。n=1的时候,有一个人由现有知识
推出来游客说的是错误的,所以才有矛盾。
但是现在已知n=5,不需要从n=1开始推,因为人家没问你n=1的时候如何,只问你n=5的
时候如何。我不该用n=2举例子,我再换一个吧,请问,n^2=5n,在n=5的时候成立吗?
你管n=1的时候成立不成立干什么。

【在 h*x 的大作中提到】
: 你这说的不就是我说的吗。。。我说当红眼睛看到有2个红眼睛的时候,就等第二天晚
: 上发生了什么就行了。
:
: 从n

h*x
发帖数: 674
165
n=1是一个归纳法的第一步假设,从这个假设可以推出现实中n=k,k为任意值的结论。
为什么要管n=1成立不成立呢。 好,现在n=5,红眼睛看到4个红眼睛,蓝眼睛看到5个
红眼睛,但他们都不知道自己是不是红眼睛对吧。于是他们开始用归纳法,如果n=1的
时候怎样怎样,如果n=2的时候怎样怎样,然后发现当n=5的时候,他们只要看第5个晚
上发生了什么就行了。
这个时候注意,n=5是我们知道的情况,不是他们知道的情况。所以对于红眼睛而言,
他要等4个晚上发生了什么,才能推断自己要不要在第5个晚上自杀,对于蓝眼睛而言,
他要等第5个晚上发生了什么,才能推断自己要不要在第6个晚上自杀。然后红眼睛在第
4个晚上发现没有人自杀,他就在第5个晚上自杀了,蓝眼睛就不需要自杀了
你说的没错,n=k时,红眼睛要等第k-1个晚上发生了什么,蓝眼睛要等第k个晚上发生
了什么。他们不需要等第一个晚上发生什么。n=1只是他们脑子里完成归纳法用到的假
设。

【在 D*a 的大作中提到】
: 我说的是,当n=5,每个红眼睛都知道其他的红眼睛能看到别的红眼睛,根本不需要推
: 理,就可以由现有知识推出来游客说的话是正确的。n=1的时候,有一个人由现有知识
: 推出来游客说的是错误的,所以才有矛盾。
: 但是现在已知n=5,不需要从n=1开始推,因为人家没问你n=1的时候如何,只问你n=5的
: 时候如何。我不该用n=2举例子,我再换一个吧,请问,n^2=5n,在n=5的时候成立吗?
: 你管n=1的时候成立不成立干什么。

S*E
发帖数: 3662
166
问题就是:过了两天没有人自杀,红眼人也不能判断自己也是红眼。

【在 h*x 的大作中提到】
: 你这说的不就是我说的吗。。。我说当红眼睛看到有2个红眼睛的时候,就等第二天晚
: 上发生了什么就行了。
:
: 从n

h*x
发帖数: 674
167
过了两天没有人自杀,三个红眼睛就会在第三天一起自杀。
你们老在想,他知不知道他知不知道他知不知道怎样。。。
不要这样想,这不是归纳法,归纳法只嵌套两层:
如果n=k时怎样,n=k+1时就怎样,对吧。
他们不需要判断别人怎么想,因为题目假设他们都会归纳法。所有人都知道如果他们看
到有k个红眼睛,他们就需要在第k个晚上看有没有人自杀就行了。
只有一个前提条件是他们需要知道的,就是岛上所有人都知道有红眼睛的人存在。当岛
上有三个或三个以上红眼睛的时候,这个前提条件就已经满足了,剩下的就是从选定的
第一天开始执行了。

【在 S*E 的大作中提到】
: 问题就是:过了两天没有人自杀,红眼人也不能判断自己也是红眼。
A********8
发帖数: 5
168
牛!

95
有5

【在 z****e 的大作中提到】
: 陶哲轩说,这个旅行者事实上讲了一句废话,没有带来任何新的信息。因为这岛上有95
: 个蓝眼睛,5个红眼睛。每个人都知道这岛上有红眼睛的人。无非是蓝眼睛的人看到有5
: 个红眼睛,红眼睛的人看到有4个红眼睛而已。旅行者说的那句【岛上有红眼睛的人】
: ,没有输入任何新的信息,他说的就是岛上的人每天都看到的景象。所以哪怕岛上的人
: 思维再缜密严谨,也不会有任何自杀的情况发生。
: 红眼睛的人知道有4个红眼睛,但是他因为规矩之一的缘故
: 所以不能告诉其它四个红眼睛,这个窗户纸只能由旅行者来点破
: 所以如果旅行者不说,都不会死
: 一旦点破,那就是每个人根据自身的观察等n天
: 那这个n的数字可能是4也可能是5,对于红眼睛来说,因为他不知道是4还是5

D*a
发帖数: 6830
169
我同意从n=1推到n=100是没问题的,但是在n=5的时候,一个红眼睛判断这句话是否正
确需要不需要推理?不需要推理,他看到的跟这句话并没有矛盾,所以根本不需要用归
纳法去解决什么问题,因为没有什么矛盾需要解决。

【在 h*x 的大作中提到】
: n=1是一个归纳法的第一步假设,从这个假设可以推出现实中n=k,k为任意值的结论。
: 为什么要管n=1成立不成立呢。 好,现在n=5,红眼睛看到4个红眼睛,蓝眼睛看到5个
: 红眼睛,但他们都不知道自己是不是红眼睛对吧。于是他们开始用归纳法,如果n=1的
: 时候怎样怎样,如果n=2的时候怎样怎样,然后发现当n=5的时候,他们只要看第5个晚
: 上发生了什么就行了。
: 这个时候注意,n=5是我们知道的情况,不是他们知道的情况。所以对于红眼睛而言,
: 他要等4个晚上发生了什么,才能推断自己要不要在第5个晚上自杀,对于蓝眼睛而言,
: 他要等第5个晚上发生了什么,才能推断自己要不要在第6个晚上自杀。然后红眼睛在第
: 4个晚上发现没有人自杀,他就在第5个晚上自杀了,蓝眼睛就不需要自杀了
: 你说的没错,n=k时,红眼睛要等第k-1个晚上发生了什么,蓝眼睛要等第k个晚上发生

h*x
发帖数: 674
170
如果不用归纳法,n=5的时候,红眼睛和蓝眼睛应该分别怎么做?

【在 D*a 的大作中提到】
: 我同意从n=1推到n=100是没问题的,但是在n=5的时候,一个红眼睛判断这句话是否正
: 确需要不需要推理?不需要推理,他看到的跟这句话并没有矛盾,所以根本不需要用归
: 纳法去解决什么问题,因为没有什么矛盾需要解决。

相关主题
再贴几张一个外嫁后生的孩子的照片大家承认吧蓝眼睛就是比粽眼睛好看 (转载)
娶妻三次, 三个女人致其性无能葛军,男,秒杀了52万江苏考生。。来做最后两题吧
中国北方人跟南方人基本上是两个种族,无法协调别做小学题了,来个初中的
进入WaterWorld版参与讨论
S*E
发帖数: 3662
171
现在不用知道知道的嵌套,你就仔细地说明N=3的情况。
一个红眼人在过了两天后看还没有人自杀,为什么就能判断自己是红眼睛。
你不用引用别人的推理,就从头来写。

【在 h*x 的大作中提到】
: 过了两天没有人自杀,三个红眼睛就会在第三天一起自杀。
: 你们老在想,他知不知道他知不知道他知不知道怎样。。。
: 不要这样想,这不是归纳法,归纳法只嵌套两层:
: 如果n=k时怎样,n=k+1时就怎样,对吧。
: 他们不需要判断别人怎么想,因为题目假设他们都会归纳法。所有人都知道如果他们看
: 到有k个红眼睛,他们就需要在第k个晚上看有没有人自杀就行了。
: 只有一个前提条件是他们需要知道的,就是岛上所有人都知道有红眼睛的人存在。当岛
: 上有三个或三个以上红眼睛的时候,这个前提条件就已经满足了,剩下的就是从选定的
: 第一天开始执行了。

D*a
发帖数: 6830
172
他为什么要做什么?他看到的和他听到的没有矛盾啊。

【在 h*x 的大作中提到】
: 如果不用归纳法,n=5的时候,红眼睛和蓝眼睛应该分别怎么做?
h*x
发帖数: 674
173
首先,这个红眼人会归纳法。他开始在心里用归纳法推断自己该怎么做:
他假设,有一群人,如果只有一个人有红眼睛,那么这个红眼睛的人会在第一天夜里自
杀。如果有两个红眼睛,那么这两个红眼睛的人会在第二天看到没人自杀的情况下在第
二天夜里一起自杀。然后他得出一个结论,如果这群人中有k个人有红眼睛,那么这个k
个红眼睛的人会一起在第k个晚上自杀。
然后他把这个结论用到自己的情况中,他看到了2个红眼睛,这个时候他判断,这个岛
上要么有2个红眼睛的人,要么有3个红眼睛的人。于是他用到上面归纳法的结论,如果
这个岛上只有2个有红眼睛的人,这个两个红眼睛的人会一起在第二天晚上自杀,那我
只要等第二天晚上有没有人自杀就行了。结果他发现,第二天玩上没人自杀,于是他知
道自己也是红眼睛,然后他就在第三个晚上自杀了。

【在 S*E 的大作中提到】
: 现在不用知道知道的嵌套,你就仔细地说明N=3的情况。
: 一个红眼人在过了两天后看还没有人自杀,为什么就能判断自己是红眼睛。
: 你不用引用别人的推理,就从头来写。

h*x
发帖数: 674
174
你确实没有懂归纳法。你再去看一下一楼的详细过程吧。那个是数学家写的,对你们而
言应该比我写的更有说服力。

【在 D*a 的大作中提到】
: 他为什么要做什么?他看到的和他听到的没有矛盾啊。
S*E
发帖数: 3662
175

这个推理不正确。唯一的红眼睛不能判断自己是红眼睛。

【在 h*x 的大作中提到】
: 首先,这个红眼人会归纳法。他开始在心里用归纳法推断自己该怎么做:
: 他假设,有一群人,如果只有一个人有红眼睛,那么这个红眼睛的人会在第一天夜里自
: 杀。如果有两个红眼睛,那么这两个红眼睛的人会在第二天看到没人自杀的情况下在第
: 二天夜里一起自杀。然后他得出一个结论,如果这群人中有k个人有红眼睛,那么这个k
: 个红眼睛的人会一起在第k个晚上自杀。
: 然后他把这个结论用到自己的情况中,他看到了2个红眼睛,这个时候他判断,这个岛
: 上要么有2个红眼睛的人,要么有3个红眼睛的人。于是他用到上面归纳法的结论,如果
: 这个岛上只有2个有红眼睛的人,这个两个红眼睛的人会一起在第二天晚上自杀,那我
: 只要等第二天晚上有没有人自杀就行了。结果他发现,第二天玩上没人自杀,于是他知
: 道自己也是红眼睛,然后他就在第三个晚上自杀了。

D*a
发帖数: 6830
176
我同意你从n=1到n=k的推理没有问题,我不同意的是需要不需要用归纳法。
他用归纳法的动力在哪里?又没有说不知道自己眼睛颜色的要自杀。

【在 h*x 的大作中提到】
: 你确实没有懂归纳法。你再去看一下一楼的详细过程吧。那个是数学家写的,对你们而
: 言应该比我写的更有说服力。

D*a
发帖数: 6830
177
红眼睛看了一圈没看见有红眼睛,这样他看到的和游客看到的就出现了矛盾。

【在 S*E 的大作中提到】
:
: 这个推理不正确。唯一的红眼睛不能判断自己是红眼睛。

h*x
发帖数: 674
178
这个只是归纳法用到的假设,不是现实存在的事情。
现实中每个人都知道岛上有红眼人的存在,不存在那个唯一的红眼人要假设岛上没有红
眼人存在的情况。

【在 S*E 的大作中提到】
:
: 这个推理不正确。唯一的红眼睛不能判断自己是红眼睛。

S*E
发帖数: 3662
179
当你讨论n=1的情况时,唯一的红眼人不知道岛上有红眼人的存在。
你不能再回头引用n=3的情况。

【在 h*x 的大作中提到】
: 这个只是归纳法用到的假设,不是现实存在的事情。
: 现实中每个人都知道岛上有红眼人的存在,不存在那个唯一的红眼人要假设岛上没有红
: 眼人存在的情况。

S*E
发帖数: 3662
180
hyx假设没有游客提供的公开信息。

【在 D*a 的大作中提到】
: 红眼睛看了一圈没看见有红眼睛,这样他看到的和游客看到的就出现了矛盾。
相关主题
问个奥数题每周1评:[2] 方舟子 的 16点
问个中文标点符号的嵌套问题陕西出现红眼兽 (转载)
大部分人不懂反证法在自己国家岛上杀死敌人侵略者应该不算犯罪,是英雄
进入WaterWorld版参与讨论
h*x
发帖数: 674
181
当n=3时,岛上的每一个人,都知道了有红眼睛存在的事实。
红眼睛的人要考虑的是n=2或n=3的情况,蓝眼睛的人要考虑的是n=3或n=4的情况。
但他们每一个人,都要从n=1开始做归纳法,然后红眼睛的人得出他要从第二个晚上开
始观察的结论,蓝眼睛的人得出他要从第三个晚上开始观察的结论。
同学们,这个归纳法是存在每个人脑子里的,不是现实中要从n=1开始做起。现实中n=k。

【在 S*E 的大作中提到】
: hyx假设没有游客提供的公开信息。
h*x
发帖数: 674
182
这个“唯一的红眼人”是存在于归纳法中的,不存在于当n=3的时候岛上的情况。
这个n=1是岛上所有的人在做归纳法开始用到的假设,但对他们而言,他们要考虑的情
况是他们已经看到的红眼睛的人数。

【在 S*E 的大作中提到】
: 当你讨论n=1的情况时,唯一的红眼人不知道岛上有红眼人的存在。
: 你不能再回头引用n=3的情况。

D*a
发帖数: 6830
183
这样啊,那就是我跟他的分歧了,我觉得在不存在矛盾的前提下,不需要进行归纳法的
推理。

【在 S*E 的大作中提到】
: hyx假设没有游客提供的公开信息。
D*a
发帖数: 6830
184
这个假设已经是错误的了,因为每个人都看到了多于一个的红眼人。
数学归纳法的应用中,有n的取值范围。如果题干让你证明n>2的时候都成立,那么就不
需要考虑n=1的情况。现在题干已经告诉你每个人看到大于等于4个红眼人了,应该直接
n大于等于4开始推。
求证,当n ≥ 3 时,n^2 > 2n,你管n=1的时候干什么
同理,当每个人看到≥ 4个红眼人的时候,会发生什么?

【在 h*x 的大作中提到】
: 这个“唯一的红眼人”是存在于归纳法中的,不存在于当n=3的时候岛上的情况。
: 这个n=1是岛上所有的人在做归纳法开始用到的假设,但对他们而言,他们要考虑的情
: 况是他们已经看到的红眼睛的人数。

h*x
发帖数: 674
185
所以让你去看一下一楼的归纳法呀。 那个是数学家写的。
还有,是他们让我从n=3开始说起的。

【在 D*a 的大作中提到】
: 这个假设已经是错误的了,因为每个人都看到了多于一个的红眼人。
: 数学归纳法的应用中,有n的取值范围。如果题干让你证明n>2的时候都成立,那么就不
: 需要考虑n=1的情况。现在题干已经告诉你每个人看到大于等于4个红眼人了,应该直接
: n大于等于4开始推。
: 求证,当n ≥ 3 时,n^2 > 2n,你管n=1的时候干什么
: 同理,当每个人看到≥ 4个红眼人的时候,会发生什么?

S*E
发帖数: 3662
186
你想不通就算了吧。

【在 h*x 的大作中提到】
: 所以让你去看一下一楼的归纳法呀。 那个是数学家写的。
: 还有,是他们让我从n=3开始说起的。

h*x
发帖数: 674
187
我认为是你想不通。
但至少我们达成就算了的共识。 恩,算了吧,我去吃饭了。

【在 S*E 的大作中提到】
: 你想不通就算了吧。
D*a
发帖数: 6830
188
我直接回他吧,听听他怎么说

【在 h*x 的大作中提到】
: 所以让你去看一下一楼的归纳法呀。 那个是数学家写的。
: 还有,是他们让我从n=3开始说起的。

p*****y
发帖数: 1982
189
刚看了标准答案了,嵌套才是王道阿。
他前面不该用“知道”,这个词意太含混了,把我弄傻了。后面全改成"认为"和“自认
为”,我就清楚了。

【在 S*E 的大作中提到】
: 知道知道的嵌套是用来解释“悖论”的,不是用来证明推理的。
D*a
发帖数: 6830
190
但是我觉得,假设n=1已经是错误的了,因为每个人都看到了多于一个的红眼人。
数学归纳法的应用中,如果题干让证明n>2的时候都成立,那么就不
需要考虑n=1的情况。现在题干已经说了每个人看到大于等于4个红眼人了,应该直接n
大于等于4开始推。
求证,当n ≥ 3 时,n^2 > 2n,应该从n=3开始推,n=1的时候成立不成立都跟题目没
有关系。
同理,当每个人看到≥ 4个红眼人的时候,会发生什么?不应该从n=1开始推啊,n=1的
时候成立他哪天自杀也是不相干的。

95
有5

【在 z****e 的大作中提到】
: 陶哲轩说,这个旅行者事实上讲了一句废话,没有带来任何新的信息。因为这岛上有95
: 个蓝眼睛,5个红眼睛。每个人都知道这岛上有红眼睛的人。无非是蓝眼睛的人看到有5
: 个红眼睛,红眼睛的人看到有4个红眼睛而已。旅行者说的那句【岛上有红眼睛的人】
: ,没有输入任何新的信息,他说的就是岛上的人每天都看到的景象。所以哪怕岛上的人
: 思维再缜密严谨,也不会有任何自杀的情况发生。
: 红眼睛的人知道有4个红眼睛,但是他因为规矩之一的缘故
: 所以不能告诉其它四个红眼睛,这个窗户纸只能由旅行者来点破
: 所以如果旅行者不说,都不会死
: 一旦点破,那就是每个人根据自身的观察等n天
: 那这个n的数字可能是4也可能是5,对于红眼睛来说,因为他不知道是4还是5

相关主题
动物园举办【我和红眼蝉】活动 (转载)人不能蒙着眼睛说瞎话
[合集] 内贾德第一个在钓岛上声援中国 一席话太解气了!神啊,我老公眼睛都直了,我这个恨啊!
靠~ 受不了,宇宙大得让我头晕[讨论]携带iPad进入中国内地要交税 (ZT)
进入WaterWorld版参与讨论
h*x
发帖数: 674
191
哎,用嵌套的人可以回答一下这个问题:
如果岛上100个都是红眼睛,那他们是在第100天集体自杀,还是在第二天集体自杀?
这个问题可以看出到底该用归纳,还是该用嵌套。
我的答案是:他们会在第100天集体自杀。

【在 p*****y 的大作中提到】
: 刚看了标准答案了,嵌套才是王道阿。
: 他前面不该用“知道”,这个词意太含混了,把我弄傻了。后面全改成"认为"和“自认
: 为”,我就清楚了。

p*****y
发帖数: 1982
192
在旅行者说有红眼睛之后的100天集体自杀。

【在 h*x 的大作中提到】
: 哎,用嵌套的人可以回答一下这个问题:
: 如果岛上100个都是红眼睛,那他们是在第100天集体自杀,还是在第二天集体自杀?
: 这个问题可以看出到底该用归纳,还是该用嵌套。
: 我的答案是:他们会在第100天集体自杀。

h*x
发帖数: 674
193
难道旅行者不说有红眼睛,他们就不知道有红眼睛吗? 就不知道别人知道岛上有红眼
睛吗? 要这个旅行者说有红眼睛干吗?

【在 p*****y 的大作中提到】
: 在旅行者说有红眼睛之后的100天集体自杀。
p*****y
发帖数: 1982
194
wokao你又来了,他们知道有红眼睛但是不能推出有人会自杀。
逻辑是不可以跳步的,必须一环扣一环
A应该自杀只能由A确定自己是红眼睛得到。其他任何条件都不可以。

【在 h*x 的大作中提到】
: 难道旅行者不说有红眼睛,他们就不知道有红眼睛吗? 就不知道别人知道岛上有红眼
: 睛吗? 要这个旅行者说有红眼睛干吗?

h*x
发帖数: 674
195
wokao你少来了。即使他们知道有红眼睛,即使有红眼睛自杀,他们也不能通过该红眼
睛的自杀来判断自己是不是红眼睛。所以才要用归纳法的第几天有没有人自杀来判断,
所以才要有第几天的概念,所以那个旅行者带来的信息是什么时候是第一天。

【在 p*****y 的大作中提到】
: wokao你又来了,他们知道有红眼睛但是不能推出有人会自杀。
: 逻辑是不可以跳步的,必须一环扣一环
: A应该自杀只能由A确定自己是红眼睛得到。其他任何条件都不可以。

h*x
发帖数: 674
196
对的,逻辑不可以跳步。
但是你们假设了别人都不懂归纳法,然后自己在这用嵌套。
所以的一环扣一环的归纳法已经在第一天被岛上所有的人完成了,这个有异议吗?
当A看到k个红眼睛时,然后等到第k天还没有人自杀时,他就确定了自己是红眼睛,然
后在第k+1个晚上自杀了,这个不满足题目要求吗?

【在 p*****y 的大作中提到】
: wokao你又来了,他们知道有红眼睛但是不能推出有人会自杀。
: 逻辑是不可以跳步的,必须一环扣一环
: A应该自杀只能由A确定自己是红眼睛得到。其他任何条件都不可以。

h*x
发帖数: 674
197
他们不需要推出会不会有人自杀。 他们需要的是观察有没有人自杀。
对于其中的一个人而言,只要他看到k个红眼睛,那么他只要等第k个晚上有没有人自杀
就可以判断自己是不是红眼睛。 这就是归纳法。

【在 p*****y 的大作中提到】
: wokao你又来了,他们知道有红眼睛但是不能推出有人会自杀。
: 逻辑是不可以跳步的,必须一环扣一环
: A应该自杀只能由A确定自己是红眼睛得到。其他任何条件都不可以。

p*****y
发帖数: 1982
198
归纳法是对的,但是你用的归纳法是错的。

【在 h*x 的大作中提到】
: 对的,逻辑不可以跳步。
: 但是你们假设了别人都不懂归纳法,然后自己在这用嵌套。
: 所以的一环扣一环的归纳法已经在第一天被岛上所有的人完成了,这个有异议吗?
: 当A看到k个红眼睛时,然后等到第k天还没有人自杀时,他就确定了自己是红眼睛,然
: 后在第k+1个晚上自杀了,这个不满足题目要求吗?

h*x
发帖数: 674
199
我不懒,我已经竭尽全力用文字解释了。
红眼A可以用反证法,假设自己不是红眼,然后在第99天检查是不是其他人都自杀了,
来推理自己的假设是不是正确,这个我们没有异议。
那个旅行者只说开始,这个方法就不能用了吗?
你们不停地说他们不能推断别人会不会自杀,我的point是,没人在推断,大家都在观
察,在观察第99天有没有人自杀,如果没有人自杀 ,这个就跟自己的假设矛盾,也就
是你上面说的反证法。

【在 p*****y 的大作中提到】
: 归纳法是对的,但是你用的归纳法是错的。
h*x
发帖数: 674
200
我的归纳法没错。而且我的归纳法是他们在第一天就完成了所有的归纳法,只是在等待
第k天的到来。而不是第k天突然冒出了归纳法的新信息。
你们在假设这个岛上的人不会用归纳法。跟你们过一天推算一天。

【在 p*****y 的大作中提到】
: 归纳法是对的,但是你用的归纳法是错的。
相关主题
[合集] [转] 红蓝眼睛的逻辑陷阱中国北方人跟南方人基本上是两个种族,无法协调
再贴几张一个外嫁后生的孩子的照片大家承认吧蓝眼睛就是比粽眼睛好看 (转载)
娶妻三次, 三个女人致其性无能葛军,男,秒杀了52万江苏考生。。来做最后两题吧
进入WaterWorld版参与讨论
p*****y
发帖数: 1982
201
每个人都必须同时做观察和推断。因为用来和观察到的事实比较的那个对象(或者叫命
题),是推理才能得到的。

【在 h*x 的大作中提到】
: 我不懒,我已经竭尽全力用文字解释了。
: 红眼A可以用反证法,假设自己不是红眼,然后在第99天检查是不是其他人都自杀了,
: 来推理自己的假设是不是正确,这个我们没有异议。
: 那个旅行者只说开始,这个方法就不能用了吗?
: 你们不停地说他们不能推断别人会不会自杀,我的point是,没人在推断,大家都在观
: 察,在观察第99天有没有人自杀,如果没有人自杀 ,这个就跟自己的假设矛盾,也就
: 是你上面说的反证法。

h*x
发帖数: 674
202
请你再加几个限定语,每个人都在第一天做出了观察和推断,然后根据自己观察到的k
个红眼睛,以及推断出的第k个晚上对自己很重要,得到结论。所以什么时候到k很重要
,所以什么时候是1很重要。

【在 p*****y 的大作中提到】
: 每个人都必须同时做观察和推断。因为用来和观察到的事实比较的那个对象(或者叫命
: 题),是推理才能得到的。

p*****y
发帖数: 1982
203
好吧,我前面那篇没说清楚,自宫一下。
旅行者说“开始”的时候,他们不能得到第k个晚上对自己很重要,都乐呵呵地玩去了。
只有说“有红眼人”的时候,他们才能得到第k个晚上对自己很重要。然后从1到k-1该
干啥干啥,第k天过来看看其他人死了没。

k

【在 h*x 的大作中提到】
: 请你再加几个限定语,每个人都在第一天做出了观察和推断,然后根据自己观察到的k
: 个红眼睛,以及推断出的第k个晚上对自己很重要,得到结论。所以什么时候到k很重要
: ,所以什么时候是1很重要。

h*x
发帖数: 674
204
旅行者说“开始”,他们环顾一周,哦,我看到k(k>2)个红眼睛,根据归纳法,我要等
第k个晚上有没有人自杀(此处,他们不需要判断别人会不会自杀,他们是自己从n=1开
始归纳的,同时信任其他人也会这么归纳)。
我就说你们在假设他们不懂归纳法。

了。

【在 p*****y 的大作中提到】
: 好吧,我前面那篇没说清楚,自宫一下。
: 旅行者说“开始”的时候,他们不能得到第k个晚上对自己很重要,都乐呵呵地玩去了。
: 只有说“有红眼人”的时候,他们才能得到第k个晚上对自己很重要。然后从1到k-1该
: 干啥干啥,第k天过来看看其他人死了没。
:
: k

p*****y
发帖数: 1982
205
讨论过了,3个人的都时候,说“开始”,每个人都看见2个红眼人,但是无法得到“我
要等到第二个晚上看有没有人自杀”这回事。
咱俩在这种情况下推的不一样。

【在 h*x 的大作中提到】
: 旅行者说“开始”,他们环顾一周,哦,我看到k(k>2)个红眼睛,根据归纳法,我要等
: 第k个晚上有没有人自杀(此处,他们不需要判断别人会不会自杀,他们是自己从n=1开
: 始归纳的,同时信任其他人也会这么归纳)。
: 我就说你们在假设他们不懂归纳法。
:
: 了。

h*x
发帖数: 674
206
是讨论过了。我的观点是可以得到“等到第2个晚上看有没有人自杀”。
岛上每个人都在第一天independently推出了归纳法,也就是当他们看到k个红眼睛的时
候,第k天对他们很重要。
当k>2的时候,他们知道其他人会跟自己一起开始使用这个归纳法。因为每个人都知道
岛上存在红眼睛的人了,每个人都可以用那套归纳法了。
这是我在本帖回的最后一个贴。我的话已经开始循环了。

【在 p*****y 的大作中提到】
: 讨论过了,3个人的都时候,说“开始”,每个人都看见2个红眼人,但是无法得到“我
: 要等到第二个晚上看有没有人自杀”这回事。
: 咱俩在这种情况下推的不一样。

l*3
发帖数: 2279
207
为你逻辑拙计.
咱们现在不是说A要 "确定自己是红眼" 吗? 已经明确告诉你了, A很聪明. A如果自认
为是蓝眼, 那事实只能是 "岛上只有三个红眼, 并且这三个红眼和A一样聪明", 这必然
导致 "三天后这三个红眼自杀". 而事实上, 三天后这三个红眼没有自杀. 所以三天后,
A就知道自己也是红眼了.

【在 a****z 的大作中提到】
: 废话,A是蓝眼,不就是BCD三个人红眼吗? 当然可以套三个人的结论。
: 我们不是讨论ABCD都是红眼吗。

l*3
发帖数: 2279
208
wrongwrongwrong.
the point is: 你需要证明的是: "没有外来者的时候, 每个红眼最终会自杀." (1)
而不是 "没有外来者的时候, 每个人都知道每个人都知道岛上有红眼" (2)
你始终没搞清楚这两句话的区别, 没有外来者的时候, 你从(2)推导不出来(1).


【在 D*a 的大作中提到】
: 我发懒没数写了几个。n=4的时候,每个人都看到三个,也就是每个人都知道另外的人
: 也能看到红眼,所以“岛上有红眼”就成立了,不需要继续从n=1推理。

l*3
发帖数: 2279
209
别人的意思是: 你所谓的 "对于红眼睛的人而言,他知道,如果另外两个红眼睛都只看
到一个红眼睛,他们俩就应该一起在第二天的晚上自杀。" 这句话是错的.
你这相当于是在说 "岛上只有两个红眼的话, 根据规则, 他们 (如果逻辑足够好) 应该
在第二天晚上自杀"
请问你这个是怎么推出来的?

【在 h*x 的大作中提到】
: 我就知道你们会这么说。
: 这就是为什么我前面不觉得那位同学可以用 所有人知道所有人知道 的嵌套方法。
: 整个归纳法是应该存在于每个人脑中的,对于红眼睛的人而言,他只要等第二天晚上有
: 没有人自杀就行了。而不是在第二天晚上才来推断n=2的情况。
: 前面有人假设如果100个都是红眼睛呢,我的答案是他们会在第100天一起自杀。并且前
: 面99个晚上跳不过去。

D*a
发帖数: 6830
210
我为什么要证明"没有外来者的时候, 每个红眼最终会自杀" ?
题目只是说,知道了是红眼的才自杀,哪里说只要红眼最终就会自杀了?

【在 l*3 的大作中提到】
: wrongwrongwrong.
: the point is: 你需要证明的是: "没有外来者的时候, 每个红眼最终会自杀." (1)
: 而不是 "没有外来者的时候, 每个人都知道每个人都知道岛上有红眼" (2)
: 你始终没搞清楚这两句话的区别, 没有外来者的时候, 你从(2)推导不出来(1).
:

相关主题
葛军,男,秒杀了52万江苏考生。。来做最后两题吧问个中文标点符号的嵌套问题
别做小学题了,来个初中的大部分人不懂反证法
问个奥数题每周1评:[2] 方舟子 的 16点
进入WaterWorld版参与讨论
l*3
发帖数: 2279
211
你不要用你自己的语言把你自己绕进去.
你说的一堆 "周期边界条件","时间的方向性,"定义n=1" 一类其实都是搅屎棍, 你自己
把自己搅了. 你现在有一点根本绕不过去: "如果旅行者不将 "岛上有红眼" 的信息公
开, 为什么两个红眼的情形下, 他们会在第二天晚上自杀?"

【在 h*x 的大作中提到】
: 而且没有“他们知道n=2”一说。
: n=2时,红眼睛不知道n=1还是n=2,蓝眼睛不知道n=2还是n=3.
: 这个时候,旅行者必须说岛上有红眼睛的人,因为不是所有人都知道所有人都知道岛上
: 有红眼睛的人。
: n=3时,红眼睛不知道n=2还是n=3,蓝眼睛不知道n=3还是n=4.
: 这个时候,旅行者只要定义一下第一天就可以了,因为所有人都知道所有人都知道岛上
: 有红眼睛的人。所谓n=1的情况,是他们在推归纳法的时候用到的,岛上所有人都不需
: 要等第一天发生了什么。他们只要等第二天晚上有没有人自杀就行了。
: 就好比如果岛上所有人都是红眼睛,那么这100个人必须等第99天晚上发生了什么,前
: 面的98天是跳不过去的。

l*3
发帖数: 2279
212
你这是什么逻辑呢? 你这就是这种逻辑:
我们来证明n=n+1.
如何证明呢?
首先0=1, 于是n=0时成立,
设n=k时成立, 考虑n=k+1的情形,
根据 "a=b,则a+1=b+1" 这一加法规则, 在k=k+1两端同时加1, 得到了k+1=k+2,
于是n=k+1也成立.
综上, 对于所有n, 都有n=n+1.
--------
你都不考虑你的初始假设对不对就开始乱归纳了, 在这里的具体问题中, 你的错误假设
是 "如果只有两个人, 他们会在旅行者声称 "今天开始执行规则" 的两天后同时自杀",
你从来没有论证清楚你这句话为什么是对的.

从n

【在 h*x 的大作中提到】
: 归纳法就是从n=1开始假设的,从而得出我看到几个红眼睛,就在第几天晚上等有没有
: 人自杀就行了的结论。
: 比如我看到有3个红眼睛,我就等第三天晚上有没有人自杀就行了。如果有,那我就不
: 是红眼睛。如果没有,那我就是红眼睛,我就第四天晚上自杀好了。
: 但是我怎么知道第三天晚上对我很重要呢? 这是我从归纳法得出来的,而归纳法是从n
: =1开始假设的。 但实际中,我不需要看第一个晚上发生了什么,因为我已经看到了3个
: 红眼睛。

l*3
发帖数: 2279
213
"他假设,有一群人,如果只有一个人有红眼睛,那么这个红眼睛的人会在第一天夜里自
杀"
错的很愚蠢.
只有一个人, 没人告诉这红眼睛 "岛上有红眼", 为什么这人要自杀? 他有自虐妄想症
吗?

个k

【在 h*x 的大作中提到】
: 首先,这个红眼人会归纳法。他开始在心里用归纳法推断自己该怎么做:
: 他假设,有一群人,如果只有一个人有红眼睛,那么这个红眼睛的人会在第一天夜里自
: 杀。如果有两个红眼睛,那么这两个红眼睛的人会在第二天看到没人自杀的情况下在第
: 二天夜里一起自杀。然后他得出一个结论,如果这群人中有k个人有红眼睛,那么这个k
: 个红眼睛的人会一起在第k个晚上自杀。
: 然后他把这个结论用到自己的情况中,他看到了2个红眼睛,这个时候他判断,这个岛
: 上要么有2个红眼睛的人,要么有3个红眼睛的人。于是他用到上面归纳法的结论,如果
: 这个岛上只有2个有红眼睛的人,这个两个红眼睛的人会一起在第二天晚上自杀,那我
: 只要等第二天晚上有没有人自杀就行了。结果他发现,第二天玩上没人自杀,于是他知
: 道自己也是红眼睛,然后他就在第三个晚上自杀了。

h*x
发帖数: 674
214
好吧,我食言回帖。 借那谁自宫一下。
我的话没有相当于说“岛上只有两个红眼,他们就会自杀”. n=1,n=2的情形是岛上的
人在自己独立做归纳法的时候完成的,从而让他们得出当他们看到2个红眼睛的人时,
要等第2天晚上。
现在说的就是n>2的情形,n>2的时候,所有人都知道其他人都知道岛上有红眼睛这个条
件,对于n=2的情形不适用。

【在 l*3 的大作中提到】
: 别人的意思是: 你所谓的 "对于红眼睛的人而言,他知道,如果另外两个红眼睛都只看
: 到一个红眼睛,他们俩就应该一起在第二天的晚上自杀。" 这句话是错的.
: 你这相当于是在说 "岛上只有两个红眼的话, 根据规则, 他们 (如果逻辑足够好) 应该
: 在第二天晚上自杀"
: 请问你这个是怎么推出来的?

h*x
发帖数: 674
215
你才蠢!
归纳法归纳法归纳法,你在做这个题目的归纳法的时候你会自杀吗?!
我不说了如果旅行者不通知所有人有红眼睛的时候,必须n>2才能做吗?! 因为n>2 的
时候所有人都知道其他人都知道这个岛上有红眼睛!
我没有把n=1的时候要有人告诉他们岛上有红眼睛这个条件说出来,是因为我assume大
家都会归纳法!
你才蠢!

里自

【在 l*3 的大作中提到】
: "他假设,有一群人,如果只有一个人有红眼睛,那么这个红眼睛的人会在第一天夜里自
: 杀"
: 错的很愚蠢.
: 只有一个人, 没人告诉这红眼睛 "岛上有红眼", 为什么这人要自杀? 他有自虐妄想症
: 吗?
:
: 个k

h*x
发帖数: 674
216
我从来没说清楚? 是你从来没想清楚。 再去把题目读一下!
我从来没有说过 "如果只有两个红眼睛, 他们会在旅行者声称 "今天开始执行规则" 的
两天后同时自杀",
我说的是如果多于2个红眼睛的时候! 那时候就不需要旅行者说岛上存在红眼睛了,因
为所有人都看得到红眼睛,所有人都知道别人也至少能看到的一个红眼睛。
这个就是你帖子的原问题!

【在 l*3 的大作中提到】
: 你这是什么逻辑呢? 你这就是这种逻辑:
: 我们来证明n=n+1.
: 如何证明呢?
: 首先0=1, 于是n=0时成立,
: 设n=k时成立, 考虑n=k+1的情形,
: 根据 "a=b,则a+1=b+1" 这一加法规则, 在k=k+1两端同时加1, 得到了k+1=k+2,
: 于是n=k+1也成立.
: 综上, 对于所有n, 都有n=n+1.
: --------
: 你都不考虑你的初始假设对不对就开始乱归纳了, 在这里的具体问题中, 你的错误假设

l*3
发帖数: 2279
217
本来就是 "在第一天就完成了归纳法", 甚至可以说是 "立刻就完成了归纳法", 因为他
们都 "足够聪明"
人家说的是你归纳法错了, 你不要用你错误的归纳法去揣测岛上那些 "足够聪明的人"
的想法.

【在 h*x 的大作中提到】
: 我的归纳法没错。而且我的归纳法是他们在第一天就完成了所有的归纳法,只是在等待
: 第k天的到来。而不是第k天突然冒出了归纳法的新信息。
: 你们在假设这个岛上的人不会用归纳法。跟你们过一天推算一天。

h*x
发帖数: 674
218
是呀,立刻完成了归纳法。 对于岛上有三个红眼睛的人而言,n=1,n=2的case是出现
在他们归纳法中的,不是出现在现实生活中的,懂?
既然你承认所有人完成了归纳法,那我问你,当岛上有三个红眼睛的人时,红眼睛的人
看到了另外两个红眼睛,他用他的归纳法应该得出什么结论?
你才错。

"

【在 l*3 的大作中提到】
: 本来就是 "在第一天就完成了归纳法", 甚至可以说是 "立刻就完成了归纳法", 因为他
: 们都 "足够聪明"
: 人家说的是你归纳法错了, 你不要用你错误的归纳法去揣测岛上那些 "足够聪明的人"
: 的想法.

l*3
发帖数: 2279
219
你自己套你自己.
你先搞清楚我问你问的是什么.
咱们再仔细讨论一遍三个红眼的情形, ok?
考虑A看到的视角, A如果是自认为是蓝眼, 那么A就是认为岛上只有两个红眼, 既然 "
如果只有两个红眼,他们会在旅行者声称 "今天开始执行规则" 的: 两天后同时自杀"
这句话不成立, 那么请问A凭什么在第三天自杀?

【在 h*x 的大作中提到】
: 我从来没说清楚? 是你从来没想清楚。 再去把题目读一下!
: 我从来没有说过 "如果只有两个红眼睛, 他们会在旅行者声称 "今天开始执行规则" 的
: 两天后同时自杀",
: 我说的是如果多于2个红眼睛的时候! 那时候就不需要旅行者说岛上存在红眼睛了,因
: 为所有人都看得到红眼睛,所有人都知道别人也至少能看到的一个红眼睛。
: 这个就是你帖子的原问题!

l*3
发帖数: 2279
220
引用------
我说的是如果多于2个红眼睛的时候! 那时候就不需要旅行者说岛上存在红眼睛了,因
为所有人都看得到红眼睛,所有人都知道别人也至少能看到的一个红眼睛。
---结束引用
你说的这个并没有问题, 但是请问你说的这个可以推出 "红眼最终会自杀" 吗? (在旅
行者神神叨叨的号称了一句 "大家, 从今天, 开始执行规则" 之后)

【在 h*x 的大作中提到】
: 我从来没说清楚? 是你从来没想清楚。 再去把题目读一下!
: 我从来没有说过 "如果只有两个红眼睛, 他们会在旅行者声称 "今天开始执行规则" 的
: 两天后同时自杀",
: 我说的是如果多于2个红眼睛的时候! 那时候就不需要旅行者说岛上存在红眼睛了,因
: 为所有人都看得到红眼睛,所有人都知道别人也至少能看到的一个红眼睛。
: 这个就是你帖子的原问题!

相关主题
陕西出现红眼兽 (转载)[合集] 内贾德第一个在钓岛上声援中国 一席话太解气了!
在自己国家岛上杀死敌人侵略者应该不算犯罪,是英雄靠~ 受不了,宇宙大得让我头晕[讨论]
动物园举办【我和红眼蝉】活动 (转载)人不能蒙着眼睛说瞎话
进入WaterWorld版参与讨论
l*3
发帖数: 2279
221
那你的point是什么?
你之前不是说, 即便没有外来者, 当红眼数目足够多的时候, 这些红眼最终会自杀么?
你之前难道不是这个意思? 你要是这个意思, 你难道不需要证明你说的话?

【在 D*a 的大作中提到】
: 我为什么要证明"没有外来者的时候, 每个红眼最终会自杀" ?
: 题目只是说,知道了是红眼的才自杀,哪里说只要红眼最终就会自杀了?

h*x
发帖数: 674
222
这就是我一直强调的,对A而言,他不是推测别人做什么,而是观测别人做什么,这两
点不一样ok? 你再把归纳法回忆一遍,里面是看到别人有没有自杀,再来决定自己要
不要自杀,还是推测别人要不要自杀,再来决定自己要不要自杀。
你这个嵌套里面有个错误,A自认为是蓝眼睛,他看到有两个红眼,他不能判断这两个
红眼会依照n=1的case做出决定,因为他不知道这两个红眼看到的是一个红眼,还是两
个红眼。
所以这个三个红眼没有谁嵌套谁,他们应该一起等第2个晚上有没有人自杀。

【在 l*3 的大作中提到】
: 你自己套你自己.
: 你先搞清楚我问你问的是什么.
: 咱们再仔细讨论一遍三个红眼的情形, ok?
: 考虑A看到的视角, A如果是自认为是蓝眼, 那么A就是认为岛上只有两个红眼, 既然 "
: 如果只有两个红眼,他们会在旅行者声称 "今天开始执行规则" 的: 两天后同时自杀"
: 这句话不成立, 那么请问A凭什么在第三天自杀?

h*x
发帖数: 674
223
会呀,因为他们都会归纳法! 你们自己在用嵌套,还以为人家也在用嵌套。

【在 l*3 的大作中提到】
: 引用------
: 我说的是如果多于2个红眼睛的时候! 那时候就不需要旅行者说岛上存在红眼睛了,因
: 为所有人都看得到红眼睛,所有人都知道别人也至少能看到的一个红眼睛。
: ---结束引用
: 你说的这个并没有问题, 但是请问你说的这个可以推出 "红眼最终会自杀" 吗? (在旅
: 行者神神叨叨的号称了一句 "大家, 从今天, 开始执行规则" 之后)

h*x
发帖数: 674
224
当岛上有三个红眼,每个红眼都看到有两个红眼时,这些红眼不应该认为别人是处在n=
2的case,因为他们不知道其他红眼看到的是一个红眼,还是两个红眼,如果别人是看
到一个红眼,那么他们会按照自己看到一个红眼的情况处理。如果别人看到的是两个红
眼,那么他们会按照自己看到两个红眼的情况处理。
事实上,每个红眼都看到了两个红眼,所以他们每个人都按照自己看到两个红眼的情况
处理。

【在 h*x 的大作中提到】
: 这就是我一直强调的,对A而言,他不是推测别人做什么,而是观测别人做什么,这两
: 点不一样ok? 你再把归纳法回忆一遍,里面是看到别人有没有自杀,再来决定自己要
: 不要自杀,还是推测别人要不要自杀,再来决定自己要不要自杀。
: 你这个嵌套里面有个错误,A自认为是蓝眼睛,他看到有两个红眼,他不能判断这两个
: 红眼会依照n=1的case做出决定,因为他不知道这两个红眼看到的是一个红眼,还是两
: 个红眼。
: 所以这个三个红眼没有谁嵌套谁,他们应该一起等第2个晚上有没有人自杀。

l*3
发帖数: 2279
225
那你是不是在说 "A观测到了两个红眼在两天后没有自杀, 所以A自杀了" ?
你给我一个A (观测到这个事实后) 在第三天自杀的理由好吧?
请只用正常语言逻辑, 不要自己造词, 不要扯什么 "周期边界条件" "定义了n=1" 这类
琐碎的东西, 三个人的情形不复杂吧? 你一句一句说也不会太长.
所以, 请你仔细描述清楚A为什么 "看到两个红眼在两天后没自杀, 于是在第三天自杀
了".

【在 h*x 的大作中提到】
: 这就是我一直强调的,对A而言,他不是推测别人做什么,而是观测别人做什么,这两
: 点不一样ok? 你再把归纳法回忆一遍,里面是看到别人有没有自杀,再来决定自己要
: 不要自杀,还是推测别人要不要自杀,再来决定自己要不要自杀。
: 你这个嵌套里面有个错误,A自认为是蓝眼睛,他看到有两个红眼,他不能判断这两个
: 红眼会依照n=1的case做出决定,因为他不知道这两个红眼看到的是一个红眼,还是两
: 个红眼。
: 所以这个三个红眼没有谁嵌套谁,他们应该一起等第2个晚上有没有人自杀。

c****p
发帖数: 6474
226
旅行者使时间有方向的关键还是因为他那句话泄露了信息。
假如他来了,只说了一句话:从今天起,你们开始执行你们的规则吧,之前发生了什么
都不算。
还是没人会自杀。

【在 h*x 的大作中提到】
: 虽然在旅行者来之前这100个人也是在执行这三个规则的。
: 但是,在旅行者来之后改变了时间条件。
: 大家知道时间也是有方向的吧。定义时间方向的时候是以“事件”为参照物。在旅行者
: 来之前,这个时间的方向就像一个圆圈一样,没有头没有尾,对他们来讲,今天跟明天
: 有什么区别呢?反正我昨天看到这么多红眼睛,前天看见这么多红眼睛,大前天还是看
: 到这么多红眼睛。岛上的100个人就在这个光圈中执行三个规则,所有没有人自杀。
: 旅行者来之后,他提到红眼睛的事件使这个时间有了一个方向,不再是圆圈的,有了第
: 一天,第二天。。。所以每个人可以根据自己看到的红眼睛的数量和当天是第几天来推
: 断自己是不是红眼睛。

h*x
发帖数: 674
227
当岛上有两个红眼时, 跟某个人看到两个红眼是两码事! 乱嵌套!
l*3
发帖数: 2279
228
1楼的归纳法的前提是建立在 "外人将 "岛上有红眼" 这一事实做了公开宣称" 之上的.
你后来讨论的 "外人只说 "从今天开始, 我们来执行规则" 了这一句话" , 根本和一楼
归纳的前提都不一样, 所以你不要乱用1楼归纳法的结论, 谢谢.

【在 h*x 的大作中提到】
: 这就是我一直强调的,对A而言,他不是推测别人做什么,而是观测别人做什么,这两
: 点不一样ok? 你再把归纳法回忆一遍,里面是看到别人有没有自杀,再来决定自己要
: 不要自杀,还是推测别人要不要自杀,再来决定自己要不要自杀。
: 你这个嵌套里面有个错误,A自认为是蓝眼睛,他看到有两个红眼,他不能判断这两个
: 红眼会依照n=1的case做出决定,因为他不知道这两个红眼看到的是一个红眼,还是两
: 个红眼。
: 所以这个三个红眼没有谁嵌套谁,他们应该一起等第2个晚上有没有人自杀。

h*x
发帖数: 674
229
给你一个理由? 我不是一直让你复习归纳法吗!
归纳法:
当n=1时,。。。
当n=2时,。。。
(你真的要我赘述吗? 一楼就是,自己复习)
结论,如果我看到有k个红眼,就在第k个晚上看有没有人自杀,如果没有,自己就是红
眼。
用到A这里,他看到两个红眼,结果第二天晚上没有人自杀(因为其他两个红眼也看到
两个红眼,他们也在等第二天晚上有没有人自杀,但他们不会在第二天晚上自杀),所
以第三天这三个红眼都知道自己也是红眼了。

【在 l*3 的大作中提到】
: 那你是不是在说 "A观测到了两个红眼在两天后没有自杀, 所以A自杀了" ?
: 你给我一个A (观测到这个事实后) 在第三天自杀的理由好吧?
: 请只用正常语言逻辑, 不要自己造词, 不要扯什么 "周期边界条件" "定义了n=1" 这类
: 琐碎的东西, 三个人的情形不复杂吧? 你一句一句说也不会太长.
: 所以, 请你仔细描述清楚A为什么 "看到两个红眼在两天后没自杀, 于是在第三天自杀
: 了".

l*3
发帖数: 2279
230
自己看227楼.

【在 h*x 的大作中提到】
: 给你一个理由? 我不是一直让你复习归纳法吗!
: 归纳法:
: 当n=1时,。。。
: 当n=2时,。。。
: (你真的要我赘述吗? 一楼就是,自己复习)
: 结论,如果我看到有k个红眼,就在第k个晚上看有没有人自杀,如果没有,自己就是红
: 眼。
: 用到A这里,他看到两个红眼,结果第二天晚上没有人自杀(因为其他两个红眼也看到
: 两个红眼,他们也在等第二天晚上有没有人自杀,但他们不会在第二天晚上自杀),所
: 以第三天这三个红眼都知道自己也是红眼了。

相关主题
神啊,我老公眼睛都直了,我这个恨啊!再贴几张一个外嫁后生的孩子的照片
携带iPad进入中国内地要交税 (ZT)娶妻三次, 三个女人致其性无能
[合集] [转] 红蓝眼睛的逻辑陷阱中国北方人跟南方人基本上是两个种族,无法协调
进入WaterWorld版参与讨论
h*x
发帖数: 674
231
归纳法当然是针对所有人都知道有红眼这个事实的基础上的。
当岛上有3个红眼的时候,即时外人只说了一句今天开始执行,岛上的人也知道有红眼
这个事实,并且知道别人都知道有红眼这个事实。
你应该在弄明白什么是归纳法以后,再弄明白怎么用归纳法,不谢。

的.

【在 l*3 的大作中提到】
: 1楼的归纳法的前提是建立在 "外人将 "岛上有红眼" 这一事实做了公开宣称" 之上的.
: 你后来讨论的 "外人只说 "从今天开始, 我们来执行规则" 了这一句话" , 根本和一楼
: 归纳的前提都不一样, 所以你不要乱用1楼归纳法的结论, 谢谢.

D*a
发帖数: 6830
232
晕死了我啥时候说过这话?
我说的是,
1
当有很多红眼的时候,所有人都知道岛上有红眼,别人告诉他们岛上有红眼和他们看到
的信息是一样的,没有矛盾就没有推理的必要,所以谁也不自杀。
2
如果要归纳法,那么所有人知道n=1是错误的,为什么要从n=1开始推?这个题目类似,
请证明,当n ≥ 3 时,n^2 > 2n,当然第一步是看n=3的时候成立不成立,而不是看n=
1的时候成立不成立,因为题目说了n大于1,n=1的时候成立不成立,都跟这题目没有什
么相干。

?

【在 l*3 的大作中提到】
: 那你的point是什么?
: 你之前不是说, 即便没有外来者, 当红眼数目足够多的时候, 这些红眼最终会自杀么?
: 你之前难道不是这个意思? 你要是这个意思, 你难道不需要证明你说的话?

R****6
发帖数: 16
233
Ssš
l*3
发帖数: 2279
234
噢, 那么你的结论是 "红颜足够多时, 就算外人公开宣称 "岛上有红眼", 也没人自杀"
?

n=

【在 D*a 的大作中提到】
: 晕死了我啥时候说过这话?
: 我说的是,
: 1
: 当有很多红眼的时候,所有人都知道岛上有红眼,别人告诉他们岛上有红眼和他们看到
: 的信息是一样的,没有矛盾就没有推理的必要,所以谁也不自杀。
: 2
: 如果要归纳法,那么所有人知道n=1是错误的,为什么要从n=1开始推?这个题目类似,
: 请证明,当n ≥ 3 时,n^2 > 2n,当然第一步是看n=3的时候成立不成立,而不是看n=
: 1的时候成立不成立,因为题目说了n大于1,n=1的时候成立不成立,都跟这题目没有什
: 么相干。

h*x
发帖数: 674
235
如果岛上的红眼睛的人数大于2,就可以。
因为大家都知道有红眼睛了。也知道别人都知道有红眼睛了。
然后只要根据自己看到的红眼睛的人数在相应的第几天等就可以了。

【在 c****p 的大作中提到】
: 旅行者使时间有方向的关键还是因为他那句话泄露了信息。
: 假如他来了,只说了一句话:从今天起,你们开始执行你们的规则吧,之前发生了什么
: 都不算。
: 还是没人会自杀。

p**s
发帖数: 2707
236
错,归纳法是建立在所有人都知道所有人都知道所有人都知道所有人都知道。。。。有
红眼的基础上。离开这个你理解不了的嵌套,归纳法就是错的。

【在 h*x 的大作中提到】
: 归纳法当然是针对所有人都知道有红眼这个事实的基础上的。
: 当岛上有3个红眼的时候,即时外人只说了一句今天开始执行,岛上的人也知道有红眼
: 这个事实,并且知道别人都知道有红眼这个事实。
: 你应该在弄明白什么是归纳法以后,再弄明白怎么用归纳法,不谢。
:
: 的.

D*a
发帖数: 6830
237


杀"

【在 l*3 的大作中提到】
: 噢, 那么你的结论是 "红颜足够多时, 就算外人公开宣称 "岛上有红眼", 也没人自杀"
: ?
:
: n=

l*3
发帖数: 2279
238
你先搞清楚归纳法里n=2是怎么做的.
岛上只有两个红眼(记为A,B)时, 这两个红眼 (以A为例) 会在外人宣称 "岛上有红眼"
后的第二天自杀, 是因为A有如下想法:
外人宣称 "岛上有红眼", 如果B没有看到红眼, 那么B就是唯一的红眼, B会在第一天晚
上自杀, 如果B不在地一天晚上自杀, 那只能是因为我(即A)也是红眼.
-----
现在外人只宣称了 "从今天开始我们来执行规则", 在这种情况下, 你给我一个 "岛上
只有两个红眼的话, 这两个红眼会在外人宣称后的第二天自杀" 的理由, 好吧? 我看不
出你怎么得出这个结论的.

【在 h*x 的大作中提到】
: 归纳法当然是针对所有人都知道有红眼这个事实的基础上的。
: 当岛上有3个红眼的时候,即时外人只说了一句今天开始执行,岛上的人也知道有红眼
: 这个事实,并且知道别人都知道有红眼这个事实。
: 你应该在弄明白什么是归纳法以后,再弄明白怎么用归纳法,不谢。
:
: 的.

l*3
发帖数: 2279
239
"足够多" 的下限是多少? 说个数目, 我再来反驳你.

【在 D*a 的大作中提到】
: 是
:
: 杀"

h*x
发帖数: 674
240
我知道你们为什么用嵌套,但是你们不知道只要用两层嵌套就够了,因为当A看到有k个
红眼的时候,他不知道红眼看到的是k-1个红眼,还是k个红眼。

【在 p**s 的大作中提到】
: 错,归纳法是建立在所有人都知道所有人都知道所有人都知道所有人都知道。。。。有
: 红眼的基础上。离开这个你理解不了的嵌套,归纳法就是错的。

相关主题
中国北方人跟南方人基本上是两个种族,无法协调别做小学题了,来个初中的
大家承认吧蓝眼睛就是比粽眼睛好看 (转载)问个奥数题
葛军,男,秒杀了52万江苏考生。。来做最后两题吧问个中文标点符号的嵌套问题
进入WaterWorld版参与讨论
h*x
发帖数: 674
241
所有人都知道所有人都知道所有人都知道是你自己在第一天做归纳法的时候就应该用到
的。 但是当你用这个归纳法的结论时,你用的是自己看到的k个红眼的情况。

【在 p**s 的大作中提到】
: 错,归纳法是建立在所有人都知道所有人都知道所有人都知道所有人都知道。。。。有
: 红眼的基础上。离开这个你理解不了的嵌套,归纳法就是错的。

l*3
发帖数: 2279
242
237楼我之前说的不清楚, 做了修改, 你再看看:
以下同237楼修改后的内容------
你先搞清楚归纳法里n=2是怎么做的.
岛上只有两个红眼(记为A,B)时, 这两个红眼 (以A为例) 会在外人宣称 "岛上有红眼"
后的第二天自杀, 是因为A有如下想法:
外人宣称 "岛上有红眼", 如果B没有看到红眼, 那么B就是唯一的红眼, B会在第一天晚
上自杀, 如果B不在地一天晚上自杀, 那只能是因为我(即A)也是红眼.
-----
现在外人只宣称了 "从今天开始我们来执行规则", 在这种情况下, 你给我一个 "岛上
只有两个红眼的话, 这两个红眼会在外人宣称后的第二天自杀" 的理由, 好吧? 我看不
出你怎么得出这个结论的.

【在 h*x 的大作中提到】
: 我知道你们为什么用嵌套,但是你们不知道只要用两层嵌套就够了,因为当A看到有k个
: 红眼的时候,他不知道红眼看到的是k-1个红眼,还是k个红眼。

p**s
发帖数: 2707
243
你说两个嵌套就够,说明你不知道我们为什么用嵌套。两层嵌套,只能得出n=2的结论
,要n=3成立,必须要3层嵌套。

【在 h*x 的大作中提到】
: 我知道你们为什么用嵌套,但是你们不知道只要用两层嵌套就够了,因为当A看到有k个
: 红眼的时候,他不知道红眼看到的是k-1个红眼,还是k个红眼。

p**s
发帖数: 2707
244
靠,太对了,所有人都知道所有人都知道所有人都知道就是你自己在第一天做归纳法的
时候就应该用到的。但是如果旅行者仅仅宣布开始计时,就没有所有人都知道所有人都
知道所有人都知道,所以你的归纳法缺条件。

【在 h*x 的大作中提到】
: 所有人都知道所有人都知道所有人都知道是你自己在第一天做归纳法的时候就应该用到
: 的。 但是当你用这个归纳法的结论时,你用的是自己看到的k个红眼的情况。

R****6
发帖数: 16
245

生?
Re

【在 l*3 的大作中提到】
: 转子知乎: http://www.zhihu.com/question/21262930
: 一个关于数学归纳法的悖论问题:到底是第N天有N个红眼睛自杀,还是什么都不会发生?
: 此问题最早据说是澳大利亚的华裔数学神童陶哲轩在网上贴出来让大家思考,逗大家玩
: 儿的。但却是真的把我难住了,一直百思不得其解。在此求教方家。
: 题目是这样的。说一个岛上有100个人,其中有5个红眼睛,95个蓝眼睛。这个岛有三个
: 奇怪的宗教规则。
: 1. 他们不能照镜子,不能看自己眼睛的颜色。
: 2. 他们不能告诉别人对方的眼睛是什么颜色。
: 3. 一旦有人知道了自己是红眼睛,他就必须在当天夜里自杀。
: 某天,有个旅行者到了这个岛上。由于不知道这里的规矩,所以他在和全岛人一起狂欢

h*x
发帖数: 674
246
我从来没有说过这种情况会成立。
我说的是n>2的情况。
而且我说你们不能在n=3的时候,让其中的某个红眼推测其他人会用n=2的情况。什么是
n=2?就是岛上有两个红眼的情况。这个情况下每个人会怎样?每个红眼都只看到一个
红眼。
现在岛上有三个红眼,就算其中的某个红眼假设自己是蓝眼,但他也不能由此就推断别
的红眼会用n=2的情况。因为别的红眼看到的是两个红眼!而n=2的时候每个红眼只看到
一个红眼!
所以别用嵌套!这个时候没有哪个红眼是哪个红眼的嵌套,所有的红眼都根据自己看到
2个红眼等第二天的情况。

"

【在 l*3 的大作中提到】
: 237楼我之前说的不清楚, 做了修改, 你再看看:
: 以下同237楼修改后的内容------
: 你先搞清楚归纳法里n=2是怎么做的.
: 岛上只有两个红眼(记为A,B)时, 这两个红眼 (以A为例) 会在外人宣称 "岛上有红眼"
: 后的第二天自杀, 是因为A有如下想法:
: 外人宣称 "岛上有红眼", 如果B没有看到红眼, 那么B就是唯一的红眼, B会在第一天晚
: 上自杀, 如果B不在地一天晚上自杀, 那只能是因为我(即A)也是红眼.
: -----
: 现在外人只宣称了 "从今天开始我们来执行规则", 在这种情况下, 你给我一个 "岛上
: 只有两个红眼的话, 这两个红眼会在外人宣称后的第二天自杀" 的理由, 好吧? 我看不

h*x
发帖数: 674
247
看我245楼吧,为什么不能在用归纳法的时候用嵌套。
归纳法是在推导的时候用嵌套。

【在 p**s 的大作中提到】
: 你说两个嵌套就够,说明你不知道我们为什么用嵌套。两层嵌套,只能得出n=2的结论
: ,要n=3成立,必须要3层嵌套。

p*****y
发帖数: 1982
248

这里错啦,假设自己是红眼,之后必然是假设别人用n=2。你不可以在假设自己是红眼
的同时假设另一个人能看到两个红眼,因为两个假设是互斥的。逻辑推理的前提之间不
可以互斥。

【在 h*x 的大作中提到】
: 我从来没有说过这种情况会成立。
: 我说的是n>2的情况。
: 而且我说你们不能在n=3的时候,让其中的某个红眼推测其他人会用n=2的情况。什么是
: n=2?就是岛上有两个红眼的情况。这个情况下每个人会怎样?每个红眼都只看到一个
: 红眼。
: 现在岛上有三个红眼,就算其中的某个红眼假设自己是蓝眼,但他也不能由此就推断别
: 的红眼会用n=2的情况。因为别的红眼看到的是两个红眼!而n=2的时候每个红眼只看到
: 一个红眼!
: 所以别用嵌套!这个时候没有哪个红眼是哪个红眼的嵌套,所有的红眼都根据自己看到
: 2个红眼等第二天的情况。

h*x
发帖数: 674
249
同学,推导,推导,推导。。。跟应用真的不是一个概念。
你在应用归纳法的时候,别推了,你推不起来,你不知道别人看到的是几个红眼,你没
有办法再去在这步嵌套什么,你只能用自己通过整个归纳法得出的结论去应用。并且相
信别人也会这么做。

【在 p**s 的大作中提到】
: 靠,太对了,所有人都知道所有人都知道所有人都知道就是你自己在第一天做归纳法的
: 时候就应该用到的。但是如果旅行者仅仅宣布开始计时,就没有所有人都知道所有人都
: 知道所有人都知道,所以你的归纳法缺条件。

D*a
发帖数: 6830
250
大于等于4
3我感觉两可,我再想想,
你就先说4吧。

【在 l*3 的大作中提到】
: "足够多" 的下限是多少? 说个数目, 我再来反驳你.
相关主题
大部分人不懂反证法在自己国家岛上杀死敌人侵略者应该不算犯罪,是英雄
每周1评:[2] 方舟子 的 16点动物园举办【我和红眼蝉】活动 (转载)
陕西出现红眼兽 (转载)[合集] 内贾德第一个在钓岛上声援中国 一席话太解气了!
进入WaterWorld版参与讨论
l*3
发帖数: 2279
251
服了你.
你既然从来不考虑n=2会怎样, 那你让我复习1楼中归纳法n=1和n=2的目的是什么?

【在 h*x 的大作中提到】
: 我从来没有说过这种情况会成立。
: 我说的是n>2的情况。
: 而且我说你们不能在n=3的时候,让其中的某个红眼推测其他人会用n=2的情况。什么是
: n=2?就是岛上有两个红眼的情况。这个情况下每个人会怎样?每个红眼都只看到一个
: 红眼。
: 现在岛上有三个红眼,就算其中的某个红眼假设自己是蓝眼,但他也不能由此就推断别
: 的红眼会用n=2的情况。因为别的红眼看到的是两个红眼!而n=2的时候每个红眼只看到
: 一个红眼!
: 所以别用嵌套!这个时候没有哪个红眼是哪个红眼的嵌套,所有的红眼都根据自己看到
: 2个红眼等第二天的情况。

l*3
发帖数: 2279
252
我就问你, 你的条件和题目条件一样不? 你先论证清楚为什么 "条件一样", 然后再用1
楼中归纳法的结论.
如果条件都不一样, 你还用个屁....

【在 h*x 的大作中提到】
: 给你一个理由? 我不是一直让你复习归纳法吗!
: 归纳法:
: 当n=1时,。。。
: 当n=2时,。。。
: (你真的要我赘述吗? 一楼就是,自己复习)
: 结论,如果我看到有k个红眼,就在第k个晚上看有没有人自杀,如果没有,自己就是红
: 眼。
: 用到A这里,他看到两个红眼,结果第二天晚上没有人自杀(因为其他两个红眼也看到
: 两个红眼,他们也在等第二天晚上有没有人自杀,但他们不会在第二天晚上自杀),所
: 以第三天这三个红眼都知道自己也是红眼了。

h*x
发帖数: 674
253
没错呀同学。 你不知道自己是不是红眼呀,你不知道别人看到的是一个红眼还是两个
红眼,你怎么能假设别人就是用n=2?
还有,在应用的时候,不是用n,而是用k!
因为你不知道n呀,你只能根据自己看到的k个红眼睛来等第k个晚上。

【在 p*****y 的大作中提到】
:
: 这里错啦,假设自己是红眼,之后必然是假设别人用n=2。你不可以在假设自己是红眼
: 的同时假设另一个人能看到两个红眼,因为两个假设是互斥的。逻辑推理的前提之间不
: 可以互斥。

h*x
发帖数: 674
254
n=1,和n=2的时候是他们自己在第一天做归纳法的时候用到的!
归纳法能少了n=1,n=2吗?
但是我们现在讨论的是n=3的情况,才说明旅行者不用说岛上有红眼睛也可以。
我才服了你!

【在 l*3 的大作中提到】
: 服了你.
: 你既然从来不考虑n=2会怎样, 那你让我复习1楼中归纳法n=1和n=2的目的是什么?

h*x
发帖数: 674
255
怎么不一样了? n=5的时候当然成立。
这不是你们让我从n=3开始详细说起的吗。
你才说个p!

用1

【在 l*3 的大作中提到】
: 我就问你, 你的条件和题目条件一样不? 你先论证清楚为什么 "条件一样", 然后再用1
: 楼中归纳法的结论.
: 如果条件都不一样, 你还用个屁....

R***o
发帖数: 3964
256
Wow

生?

【在 l*3 的大作中提到】
: 转子知乎: http://www.zhihu.com/question/21262930
: 一个关于数学归纳法的悖论问题:到底是第N天有N个红眼睛自杀,还是什么都不会发生?
: 此问题最早据说是澳大利亚的华裔数学神童陶哲轩在网上贴出来让大家思考,逗大家玩
: 儿的。但却是真的把我难住了,一直百思不得其解。在此求教方家。
: 题目是这样的。说一个岛上有100个人,其中有5个红眼睛,95个蓝眼睛。这个岛有三个
: 奇怪的宗教规则。
: 1. 他们不能照镜子,不能看自己眼睛的颜色。
: 2. 他们不能告诉别人对方的眼睛是什么颜色。
: 3. 一旦有人知道了自己是红眼睛,他就必须在当天夜里自杀。
: 某天,有个旅行者到了这个岛上。由于不知道这里的规矩,所以他在和全岛人一起狂欢

l*3
发帖数: 2279
257
(以下所有讨论基于题设) 如果有四个人A,B,C,D, 我们就以A为例:
因为A足够聪明, 我能想到的A都想到了, 所以我认为在外人将 "岛上有红眼" 这一事实
做了公开宣称后, A会有如下想法:
"如果我 (即A) 是蓝眼, 那么岛上只有三个红眼, 那么这三个红眼应该会在第三天晚上
自杀, 所以我先等三天看看"
结果过了第三天晚上, 三个红眼都没自杀 (因为他们有和A一样的想法), 于是悲剧的A
发现自己不是蓝眼, 就在第四天晚上自杀了.
------
以上用到了一个命题: "若岛上只有三个红眼, 在外人公开宣称<岛上有红眼> 之后, 这
三个红眼会在第三天晚上自杀"
这一命题是n=3的情形, 不过你现在又感觉是 "两可", 所以我推荐你先想清楚n=3的情
形.

【在 D*a 的大作中提到】
: 大于等于4
: 3我感觉两可,我再想想,
: 你就先说4吧。

D*a
发帖数: 6830
258
归纳法为什么不能少了n=1,n=2?直接从n=3的归纳法不存在吗?

【在 h*x 的大作中提到】
: n=1,和n=2的时候是他们自己在第一天做归纳法的时候用到的!
: 归纳法能少了n=1,n=2吗?
: 但是我们现在讨论的是n=3的情况,才说明旅行者不用说岛上有红眼睛也可以。
: 我才服了你!

p*****y
发帖数: 1982
259
反证法阿大哥,假设自己是蓝眼,然后在这个基础上推。后面所有的设定都不能和这个
设定矛盾。这个假设决定了另外两个人此时只能看到一个红眼。
也可以另开一个线程,假设自己是红眼,这样另外两个人可以看到两个红眼。虽然这才
是真相,但是这条线推下去没法得出可以证伪的东西,只有等另一个线程确定错误了之
后才知道这是真的。

【在 h*x 的大作中提到】
: 没错呀同学。 你不知道自己是不是红眼呀,你不知道别人看到的是一个红眼还是两个
: 红眼,你怎么能假设别人就是用n=2?
: 还有,在应用的时候,不是用n,而是用k!
: 因为你不知道n呀,你只能根据自己看到的k个红眼睛来等第k个晚上。

l*3
发帖数: 2279
260
n=5怎么就 "当然成立" (当然一样) 了?
你给论证一下怎么个 "当然成立" 法.
-----
我先给你论证一下为什么不一样:
外人做了 <岛上有红眼> 公开宣称后, 则 "每个人都知道每个人都知道...每个人都知
道岛上有红眼"
这个陈述中, 不管有多少个 (有限个) "每个人都知道", 那都是对的.
如果外人不做这个 <岛上有红眼> 的公开宣称, 至多只能有4个 "每个人都知道" 是对
的, 再多就有问题.
这就是区别!!!

【在 h*x 的大作中提到】
: 怎么不一样了? n=5的时候当然成立。
: 这不是你们让我从n=3开始详细说起的吗。
: 你才说个p!
:
: 用1

相关主题
靠~ 受不了,宇宙大得让我头晕[讨论]携带iPad进入中国内地要交税 (ZT)
人不能蒙着眼睛说瞎话[合集] [转] 红蓝眼睛的逻辑陷阱
神啊,我老公眼睛都直了,我这个恨啊!再贴几张一个外嫁后生的孩子的照片
进入WaterWorld版参与讨论
h*x
发帖数: 674
261
你再看看你的第二句。
现在我们讨论的是n=3的情况对吧。
如果我假设自己是红眼,那就等于是说另一个红眼能看到两个红眼,总共是3个红眼嘛
,这个一点不矛盾。
如果我假设自己是蓝眼,那就是在假设另一个红眼只看到一个红眼。
所以我不能假设另一个红眼用n=2的情况。

【在 p*****y 的大作中提到】
: 反证法阿大哥,假设自己是蓝眼,然后在这个基础上推。后面所有的设定都不能和这个
: 设定矛盾。这个假设决定了另外两个人此时只能看到一个红眼。
: 也可以另开一个线程,假设自己是红眼,这样另外两个人可以看到两个红眼。虽然这才
: 是真相,但是这条线推下去没法得出可以证伪的东西,只有等另一个线程确定错误了之
: 后才知道这是真的。

D*a
发帖数: 6830
262
你这又开始推理了,我根本就否认推理的必要,没有推理就没有假设他自己和别人是不
是红眼,也没有等几天看看的说法。

A

【在 l*3 的大作中提到】
: (以下所有讨论基于题设) 如果有四个人A,B,C,D, 我们就以A为例:
: 因为A足够聪明, 我能想到的A都想到了, 所以我认为在外人将 "岛上有红眼" 这一事实
: 做了公开宣称后, A会有如下想法:
: "如果我 (即A) 是蓝眼, 那么岛上只有三个红眼, 那么这三个红眼应该会在第三天晚上
: 自杀, 所以我先等三天看看"
: 结果过了第三天晚上, 三个红眼都没自杀 (因为他们有和A一样的想法), 于是悲剧的A
: 发现自己不是蓝眼, 就在第四天晚上自杀了.
: ------
: 以上用到了一个命题: "若岛上只有三个红眼, 在外人公开宣称<岛上有红眼> 之后, 这
: 三个红眼会在第三天晚上自杀"

l*3
发帖数: 2279
263
你这就是在说 "1楼中的方法是错的"
既然是错的, 你为什么在用人家n=2的结论?

【在 h*x 的大作中提到】
: 你再看看你的第二句。
: 现在我们讨论的是n=3的情况对吧。
: 如果我假设自己是红眼,那就等于是说另一个红眼能看到两个红眼,总共是3个红眼嘛
: ,这个一点不矛盾。
: 如果我假设自己是蓝眼,那就是在假设另一个红眼只看到一个红眼。
: 所以我不能假设另一个红眼用n=2的情况。

l*3
发帖数: 2279
264
原题的条件是 "A足够聪明"
你现在要否认 "推理的必要", 等于是在说 "A如果足够笨, 则他不会自杀"
你这是废话.

【在 D*a 的大作中提到】
: 你这又开始推理了,我根本就否认推理的必要,没有推理就没有假设他自己和别人是不
: 是红眼,也没有等几天看看的说法。
:
: A

h*x
发帖数: 674
265
可以用反证法呀。
假设自己是蓝眼睛,那另外两个红眼睛就只看到一个红眼睛。
假设自己是红眼睛,那另外两个红眼睛就看到两个红眼睛。
这条线没法推下去,太对了,我也是这个意思啊。所以我说你们不能假定其他红眼睛用
n=2的case。
所以这三个红眼睛没有谁嵌套谁呀,只有一起根据自己看到了2个红眼睛在等第2天晚上
发生什么呀。

【在 p*****y 的大作中提到】
: 反证法阿大哥,假设自己是蓝眼,然后在这个基础上推。后面所有的设定都不能和这个
: 设定矛盾。这个假设决定了另外两个人此时只能看到一个红眼。
: 也可以另开一个线程,假设自己是红眼,这样另外两个人可以看到两个红眼。虽然这才
: 是真相,但是这条线推下去没法得出可以证伪的东西,只有等另一个线程确定错误了之
: 后才知道这是真的。

l*3
发帖数: 2279
266
你干脆直接说 "A如果无视规则, 那么A就不用自杀" 好了.

【在 D*a 的大作中提到】
: 你这又开始推理了,我根本就否认推理的必要,没有推理就没有假设他自己和别人是不
: 是红眼,也没有等几天看看的说法。
:
: A

h*x
发帖数: 674
267
这就是我们前面说的我们推导的时候用的是n等于什么,
但是推导完了,在应用的时候要用k等于什么。
你怎么用n啊,你又不知道n。

【在 l*3 的大作中提到】
: 你这就是在说 "1楼中的方法是错的"
: 既然是错的, 你为什么在用人家n=2的结论?

p**s
发帖数: 2707
268
就说n=2,如果旅行者只宣布开始计时,会不会有人自杀?请回答会还是不会。

【在 h*x 的大作中提到】
: 我从来没有说过这种情况会成立。
: 我说的是n>2的情况。
: 而且我说你们不能在n=3的时候,让其中的某个红眼推测其他人会用n=2的情况。什么是
: n=2?就是岛上有两个红眼的情况。这个情况下每个人会怎样?每个红眼都只看到一个
: 红眼。
: 现在岛上有三个红眼,就算其中的某个红眼假设自己是蓝眼,但他也不能由此就推断别
: 的红眼会用n=2的情况。因为别的红眼看到的是两个红眼!而n=2的时候每个红眼只看到
: 一个红眼!
: 所以别用嵌套!这个时候没有哪个红眼是哪个红眼的嵌套,所有的红眼都根据自己看到
: 2个红眼等第二天的情况。

l*3
发帖数: 2279
269
我根本不知道尼前面说的 "n等于xx" 是什么奇怪的东西.
你既然想讨论清楚这个问题 (尤其是结论还没确定, 不是让你乱发感慨) 的时候, 就别
说话说半截. 你把红眼蓝眼等条件都加上, 说清楚.

【在 h*x 的大作中提到】
: 这就是我们前面说的我们推导的时候用的是n等于什么,
: 但是推导完了,在应用的时候要用k等于什么。
: 你怎么用n啊,你又不知道n。

p*****y
发帖数: 1982
270
第一条线可以推下去,第二条线不可以。第一条线后面就是n=2

【在 h*x 的大作中提到】
: 可以用反证法呀。
: 假设自己是蓝眼睛,那另外两个红眼睛就只看到一个红眼睛。
: 假设自己是红眼睛,那另外两个红眼睛就看到两个红眼睛。
: 这条线没法推下去,太对了,我也是这个意思啊。所以我说你们不能假定其他红眼睛用
: n=2的case。
: 所以这三个红眼睛没有谁嵌套谁呀,只有一起根据自己看到了2个红眼睛在等第2天晚上
: 发生什么呀。

相关主题
再贴几张一个外嫁后生的孩子的照片大家承认吧蓝眼睛就是比粽眼睛好看 (转载)
娶妻三次, 三个女人致其性无能葛军,男,秒杀了52万江苏考生。。来做最后两题吧
中国北方人跟南方人基本上是两个种族,无法协调别做小学题了,来个初中的
进入WaterWorld版参与讨论
h*x
发帖数: 674
271
。。。呃,别疯,我们可以不讨论。
像我们做理论的,都是把所有情况考虑遍了,得出通解后再带入具体参数。
就按你说的第二个假设吧,但是另一个人用的不是n,是k。用k=1,或2对不对?
但实际上,另一个人看到的是两个红眼睛,那他会用k=2.他也在等第二天。
我们这不统一起来了嘛!

【在 p*****y 的大作中提到】
: 第一条线可以推下去,第二条线不可以。第一条线后面就是n=2
h*x
发帖数: 674
272
不会。

【在 p**s 的大作中提到】
: 就说n=2,如果旅行者只宣布开始计时,会不会有人自杀?请回答会还是不会。
D*a
发帖数: 6830
273
根本没有矛盾,为什么要引入推理?难道不顾一切去玩推理才是聪明?

【在 l*3 的大作中提到】
: 原题的条件是 "A足够聪明"
: 你现在要否认 "推理的必要", 等于是在说 "A如果足够笨, 则他不会自杀"
: 你这是废话.

p*****y
发帖数: 1982
274
重新写了下。希望你这次可以理解我。

【在 h*x 的大作中提到】
: 。。。呃,别疯,我们可以不讨论。
: 像我们做理论的,都是把所有情况考虑遍了,得出通解后再带入具体参数。
: 就按你说的第二个假设吧,但是另一个人用的不是n,是k。用k=1,或2对不对?
: 但实际上,另一个人看到的是两个红眼睛,那他会用k=2.他也在等第二天。
: 我们这不统一起来了嘛!

h*x
发帖数: 674
275
恩,我前面态度也有点不好,谁让你先攻击我的。
这样吧,我们先说好了,就算大家在这个题目上观点不一致,也和而不用,好吧。

【在 l*3 的大作中提到】
: 我根本不知道尼前面说的 "n等于xx" 是什么奇怪的东西.
: 你既然想讨论清楚这个问题 (尤其是结论还没确定, 不是让你乱发感慨) 的时候, 就别
: 说话说半截. 你把红眼蓝眼等条件都加上, 说清楚.

l*3
发帖数: 2279
276
一步一步来:
现在总共有三个红眼A,B,C, 外地人只宣称了 "从今天开始, 我们来执行规则"
你说 "这三个红眼会在第三天自杀", 于是我认为你的意思至少包含了 "A会在第三天自
杀"
那我请问, 你的理由是不是如下这个?:
<如果前两天没人自杀, 则A就会确认自己是红眼>
请回答 "是" 或 "不是"

【在 h*x 的大作中提到】
: 恩,我前面态度也有点不好,谁让你先攻击我的。
: 这样吧,我们先说好了,就算大家在这个题目上观点不一致,也和而不用,好吧。

l*3
发帖数: 2279
277
真拙计.. 谁攻击谁了.. 我都没觉得你攻击我. 观点不同讨论的时候讽刺几句很正常,
可以理解. 我不觉得你攻击我了, 我只是说, 你要讨论清楚的话, 就每次把话说完整,
否则我也不知道你又引用了之前哪一楼的内容. 我在276楼重新开始调理思路, 你可以
回答一下275楼的问题先.

【在 h*x 的大作中提到】
: 恩,我前面态度也有点不好,谁让你先攻击我的。
: 这样吧,我们先说好了,就算大家在这个题目上观点不一致,也和而不用,好吧。

p**s
发帖数: 2707
278
好,现在假设BC都是红眼,A不确定自己是不是,A就想,
1,如果自己是红眼,那就是3个红眼,按hyx的理论,等第二天,第二天没人开枪,自
己就第三天自杀。
2,如果自己是蓝眼,那一共2个红眼,按hyx的理论,第二天也不会有人开枪。
两种假设,A观察到的,同样都是第二天没人开枪,请问A如何区分自己是处在哪一种假
设?

【在 h*x 的大作中提到】
: 不会。
p*****y
发帖数: 1982
279
你这样假设就没有意义了。假设的目的是减少环境中的变量,然后分情况讨论。就像我
们做实验总是固定一个参数,然后观察其他参数对结果的影响。
所以你在做了两个假设之后不应该去综合两个假设,得到"不能假设另一个红眼人
用n=2"。分情况之后,在"自己是蓝眼"这个假设上必然是另一个的人用n=2,它只能看
见一个红眼睛和一个蓝眼睛,你凭什么不让他用n=2阿。这里你不可以调用“他还有可
能看到两个红眼”的情况了,因为那样你就切换到另一个假设的框架里面了。

【在 h*x 的大作中提到】
: 你再看看你的第二句。
: 现在我们讨论的是n=3的情况对吧。
: 如果我假设自己是红眼,那就等于是说另一个红眼能看到两个红眼,总共是3个红眼嘛
: ,这个一点不矛盾。
: 如果我假设自己是蓝眼,那就是在假设另一个红眼只看到一个红眼。
: 所以我不能假设另一个红眼用n=2的情况。

l*3
发帖数: 2279
280
有矛盾呀.
在A看来: "A是蓝眼" 和 "第三天没人自杀" 矛盾.
(我指总共4个红眼的情形)

【在 D*a 的大作中提到】
: 根本没有矛盾,为什么要引入推理?难道不顾一切去玩推理才是聪明?
相关主题
问个奥数题每周1评:[2] 方舟子 的 16点
问个中文标点符号的嵌套问题陕西出现红眼兽 (转载)
大部分人不懂反证法在自己国家岛上杀死敌人侵略者应该不算犯罪,是英雄
进入WaterWorld版参与讨论
D*a
发帖数: 6830
281
我说的不就是
-----
陶哲轩说,这个旅行者事实上讲了一句废话,没有带来任何新的信息。因为这岛上有95
个蓝眼睛,5个红眼睛。每个人都知道这岛上有红眼睛的人。无非是蓝眼睛的人看到有5
个红眼睛,红眼睛的人看到有4个红眼睛而已。旅行者说的那句【岛上有红眼睛的人】
,没有输入任何新的信息,他说的就是岛上的人每天都看到的景象。所以哪怕岛上的人
思维再缜密严谨,也不会有任何自杀的情况发生。
-------
这一段吗,你不迷茫你认同你自己帖的第一个推理,上来问什么啊

【在 l*3 的大作中提到】
: 你干脆直接说 "A如果无视规则, 那么A就不用自杀" 好了.
h*x
发帖数: 674
282
好,我们就说两个假设其中的一个假设。假设我是其中一个红眼睛哈。
分情况后,在假设“自己是蓝眼睛”的基础上,我觉得n=2,但我不会认为另一个人用n
=2,我会认为他用k=1. 这两个不是同一个概念。他用k=1的时候,他会认为n=1,或n=2
. 所以他顶多第二天晚上就死了,这个符合我假设自己是蓝眼睛对吧。
但是我发现他第二天晚上没死,那说明他没有用k=1,他也是用的k=2。那么到第三天,
三个红眼睛都知道了n应该等于3.

【在 p*****y 的大作中提到】
: 你这样假设就没有意义了。假设的目的是减少环境中的变量,然后分情况讨论。就像我
: 们做实验总是固定一个参数,然后观察其他参数对结果的影响。
: 所以你在做了两个假设之后不应该去综合两个假设,得到"不能假设另一个红眼人
: 用n=2"。分情况之后,在"自己是蓝眼"这个假设上必然是另一个的人用n=2,它只能看
: 见一个红眼睛和一个蓝眼睛,你凭什么不让他用n=2阿。这里你不可以调用“他还有可
: 能看到两个红眼”的情况了,因为那样你就切换到另一个假设的框架里面了。

l*3
发帖数: 2279
283
这一段是不对的呀.

95
有5

【在 D*a 的大作中提到】
: 我说的不就是
: -----
: 陶哲轩说,这个旅行者事实上讲了一句废话,没有带来任何新的信息。因为这岛上有95
: 个蓝眼睛,5个红眼睛。每个人都知道这岛上有红眼睛的人。无非是蓝眼睛的人看到有5
: 个红眼睛,红眼睛的人看到有4个红眼睛而已。旅行者说的那句【岛上有红眼睛的人】
: ,没有输入任何新的信息,他说的就是岛上的人每天都看到的景象。所以哪怕岛上的人
: 思维再缜密严谨,也不会有任何自杀的情况发生。
: -------
: 这一段吗,你不迷茫你认同你自己帖的第一个推理,上来问什么啊

p*****y
发帖数: 1982
284
假设2的条件下 hyx不允许我们用n=2,他说有可能看见两个红眼。
hyx是这样吧,错了我自宫

【在 p**s 的大作中提到】
: 好,现在假设BC都是红眼,A不确定自己是不是,A就想,
: 1,如果自己是红眼,那就是3个红眼,按hyx的理论,等第二天,第二天没人开枪,自
: 己就第三天自杀。
: 2,如果自己是蓝眼,那一共2个红眼,按hyx的理论,第二天也不会有人开枪。
: 两种假设,A观察到的,同样都是第二天没人开枪,请问A如何区分自己是处在哪一种假
: 设?

p**s
发帖数: 2707
285
换句话说,你认为n=3的情形应该是,每个红眼的,看见两个红眼,而这两个红眼在第
二天没自杀,那他就应该在第三天自杀。这个没问题吧。
再回到n=2,你也说了不会有人自杀,也就是第二天没人自杀,那对于每一个蓝眼的,
他们也是看到两个红眼的,这两个红眼的第二天没自杀,那这完全符合你在n=3中描述
的情形,所以第三天,所有的蓝眼都会自杀。

【在 h*x 的大作中提到】
: 不会。
l*3
发帖数: 2279
286
又是奇奇怪怪的符号.
你把n和k的概念能不能先说清楚? 另外回答一下275楼的问题.

用n
=2

【在 h*x 的大作中提到】
: 好,我们就说两个假设其中的一个假设。假设我是其中一个红眼睛哈。
: 分情况后,在假设“自己是蓝眼睛”的基础上,我觉得n=2,但我不会认为另一个人用n
: =2,我会认为他用k=1. 这两个不是同一个概念。他用k=1的时候,他会认为n=1,或n=2
: . 所以他顶多第二天晚上就死了,这个符合我假设自己是蓝眼睛对吧。
: 但是我发现他第二天晚上没死,那说明他没有用k=1,他也是用的k=2。那么到第三天,
: 三个红眼睛都知道了n应该等于3.

p*****y
发帖数: 1982
287
啥是k=1阿

用n
=2

【在 h*x 的大作中提到】
: 好,我们就说两个假设其中的一个假设。假设我是其中一个红眼睛哈。
: 分情况后,在假设“自己是蓝眼睛”的基础上,我觉得n=2,但我不会认为另一个人用n
: =2,我会认为他用k=1. 这两个不是同一个概念。他用k=1的时候,他会认为n=1,或n=2
: . 所以他顶多第二天晚上就死了,这个符合我假设自己是蓝眼睛对吧。
: 但是我发现他第二天晚上没死,那说明他没有用k=1,他也是用的k=2。那么到第三天,
: 三个红眼睛都知道了n应该等于3.

h*x
发帖数: 674
288
呵呵,是的。
我自己可以假设n=2,但我不会推断别的红眼用n=2. 在这个假设下,我会假设别的红眼
用k=1. 但别的红眼看到的是两个红眼呀,他们用的是k=2. 所以第二天晚上我就知道自
己假设错误了。

【在 p*****y 的大作中提到】
: 假设2的条件下 hyx不允许我们用n=2,他说有可能看见两个红眼。
: hyx是这样吧,错了我自宫

l*3
发帖数: 2279
289
一步一步来:
现在总共有三个红眼A,B,C, 外地人只宣称了 "从今天开始, 我们来执行规则"
你说 "这三个红眼会在第三天自杀", 于是我认为你的意思至少包含了 "A会在第三天自
杀"
那我请问, 你的理由是不是如下这个?:
<如果前两天没人自杀, 则A就会确认自己是红眼>
请回答 "是" 或 "不是"

用n
=2

【在 h*x 的大作中提到】
: 好,我们就说两个假设其中的一个假设。假设我是其中一个红眼睛哈。
: 分情况后,在假设“自己是蓝眼睛”的基础上,我觉得n=2,但我不会认为另一个人用n
: =2,我会认为他用k=1. 这两个不是同一个概念。他用k=1的时候,他会认为n=1,或n=2
: . 所以他顶多第二天晚上就死了,这个符合我假设自己是蓝眼睛对吧。
: 但是我发现他第二天晚上没死,那说明他没有用k=1,他也是用的k=2。那么到第三天,
: 三个红眼睛都知道了n应该等于3.

h*x
发帖数: 674
290
k=1就是看到一个红眼啊。

【在 p*****y 的大作中提到】
: 啥是k=1阿
:
: 用n
: =2

相关主题
动物园举办【我和红眼蝉】活动 (转载)人不能蒙着眼睛说瞎话
[合集] 内贾德第一个在钓岛上声援中国 一席话太解气了!神啊,我老公眼睛都直了,我这个恨啊!
靠~ 受不了,宇宙大得让我头晕[讨论]携带iPad进入中国内地要交税 (ZT)
进入WaterWorld版参与讨论
h*x
发帖数: 674
291
呵呵,好,一步一步来,回答你的问题:是。

【在 l*3 的大作中提到】
: 一步一步来:
: 现在总共有三个红眼A,B,C, 外地人只宣称了 "从今天开始, 我们来执行规则"
: 你说 "这三个红眼会在第三天自杀", 于是我认为你的意思至少包含了 "A会在第三天自
: 杀"
: 那我请问, 你的理由是不是如下这个?:
: <如果前两天没人自杀, 则A就会确认自己是红眼>
: 请回答 "是" 或 "不是"
:
: 用n
: =2

D*a
发帖数: 6830
292
如果有四个红眼,那么每个红眼看见3个红眼,所以他认为这3个红眼毎人看见两个红眼
,这就是,
A知道每个人知道“岛上有红眼”,
B知道每个人知道“岛上有红眼”,
C知道每个人知道“岛上有红眼”
D知道每个人知道“岛上有红眼”
了,所以游客说的”岛上有红眼“跟他看见的是一样的情况。
直接推理能推出来的东西,为什么要用数学归纳法?不问三七二十一就从n=1开始推,
但是实际上n=1根本就是错的(因为红眼觉得n=3)。
我再帖一遍题,当n ≥ 3 时,n^2 > 2n,你第一步是看n=1,还是n=3?
为什么在岛上明明知道n=3或4,却要第一步看n=1?上面那道题为什么不看n=1?

【在 l*3 的大作中提到】
: 有矛盾呀.
: 在A看来: "A是蓝眼" 和 "第三天没人自杀" 矛盾.
: (我指总共4个红眼的情形)

h*x
发帖数: 674
293
所以我说不能用n=2呀,现在的情况是n=3.
红眼睛用k=2的条件,蓝眼睛用k=3的条件。
k的数值就是看到几个红眼睛。

【在 p**s 的大作中提到】
: 换句话说,你认为n=3的情形应该是,每个红眼的,看见两个红眼,而这两个红眼在第
: 二天没自杀,那他就应该在第三天自杀。这个没问题吧。
: 再回到n=2,你也说了不会有人自杀,也就是第二天没人自杀,那对于每一个蓝眼的,
: 他们也是看到两个红眼的,这两个红眼的第二天没自杀,那这完全符合你在n=3中描述
: 的情形,所以第三天,所有的蓝眼都会自杀。

l*3
发帖数: 2279
294
翻译成中文, 你的意思是不是:
如果我只看到了一个红眼, 则这个红眼必然会在外地人宣布 <今天开始执行规则> 的当
晚自杀?

【在 h*x 的大作中提到】
: k=1就是看到一个红眼啊。
l*3
发帖数: 2279
295
其实1楼的方法并不是严格意义上的 "数学归纳法", 但是确实是对的.
另外, 游客公开宣称的 "岛上有红眼" 要比 "每个人都知道每个人都知道有红眼" 带来
了更多的信息.
你还是看一下我一楼的出处链接吧 http://www.zhihu.com/question/21262930
看完第一个回答后你就知道区别在哪了.

【在 D*a 的大作中提到】
: 如果有四个红眼,那么每个红眼看见3个红眼,所以他认为这3个红眼毎人看见两个红眼
: ,这就是,
: A知道每个人知道“岛上有红眼”,
: B知道每个人知道“岛上有红眼”,
: C知道每个人知道“岛上有红眼”
: D知道每个人知道“岛上有红眼”
: 了,所以游客说的”岛上有红眼“跟他看见的是一样的情况。
: 直接推理能推出来的东西,为什么要用数学归纳法?不问三七二十一就从n=1开始推,
: 但是实际上n=1根本就是错的(因为红眼觉得n=3)。
: 我再帖一遍题,当n ≥ 3 时,n^2 > 2n,你第一步是看n=1,还是n=3?

h*x
发帖数: 674
296
不是不是。
你要是把15页所有的假设看下来,就是发现我们在讨论“我是一个红眼睛,我看到了两
个红眼睛,我在假设自己是蓝眼睛,然后我假设另一个红眼睛用k=1” 的情况。

【在 l*3 的大作中提到】
: 翻译成中文, 你的意思是不是:
: 如果我只看到了一个红眼, 则这个红眼必然会在外地人宣布 <今天开始执行规则> 的当
: 晚自杀?

l*3
发帖数: 2279
297
那么你认同了: <如果前两天没人自杀, 则A就会确认自己是红眼>
请问, 这是不是等同于说?:
<如果A是蓝眼, 则前两天必有人自杀>
回答 "是" 或 "不是"

【在 h*x 的大作中提到】
: 呵呵,好,一步一步来,回答你的问题:是。
p**s
发帖数: 2707
298
什么叫不能用n=2,你知道n=3,A又不知道。
假如你是A,看见了BC是红眼,然后旅行者跑来宣布计时开始,第二天又没人自杀,你
说A第三天是自杀还是不自杀?请回答自杀还是不自杀。

【在 h*x 的大作中提到】
: 所以我说不能用n=2呀,现在的情况是n=3.
: 红眼睛用k=2的条件,蓝眼睛用k=3的条件。
: k的数值就是看到几个红眼睛。

l*3
发帖数: 2279
299
我记不了15页所有的假设.
还是请你回答296楼的问题吧.

【在 h*x 的大作中提到】
: 不是不是。
: 你要是把15页所有的假设看下来,就是发现我们在讨论“我是一个红眼睛,我看到了两
: 个红眼睛,我在假设自己是蓝眼睛,然后我假设另一个红眼睛用k=1” 的情况。

p*****y
发帖数: 1982
300
搬个凳子看直播,今天太累了

【在 l*3 的大作中提到】
: 那么你认同了: <如果前两天没人自杀, 则A就会确认自己是红眼>
: 请问, 这是不是等同于说?:
: <如果A是蓝眼, 则前两天必有人自杀>
: 回答 "是" 或 "不是"

相关主题
[合集] [转] 红蓝眼睛的逻辑陷阱中国北方人跟南方人基本上是两个种族,无法协调
再贴几张一个外嫁后生的孩子的照片大家承认吧蓝眼睛就是比粽眼睛好看 (转载)
娶妻三次, 三个女人致其性无能葛军,男,秒杀了52万江苏考生。。来做最后两题吧
进入WaterWorld版参与讨论
h*x
发帖数: 674
301
对嘛,我们知道n等于几,岛上的人不知道啊,他们只知道自己看到了几个红眼睛,他
们只能用k等于几啊。
回答你的问题:自杀。

【在 p**s 的大作中提到】
: 什么叫不能用n=2,你知道n=3,A又不知道。
: 假如你是A,看见了BC是红眼,然后旅行者跑来宣布计时开始,第二天又没人自杀,你
: 说A第三天是自杀还是不自杀?请回答自杀还是不自杀。

l*3
发帖数: 2279
302
还是请你把你的话翻译完整.
你最后又用了一个 "k=1", 这使得整段话的意思不明确.

【在 h*x 的大作中提到】
: 不是不是。
: 你要是把15页所有的假设看下来,就是发现我们在讨论“我是一个红眼睛,我看到了两
: 个红眼睛,我在假设自己是蓝眼睛,然后我假设另一个红眼睛用k=1” 的情况。

l*3
发帖数: 2279
303
那么你认同了: <如果前两天没人自杀, 则A就会确认自己是红眼>
请问, 这是不是等同于说?:
<如果A是蓝眼, 则前两天必有人自杀>
回答 "是" 或 "不是"

【在 h*x 的大作中提到】
: 呵呵,好,一步一步来,回答你的问题:是。
l*3
发帖数: 2279
304
那么你认同了: <如果前两天没人自杀, 则A就会确认自己是红眼>
请问, 这是不是等同于说?:
A的想法是: <如果我(A)是蓝眼, 则前两天必有人自杀>
回答 "是" 或 "不是"
-----
以上问题基于对296楼的修改, 表达的更严密了一些.

【在 h*x 的大作中提到】
: 呵呵,好,一步一步来,回答你的问题:是。
h*x
发帖数: 674
305
现在有两个字母n和k,n是岛上有几个红眼睛,k是岛上的每个人看到了几个红眼睛。
我们在推导的时候,推导的是n=1,2,3。。。怎样。
但岛上的人在应用的时候,只能用k=1,2,3。。。怎样。
k有可能等于n,有可能不等于n,这个要到第k个晚上红眼睛的人才会知道,继而到k+1
天蓝眼睛的人知道。
k=1的意思就是对方看见了一个红眼睛的人。 刚才我跟那个同学不是在假设我自己是蓝
眼睛吗(我其实是红眼睛),那我当然就假设另外两个红眼睛分别只看到一个红眼睛了
,也就是他们在用k=1的条件。

【在 l*3 的大作中提到】
: 还是请你把你的话翻译完整.
: 你最后又用了一个 "k=1", 这使得整段话的意思不明确.

h*x
发帖数: 674
306
是。
但这个前提是,A是红眼睛,他在假设自己是蓝眼睛。
结果第二天晚上没人自杀,于是A知道了自己也是红眼睛。

【在 l*3 的大作中提到】
: 那么你认同了: <如果前两天没人自杀, 则A就会确认自己是红眼>
: 请问, 这是不是等同于说?:
: A的想法是: <如果我(A)是蓝眼, 则前两天必有人自杀>
: 回答 "是" 或 "不是"
: -----
: 以上问题基于对296楼的修改, 表达的更严密了一些.

p**s
发帖数: 2707
307
你答错了,A白死了,他其实是个蓝眼,岛上就只有BC两个红眼。这是你自己回答的两
个红眼会发生什么
发信人: hyx (阿迪), 信区: WaterWorld
标 题: Re: [转] 红蓝眼睛的逻辑陷阱
发信站: BBS 未名空间站 (Fri Jul 5 21:35:30 2013, 美东)
不会。

【在 p**s 的大作中提到】
: 就说n=2,如果旅行者只宣布开始计时,会不会有人自杀?请回答会还是不会。
l*3
发帖数: 2279
308
引用-----
他用k=1的时候,他会认为n=1,或n=2
. 所以他顶多第二天晚上就死了
----结束引用
以上就是你错误的地方, 试问: 假如只有1个红眼, 那一个蓝眼睛也会认为k=1, n=1或2
, 难道这个蓝眼睛会在第二天晚上之前死掉么?
事实是: 一直不会有人死. (如果外地人只是说 "今天开始执行规则" 的话)
你就没有论证清楚为什么 "他顶多第二天晚上就死了"

用n
=2

【在 h*x 的大作中提到】
: 好,我们就说两个假设其中的一个假设。假设我是其中一个红眼睛哈。
: 分情况后,在假设“自己是蓝眼睛”的基础上,我觉得n=2,但我不会认为另一个人用n
: =2,我会认为他用k=1. 这两个不是同一个概念。他用k=1的时候,他会认为n=1,或n=2
: . 所以他顶多第二天晚上就死了,这个符合我假设自己是蓝眼睛对吧。
: 但是我发现他第二天晚上没死,那说明他没有用k=1,他也是用的k=2。那么到第三天,
: 三个红眼睛都知道了n应该等于3.

h*x
发帖数: 674
309
n=2的时候,岛上的人如果只宣布计时开始,没有人会死啊。
因为红眼睛的人不知道另一个红眼睛是不是看见了99个蓝眼睛,也就是k=0.
而k=0就是n=1的情况,必然要由其他人告诉他们存在红眼睛。
我不是说不会吗,我没说A会死啊。

【在 p**s 的大作中提到】
: 你答错了,A白死了,他其实是个蓝眼,岛上就只有BC两个红眼。这是你自己回答的两
: 个红眼会发生什么
: 发信人: hyx (阿迪), 信区: WaterWorld
: 标 题: Re: [转] 红蓝眼睛的逻辑陷阱
: 发信站: BBS 未名空间站 (Fri Jul 5 21:35:30 2013, 美东)
: 不会。

l*3
发帖数: 2279
310
为什么要有这个前提?
你自认为有这个前提, 是不是等于是认同了以下这句话?:
<如果A是蓝眼, 红眼只有B,C, 则B,C会在两天内自杀>

【在 h*x 的大作中提到】
: 是。
: 但这个前提是,A是红眼睛,他在假设自己是蓝眼睛。
: 结果第二天晚上没人自杀,于是A知道了自己也是红眼睛。

相关主题
葛军,男,秒杀了52万江苏考生。。来做最后两题吧问个中文标点符号的嵌套问题
别做小学题了,来个初中的大部分人不懂反证法
问个奥数题每周1评:[2] 方舟子 的 16点
进入WaterWorld版参与讨论
p**s
发帖数: 2707
311
我问你A自杀还是不自杀,你说自杀,然后又说你没说A会死?请解释A到底是自杀了还
是没自杀,死了还是没死?
发信人: hyx (阿迪), 信区: WaterWorld
标 题: Re: [转] 红蓝眼睛的逻辑陷阱
发信站: BBS 未名空间站 (Fri Jul 5 22:02:41 2013, 美东)
对嘛,我们知道n等于几,岛上的人不知道啊,他们只知道自己看到了几个红眼睛,他
们只能用k等于几啊。
回答你的问题:自杀。

【在 h*x 的大作中提到】
: n=2的时候,岛上的人如果只宣布计时开始,没有人会死啊。
: 因为红眼睛的人不知道另一个红眼睛是不是看见了99个蓝眼睛,也就是k=0.
: 而k=0就是n=1的情况,必然要由其他人告诉他们存在红眼睛。
: 我不是说不会吗,我没说A会死啊。

l*3
发帖数: 2279
312
no offence, but
may I ask: what's your major?

【在 h*x 的大作中提到】
: 是。
: 但这个前提是,A是红眼睛,他在假设自己是蓝眼睛。
: 结果第二天晚上没人自杀,于是A知道了自己也是红眼睛。

p**s
发帖数: 2707
313
我推理的结果是,理论物理

【在 l*3 的大作中提到】
: no offence, but
: may I ask: what's your major?

p*****y
发帖数: 1982
314
大哥,不知道你这个前提哪里搞来的。他就是问个逆否命题,有什么前提阿

【在 h*x 的大作中提到】
: 是。
: 但这个前提是,A是红眼睛,他在假设自己是蓝眼睛。
: 结果第二天晚上没人自杀,于是A知道了自己也是红眼睛。

p**s
发帖数: 2707
315
你这个前提怎么实现,对于岛上的某一个A,他应该作为红眼假设自己是蓝眼,还是作
为蓝眼假设自己是蓝眼?

【在 h*x 的大作中提到】
: 是。
: 但这个前提是,A是红眼睛,他在假设自己是蓝眼睛。
: 结果第二天晚上没人自杀,于是A知道了自己也是红眼睛。

h*x
发帖数: 674
316
ok,前提是三个红眼睛,然后我是其中一个,我在假设我自己是蓝眼睛。
我的这个假设使得我认为另外两个红眼睛顶多在第二天就死了。因为我以为他们只看到
了一个红眼睛(这个红眼睛不是我)对吧,这是k=1,但是他们不知道n=1还是2,我们
再让他们也假设自己是蓝眼睛,这时候你们觉得推不出他们觉得对方是红眼睛的会死。
因为他们觉得对方不知道这个岛上有没有红眼睛,这是k=0。但是,他们其实看到我和
另一个都是红眼睛呀,所以他们知道对方知道这个岛上有红眼睛。所以我们三个红眼睛
每个人都知道另外红眼睛k的值至少为1,对吧,不存在k=0的情况。那么n至少等于2,
如果n=2,就应该是第二天晚上有人自杀。如果没有人自杀,那n就等于3,那我自己也
是红眼睛。

或2

【在 l*3 的大作中提到】
: 引用-----
: 他用k=1的时候,他会认为n=1,或n=2
: . 所以他顶多第二天晚上就死了
: ----结束引用
: 以上就是你错误的地方, 试问: 假如只有1个红眼, 那一个蓝眼睛也会认为k=1, n=1或2
: , 难道这个蓝眼睛会在第二天晚上之前死掉么?
: 事实是: 一直不会有人死. (如果外地人只是说 "今天开始执行规则" 的话)
: 你就没有论证清楚为什么 "他顶多第二天晚上就死了"
:
: 用n

p**s
发帖数: 2707
317
为啥必然要由其他人告诉他们存在红眼睛。其他人就偏不告诉岛上的人,他们缺条件判
断不出自己是什么眼睛,判断不出就先活着呗,不行吗?

【在 h*x 的大作中提到】
: n=2的时候,岛上的人如果只宣布计时开始,没有人会死啊。
: 因为红眼睛的人不知道另一个红眼睛是不是看见了99个蓝眼睛,也就是k=0.
: 而k=0就是n=1的情况,必然要由其他人告诉他们存在红眼睛。
: 我不是说不会吗,我没说A会死啊。

h*x
发帖数: 674
318
他的问题是,如果A觉得自己是蓝眼睛。
但我们讨论的是三个红眼睛,所以我强调一下前提,就是A其实是红眼睛。这不是我们
一直讨论的假设嘛。
你有没有看我给你回的281楼啊,我可是按照你的假设说的。

【在 p*****y 的大作中提到】
: 大哥,不知道你这个前提哪里搞来的。他就是问个逆否命题,有什么前提阿
l*3
发帖数: 2279
319
okok, 不管这个, 这不影响辩论结果.
反正你现在的意思是:
1. A是红眼.
2. A有这个想法: <如果我是蓝眼, 则B,C会在第二天晚上自杀>
对不对?

【在 p**s 的大作中提到】
: 你这个前提怎么实现,对于岛上的某一个A,他应该作为红眼假设自己是蓝眼,还是作
: 为蓝眼假设自己是蓝眼?

h*x
发帖数: 674
320
太对了。
所以你们一直在用假设,一直在用n等于什么。
我说要用k等于什么,用看到了几个红眼睛来算啊。

【在 p*****y 的大作中提到】
: 大哥,不知道你这个前提哪里搞来的。他就是问个逆否命题,有什么前提阿
相关主题
陕西出现红眼兽 (转载)[合集] 内贾德第一个在钓岛上声援中国 一席话太解气了!
在自己国家岛上杀死敌人侵略者应该不算犯罪,是英雄靠~ 受不了,宇宙大得让我头晕[讨论]
动物园举办【我和红眼蝉】活动 (转载)人不能蒙着眼睛说瞎话
进入WaterWorld版参与讨论
p*****y
发帖数: 1982
321
好吧,我算我没说,再自宫一次。

【在 h*x 的大作中提到】
: 太对了。
: 所以你们一直在用假设,一直在用n等于什么。
: 我说要用k等于什么,用看到了几个红眼睛来算啊。

p**s
发帖数: 2707
322
你回错帖子了,这个会给已经混乱的思维造成更大混淆的,LOL

【在 l*3 的大作中提到】
: okok, 不管这个, 这不影响辩论结果.
: 反正你现在的意思是:
: 1. A是红眼.
: 2. A有这个想法: <如果我是蓝眼, 则B,C会在第二天晚上自杀>
: 对不对?

h*x
发帖数: 674
323
行啊,n=2的时候,旅行者到来之前,他们可不那么或者吗。

【在 p**s 的大作中提到】
: 为啥必然要由其他人告诉他们存在红眼睛。其他人就偏不告诉岛上的人,他们缺条件判
: 断不出自己是什么眼睛,判断不出就先活着呗,不行吗?

l*3
发帖数: 2279
324
不是我们在用假设, 是A在用假设.
我们不假设, 我们是站在 "上帝视角" 观察岛上人的思维.
说这些就有点假大空了, 你还是回答319楼的问题吧.
附:319楼内容节选-----
你现在的意思是:
1. A是红眼.
2. A有这个想法: <如果我是蓝眼, 则B,C会在第二天晚上自杀>
对不对?

【在 h*x 的大作中提到】
: 太对了。
: 所以你们一直在用假设,一直在用n等于什么。
: 我说要用k等于什么,用看到了几个红眼睛来算啊。

h*x
发帖数: 674
325
我的思维一直很清晰好不好, 在我看来这个就像1+1=2那么清晰好不好。 lol

【在 p**s 的大作中提到】
: 你回错帖子了,这个会给已经混乱的思维造成更大混淆的,LOL
h*x
发帖数: 674
326
我前面回答你了,对。

【在 l*3 的大作中提到】
: 不是我们在用假设, 是A在用假设.
: 我们不假设, 我们是站在 "上帝视角" 观察岛上人的思维.
: 说这些就有点假大空了, 你还是回答319楼的问题吧.
: 附:319楼内容节选-----
: 你现在的意思是:
: 1. A是红眼.
: 2. A有这个想法: <如果我是蓝眼, 则B,C会在第二天晚上自杀>
: 对不对?

p**s
发帖数: 2707
327
清晰的话,解释一下蓝眼A怎么就被你自杀了?

【在 h*x 的大作中提到】
: 我的思维一直很清晰好不好, 在我看来这个就像1+1=2那么清晰好不好。 lol
D*a
发帖数: 6830
328
最后看了维基百科的common knowledge的页面,我觉得我搞清楚了。
我觉得知乎里面一楼那个,
如果有4个红眼,每个红眼认为我看到3个红眼,这可以划分成一共5种情况:
1、我是红的;
2、我是蓝的,且B自认为是红的;
3、我是蓝的,且B自认为是蓝的,且B认为C自认为是红的;
4、我是蓝的,且B自认为是蓝的,且B认为C自认为是蓝的,且B认为C认为D自认为是红
的;
5、我是蓝的,且B自认为是蓝的,且B认为C自认为是蓝的,且B认为C认为D自认为是蓝
的。
这个思路是对的。不能从n=1,n=2 这样推,应该直接展开就是n的情况。数学归纳法的
想法应该是有问题的。

【在 l*3 的大作中提到】
: 其实1楼的方法并不是严格意义上的 "数学归纳法", 但是确实是对的.
: 另外, 游客公开宣称的 "岛上有红眼" 要比 "每个人都知道每个人都知道有红眼" 带来
: 了更多的信息.
: 你还是看一下我一楼的出处链接吧 http://www.zhihu.com/question/21262930
: 看完第一个回答后你就知道区别在哪了.

l*3
发帖数: 2279
329
我没看到你前面回复318楼的帖子 (这个可以不去纠结)
ok, 那么我请问你, A为什么会有这个想法?:
<如果我是蓝眼, 则B,C会在第二天晚上自杀>

【在 h*x 的大作中提到】
: 我前面回答你了,对。
h*x
发帖数: 674
330
所以你同意n=3的时候,我们都用k来计算,然后都等第二天晚上发生了什么,对吧?

【在 p*****y 的大作中提到】
: 好吧,我算我没说,再自宫一次。
相关主题
神啊,我老公眼睛都直了,我这个恨啊!再贴几张一个外嫁后生的孩子的照片
携带iPad进入中国内地要交税 (ZT)娶妻三次, 三个女人致其性无能
[合集] [转] 红蓝眼睛的逻辑陷阱中国北方人跟南方人基本上是两个种族,无法协调
进入WaterWorld版参与讨论
l*3
发帖数: 2279
331
1楼的做法毫无问题, 和这个是等价的.
只是那个做法不是数学意义上的 "数学归纳法"
换一种描述, 可以说是 "对道上人的思维内容的递归推演"

【在 D*a 的大作中提到】
: 最后看了维基百科的common knowledge的页面,我觉得我搞清楚了。
: 我觉得知乎里面一楼那个,
: 如果有4个红眼,每个红眼认为我看到3个红眼,这可以划分成一共5种情况:
: 1、我是红的;
: 2、我是蓝的,且B自认为是红的;
: 3、我是蓝的,且B自认为是蓝的,且B认为C自认为是红的;
: 4、我是蓝的,且B自认为是蓝的,且B认为C自认为是蓝的,且B认为C认为D自认为是红
: 的;
: 5、我是蓝的,且B自认为是蓝的,且B认为C自认为是蓝的,且B认为C认为D自认为是蓝
: 的。

p**s
发帖数: 2707
332
好,再重新再来一次,完全是一道新题,跟前面的讨论没有任何关系,
假如你是A,在这个有三条规定的小岛上,你看到BC是红眼,其他97个蓝眼,然后有这
么一天,旅行者来宣布计时开始,第一天没人自杀,第二天也没人自杀,你作为A,第
三天自杀还是不自杀?

【在 h*x 的大作中提到】
: 行啊,n=2的时候,旅行者到来之前,他们可不那么或者吗。
h*x
发帖数: 674
333
前面我回答你好多次了呀,当n=2,那个旅行者不说岛上有红眼睛的人时,没有人会自
杀。
我一直说的是岛上有三个红眼睛,A是其中一个,他对自杀。
你看,300楼,我引用你的话说的是n=3,A会自杀,A也是红眼睛。
306楼,你引用了我引用你的话,我在回答你说n=2时,说的是A不会自杀。
我一直都没有说错啊。

【在 p**s 的大作中提到】
: 清晰的话,解释一下蓝眼A怎么就被你自杀了?
l*3
发帖数: 2279
334
其实你不见得搞清楚了.
比如这里的5 是说其中一个红眼(A)的内心想法: "我是蓝的,且B自认为是蓝的,且B认
为C自认为是蓝的,且B认为C认为D自认为是蓝的。"
我问你: 这个为什么在旅行者公开宣布 "岛上有红眼" 后, 可以被直接排除? A的这个
想法错在哪了?

【在 D*a 的大作中提到】
: 最后看了维基百科的common knowledge的页面,我觉得我搞清楚了。
: 我觉得知乎里面一楼那个,
: 如果有4个红眼,每个红眼认为我看到3个红眼,这可以划分成一共5种情况:
: 1、我是红的;
: 2、我是蓝的,且B自认为是红的;
: 3、我是蓝的,且B自认为是蓝的,且B认为C自认为是红的;
: 4、我是蓝的,且B自认为是蓝的,且B认为C自认为是蓝的,且B认为C认为D自认为是红
: 的;
: 5、我是蓝的,且B自认为是蓝的,且B认为C自认为是蓝的,且B认为C认为D自认为是蓝
: 的。

l*3
发帖数: 2279
335
good one

【在 p**s 的大作中提到】
: 好,再重新再来一次,完全是一道新题,跟前面的讨论没有任何关系,
: 假如你是A,在这个有三条规定的小岛上,你看到BC是红眼,其他97个蓝眼,然后有这
: 么一天,旅行者来宣布计时开始,第一天没人自杀,第二天也没人自杀,你作为A,第
: 三天自杀还是不自杀?

p**s
发帖数: 2707
336
你说n=3,A又不知道n=2还是n=3,A要知道n=3,第一天就自杀了,还用等到第三天啊。

【在 h*x 的大作中提到】
: 前面我回答你好多次了呀,当n=2,那个旅行者不说岛上有红眼睛的人时,没有人会自
: 杀。
: 我一直说的是岛上有三个红眼睛,A是其中一个,他对自杀。
: 你看,300楼,我引用你的话说的是n=3,A会自杀,A也是红眼睛。
: 306楼,你引用了我引用你的话,我在回答你说n=2时,说的是A不会自杀。
: 我一直都没有说错啊。

h*x
发帖数: 674
337
恩,那就重新来一次。
我会自杀。
如果你想说我自杀错了,其实我是蓝眼睛的话,那你的结论n=2已经违背了我说旅行者
只需要宣布的前提,我说旅行者是需要宣布的前提是n>2,我从一开始就是这么说的。

【在 p**s 的大作中提到】
: 好,再重新再来一次,完全是一道新题,跟前面的讨论没有任何关系,
: 假如你是A,在这个有三条规定的小岛上,你看到BC是红眼,其他97个蓝眼,然后有这
: 么一天,旅行者来宣布计时开始,第一天没人自杀,第二天也没人自杀,你作为A,第
: 三天自杀还是不自杀?

p*****y
发帖数: 1982
338
靠我又错了

【在 h*x 的大作中提到】
: 恩,那就重新来一次。
: 我会自杀。
: 如果你想说我自杀错了,其实我是蓝眼睛的话,那你的结论n=2已经违背了我说旅行者
: 只需要宣布的前提,我说旅行者是需要宣布的前提是n>2,我从一开始就是这么说的。

l*3
发帖数: 2279
339
我们按331楼的来吧. 反正你之前一直都是在预测A的想法.
那么就出这个新题:
现在你是A, 你看到岛上有两个红眼B,C, 旅行者在第一天宣布了 <从现在起, 我们开始
遵守规则>
第二天晚上, 还没有人自杀. 请问你是否自杀?
(注意, 你作为A, 你在一开始并不能知道上帝视角中你眼睛的颜色)

【在 h*x 的大作中提到】
: 前面我回答你好多次了呀,当n=2,那个旅行者不说岛上有红眼睛的人时,没有人会自
: 杀。
: 我一直说的是岛上有三个红眼睛,A是其中一个,他对自杀。
: 你看,300楼,我引用你的话说的是n=3,A会自杀,A也是红眼睛。
: 306楼,你引用了我引用你的话,我在回答你说n=2时,说的是A不会自杀。
: 我一直都没有说错啊。

p*****y
发帖数: 1982
340
>5

【在 h*x 的大作中提到】
: 恩,那就重新来一次。
: 我会自杀。
: 如果你想说我自杀错了,其实我是蓝眼睛的话,那你的结论n=2已经违背了我说旅行者
: 只需要宣布的前提,我说旅行者是需要宣布的前提是n>2,我从一开始就是这么说的。

相关主题
中国北方人跟南方人基本上是两个种族,无法协调别做小学题了,来个初中的
大家承认吧蓝眼睛就是比粽眼睛好看 (转载)问个奥数题
葛军,男,秒杀了52万江苏考生。。来做最后两题吧问个中文标点符号的嵌套问题
进入WaterWorld版参与讨论
l*3
发帖数: 2279
341
最后一次提醒你, 不要在后续辩论中使用 "n=2" "k=几" 这类东西, 你自己的符号只有
你自己能时刻记得意思.
总共只有不到3个红眼, 你直接用汉语表述, 也不复杂, 别人能看懂.

【在 h*x 的大作中提到】
: 恩,那就重新来一次。
: 我会自杀。
: 如果你想说我自杀错了,其实我是蓝眼睛的话,那你的结论n=2已经违背了我说旅行者
: 只需要宣布的前提,我说旅行者是需要宣布的前提是n>2,我从一开始就是这么说的。

h*x
发帖数: 674
342
会自杀。
我336楼完全回答了这个问题。并且我说,如果你们说我是蓝眼睛就算我错的话,那你
们的前提只有两个红眼睛已经违背了我说的三个或三个以上红眼睛。

【在 l*3 的大作中提到】
: 我们按331楼的来吧. 反正你之前一直都是在预测A的想法.
: 那么就出这个新题:
: 现在你是A, 你看到岛上有两个红眼B,C, 旅行者在第一天宣布了 <从现在起, 我们开始
: 遵守规则>
: 第二天晚上, 还没有人自杀. 请问你是否自杀?
: (注意, 你作为A, 你在一开始并不能知道上帝视角中你眼睛的颜色)

D*a
发帖数: 6830
343
这里面A认为每个人都自认为是蓝的了啊。A认为自己是蓝的,因此认为B看见了2个,因
此认为B认为C看见了一个...
但是这个认为套认为,和直接假设n=1,我觉得是不一样的。A作这个假设的时候,说了
这是五分之一的情况,没有排除掉B认为自己是红眼的情况。这个跟假设n=1,然后推n=
2,然后推n=3 在已知n根本不等于123的前提下是不一样的。
英文维基的题目没有提n=5这茬

【在 l*3 的大作中提到】
: 其实你不见得搞清楚了.
: 比如这里的5 是说其中一个红眼(A)的内心想法: "我是蓝的,且B自认为是蓝的,且B认
: 为C自认为是蓝的,且B认为C认为D自认为是蓝的。"
: 我问你: 这个为什么在旅行者公开宣布 "岛上有红眼" 后, 可以被直接排除? A的这个
: 想法错在哪了?

h*x
发帖数: 674
344
幸亏明天是周末,不然我怎么跟我老板交代。。。其实我已经没法跟我老板交代了。。
p**s
发帖数: 2707
345
我题目里的旅行者,凭什么要遵守你的前提,我已经说了,跟前面的讨论无关,这就是
一个不论n等于几,都只会宣布计时开始,一句多余废话没有的旅行者,你作为A,自杀
还是不自杀?

【在 h*x 的大作中提到】
: 恩,那就重新来一次。
: 我会自杀。
: 如果你想说我自杀错了,其实我是蓝眼睛的话,那你的结论n=2已经违背了我说旅行者
: 只需要宣布的前提,我说旅行者是需要宣布的前提是n>2,我从一开始就是这么说的。

l*3
发帖数: 2279
346
你这个描述就很不准确.
"每个人都认为自己是蓝眼睛" 在旅行者刚刚宣布 "有红眼" 的时候有什么错误?
如果有错, 则第一天晚上岂不是有人自杀? 因为按照你的描述, "A自认为是蓝眼 且 B
自认为是蓝眼 且 C自认为是蓝眼 且 D自认为是蓝眼" 是错的.

n=

【在 D*a 的大作中提到】
: 这里面A认为每个人都自认为是蓝的了啊。A认为自己是蓝的,因此认为B看见了2个,因
: 此认为B认为C看见了一个...
: 但是这个认为套认为,和直接假设n=1,我觉得是不一样的。A作这个假设的时候,说了
: 这是五分之一的情况,没有排除掉B认为自己是红眼的情况。这个跟假设n=1,然后推n=
: 2,然后推n=3 在已知n根本不等于123的前提下是不一样的。
: 英文维基的题目没有提n=5这茬

l*3
发帖数: 2279
347
你336楼中说到 "你的结论n=2" 这种我不能理解的话. 事实上我几乎只看到你不停地在
用 "n=几" "k=几" 这类符号, 别人没怎么说过. 所以我不知道你所指的 "某人的结论n
=2" 是什么东西.
你可以把336楼的内容用普通汉语描述一遍. 谢谢.

【在 h*x 的大作中提到】
: 会自杀。
: 我336楼完全回答了这个问题。并且我说,如果你们说我是蓝眼睛就算我错的话,那你
: 们的前提只有两个红眼睛已经违背了我说的三个或三个以上红眼睛。

h*x
发帖数: 674
348
自杀。

【在 p**s 的大作中提到】
: 我题目里的旅行者,凭什么要遵守你的前提,我已经说了,跟前面的讨论无关,这就是
: 一个不论n等于几,都只会宣布计时开始,一句多余废话没有的旅行者,你作为A,自杀
: 还是不自杀?

p**s
发帖数: 2707
349
另外,也不要用336楼,因为有人在不断自宫。。。

论n

【在 l*3 的大作中提到】
: 你336楼中说到 "你的结论n=2" 这种我不能理解的话. 事实上我几乎只看到你不停地在
: 用 "n=几" "k=几" 这类符号, 别人没怎么说过. 所以我不知道你所指的 "某人的结论n
: =2" 是什么东西.
: 你可以把336楼的内容用普通汉语描述一遍. 谢谢.

p*****y
发帖数: 1982
350
你杀错了,你是个蓝眼睛的。

【在 h*x 的大作中提到】
: 自杀。
相关主题
大部分人不懂反证法在自己国家岛上杀死敌人侵略者应该不算犯罪,是英雄
每周1评:[2] 方舟子 的 16点动物园举办【我和红眼蝉】活动 (转载)
陕西出现红眼兽 (转载)[合集] 内贾德第一个在钓岛上声援中国 一席话太解气了!
进入WaterWorld版参与讨论
h*x
发帖数: 674
351
自杀。
但如果你们说我自杀错了,其实我是个蓝眼睛,这个岛上只有2个红眼睛的话,你们这
两个红眼睛的前提已经违背了我之前说的,旅行者说这句话是在岛上至少有3个红眼睛
的前提。 如果岛上只有两个红眼睛的话,旅行者是要说一下岛上有红眼睛的。

论n

【在 l*3 的大作中提到】
: 你336楼中说到 "你的结论n=2" 这种我不能理解的话. 事实上我几乎只看到你不停地在
: 用 "n=几" "k=几" 这类符号, 别人没怎么说过. 所以我不知道你所指的 "某人的结论n
: =2" 是什么东西.
: 你可以把336楼的内容用普通汉语描述一遍. 谢谢.

D*a
发帖数: 6830
352
ABCD是等价的,只能排除掉5,但是还有4种情况排除不掉,第一天晚上A也不知道自己
是四种情况里面的哪一种,BCD也是一样。

B

【在 l*3 的大作中提到】
: 你这个描述就很不准确.
: "每个人都认为自己是蓝眼睛" 在旅行者刚刚宣布 "有红眼" 的时候有什么错误?
: 如果有错, 则第一天晚上岂不是有人自杀? 因为按照你的描述, "A自认为是蓝眼 且 B
: 自认为是蓝眼 且 C自认为是蓝眼 且 D自认为是蓝眼" 是错的.
:
: n=

p**s
发帖数: 2707
353
这么会儿功夫,你已经把蓝眼A杀了4次了,你真心不觉得有问题?

【在 h*x 的大作中提到】
: 自杀。
: 但如果你们说我自杀错了,其实我是个蓝眼睛,这个岛上只有2个红眼睛的话,你们这
: 两个红眼睛的前提已经违背了我之前说的,旅行者说这句话是在岛上至少有3个红眼睛
: 的前提。 如果岛上只有两个红眼睛的话,旅行者是要说一下岛上有红眼睛的。
:
: 论n

p*****y
发帖数: 1982
354
你们都是web阿,那真太不好意思了。下次注意。

【在 p**s 的大作中提到】
: 另外,也不要用336楼,因为有人在不断自宫。。。
:
: 论n

l*3
发帖数: 2279
355
我们没有任何前提说 "一定要三个或三个以上红眼", 题目只是如下:
你是A, 你看到岛上有两个红眼B,C, 有一天来了一个旅行者, 宣布 "从现在开始我们执
行规则" (并且这个旅行者的话是权威的, 不可反抗的), 过了第二天晚上, 还没有人自
杀. 请问你是否自杀?
你的回答是 "会"?
那你冤死了.

【在 h*x 的大作中提到】
: 会自杀。
: 我336楼完全回答了这个问题。并且我说,如果你们说我是蓝眼睛就算我错的话,那你
: 们的前提只有两个红眼睛已经违背了我说的三个或三个以上红眼睛。

h*x
发帖数: 674
356
你看,我就知道你们要说我错。
所以我不停地强调,这个旅行者不说其他废话的前提是岛上至少有三个红眼睛,我从一
开始就是这么说的。 他现在跟我说旅行者不说其他废话,我当然默认他满足我的前提
条件啊。

【在 p*****y 的大作中提到】
: 你杀错了,你是个蓝眼睛的。
p**s
发帖数: 2707
357
好吧,这个旅行者是个瞎子,他就会说一句话,计时开始,行了吧。

【在 h*x 的大作中提到】
: 自杀。
: 但如果你们说我自杀错了,其实我是个蓝眼睛,这个岛上只有2个红眼睛的话,你们这
: 两个红眼睛的前提已经违背了我之前说的,旅行者说这句话是在岛上至少有3个红眼睛
: 的前提。 如果岛上只有两个红眼睛的话,旅行者是要说一下岛上有红眼睛的。
:
: 论n

l*3
发帖数: 2279
358
什么叫 "ABCD是等价的,只能排除掉5" ?
我的问题是, 如何排除掉5?

【在 D*a 的大作中提到】
: ABCD是等价的,只能排除掉5,但是还有4种情况排除不掉,第一天晚上A也不知道自己
: 是四种情况里面的哪一种,BCD也是一样。
:
: B

p*****y
发帖数: 1982
359
只因旅行者少说了句话,就有人枉死, 这也太悲剧了

【在 h*x 的大作中提到】
: 自杀。
: 但如果你们说我自杀错了,其实我是个蓝眼睛,这个岛上只有2个红眼睛的话,你们这
: 两个红眼睛的前提已经违背了我之前说的,旅行者说这句话是在岛上至少有3个红眼睛
: 的前提。 如果岛上只有两个红眼睛的话,旅行者是要说一下岛上有红眼睛的。
:
: 论n

l*3
发帖数: 2279
360
你是不是不明白 "这个问题的表述与之前的问题无关" 这句话的意思?

【在 h*x 的大作中提到】
: 你看,我就知道你们要说我错。
: 所以我不停地强调,这个旅行者不说其他废话的前提是岛上至少有三个红眼睛,我从一
: 开始就是这么说的。 他现在跟我说旅行者不说其他废话,我当然默认他满足我的前提
: 条件啊。

相关主题
靠~ 受不了,宇宙大得让我头晕[讨论]携带iPad进入中国内地要交税 (ZT)
人不能蒙着眼睛说瞎话[合集] [转] 红蓝眼睛的逻辑陷阱
神啊,我老公眼睛都直了,我这个恨啊!再贴几张一个外嫁后生的孩子的照片
进入WaterWorld版参与讨论
h*x
发帖数: 674
361
你不觉得你这个前提有问提?
我一直都说的是旅行者不说废话的前提是岛上至少有三个红眼睛啊,你说旅游者不说废
话,那我当然默认满足我的前提。
你为什么要问我旅行者不说废话的情况,你当然是想证明我说的不对。但岛上只有两个
红眼睛根本不是我说的情况,旅行者再不说废话有什么意义呢?
如果岛上只有一个红眼睛,或只有两个红眼睛,旅行者是要说一下岛上存在红眼睛的。

【在 p**s 的大作中提到】
: 这么会儿功夫,你已经把蓝眼A杀了4次了,你真心不觉得有问题?
l*3
发帖数: 2279
362
你连 "不说其他废话" 的意思都不懂么?
"不说其他废话" 的意思, 就是 "只说了某句话, 没有说其他话", 这个和你的前提有什
么关系? 难道没有你的前提, "不说其他废话" 这个表述就是完全不可理解的了?

【在 h*x 的大作中提到】
: 你看,我就知道你们要说我错。
: 所以我不停地强调,这个旅行者不说其他废话的前提是岛上至少有三个红眼睛,我从一
: 开始就是这么说的。 他现在跟我说旅行者不说其他废话,我当然默认他满足我的前提
: 条件啊。

h*x
发帖数: 674
363
好吧。
这个问题的表述跟之前的无关, 我是A,我看到两个红眼睛,如果旅行者只说了那么一
句,而不说岛上有红眼睛的话,我不会自杀。
但是这个有什么意义呢,我们都同意啊。 我说的是岛上有三个以上红眼睛,旅行者可
以不说岛上有红眼睛的话。

【在 l*3 的大作中提到】
: 你是不是不明白 "这个问题的表述与之前的问题无关" 这句话的意思?
p**s
发帖数: 2707
364
唉,你又错了,这个岛上有ABC三个红眼,你之前说的是n>2,旅行者只要宣布计时开始
,第三天三个人就会自杀,怎么现在A又不自杀了?

【在 h*x 的大作中提到】
: 好吧。
: 这个问题的表述跟之前的无关, 我是A,我看到两个红眼睛,如果旅行者只说了那么一
: 句,而不说岛上有红眼睛的话,我不会自杀。
: 但是这个有什么意义呢,我们都同意啊。 我说的是岛上有三个以上红眼睛,旅行者可
: 以不说岛上有红眼睛的话。

p*****y
发帖数: 1982
365
嗯,现在可以讨论四个人的情况了。

【在 h*x 的大作中提到】
: 好吧。
: 这个问题的表述跟之前的无关, 我是A,我看到两个红眼睛,如果旅行者只说了那么一
: 句,而不说岛上有红眼睛的话,我不会自杀。
: 但是这个有什么意义呢,我们都同意啊。 我说的是岛上有三个以上红眼睛,旅行者可
: 以不说岛上有红眼睛的话。

p**s
发帖数: 2707
366
呵呵,不用,这次三个红眼之一没有自杀,又矛盾了。

【在 p*****y 的大作中提到】
: 嗯,现在可以讨论四个人的情况了。
h*x
发帖数: 674
367
呵呵,好。 我是A,我看到了两个红眼睛,旅行者说了计时开始,我不自杀,因为我不
知道那两个红眼睛知不知道对方知道这个岛上有没有红眼睛。

【在 p**s 的大作中提到】
: 好吧,这个旅行者是个瞎子,他就会说一句话,计时开始,行了吧。
D*a
发帖数: 6830
368
你的问题是两个啊,第一个是“每个人都认为自己是蓝眼睛" 在旅行者刚刚宣布 "有红
眼" 的时候有什么错误?
第二个是,如果有错, 则第一天晚上岂不是有人自杀?
我回答了第二个。
回答第一个问题。a认为自己是蓝眼睛,同时a认为b认为b是蓝眼睛,同时a认为b认为c
认为c是蓝眼睛,同时a认为b认为c认为d认为d是蓝眼睛,跟“岛上有红眼睛”矛盾啊。

【在 l*3 的大作中提到】
: 什么叫 "ABCD是等价的,只能排除掉5" ?
: 我的问题是, 如何排除掉5?

l*3
发帖数: 2279
369
好吧, 既然你这么乐于 "诡辩", 我们再换个题. 从现在开始讨论这个题:
你的名字叫做 X, 你可能是红眼或者蓝眼, 你和其他99个人在一个古怪的岛上, 是这个
岛上的岛民. 你看到其他99个人中, 有3个人是红眼, 他们的名字分别是A,B,C, 其余人
是蓝眼.
这个岛上有三条规矩:
1. 不准以 "照镜子" 等类似方式直接观察自己眼睛的颜色.
2. 所有100个岛民互相之间严禁以任何方式讨论和眼睛颜色有关的事情.
3. 如果某人知道自己是红眼, 则这个人必须在当晚自杀.
有一天, 来了一个神秘旅行者, 这个旅行者是公然的权威人物, 他说了一句话: "从现
在起, 所有岛民严格按规矩行事"
在旅行者说完这话的第三天晚上过后, 你发现没有人自杀.
请问你是否自杀?

【在 h*x 的大作中提到】
: 好吧。
: 这个问题的表述跟之前的无关, 我是A,我看到两个红眼睛,如果旅行者只说了那么一
: 句,而不说岛上有红眼睛的话,我不会自杀。
: 但是这个有什么意义呢,我们都同意啊。 我说的是岛上有三个以上红眼睛,旅行者可
: 以不说岛上有红眼睛的话。

p**s
发帖数: 2707
370
顺便确认一下,
如果岛上有3个红眼(n>2),那么,旅行者宣布计时开始,不用其他废话,这3个人在
第三天都会自杀,虽然不是原话,但是这是你前面讨论中的观点,对不对?

【在 h*x 的大作中提到】
: 呵呵,好。 我是A,我看到了两个红眼睛,旅行者说了计时开始,我不自杀,因为我不
: 知道那两个红眼睛知不知道对方知道这个岛上有没有红眼睛。

相关主题
再贴几张一个外嫁后生的孩子的照片大家承认吧蓝眼睛就是比粽眼睛好看 (转载)
娶妻三次, 三个女人致其性无能葛军,男,秒杀了52万江苏考生。。来做最后两题吧
中国北方人跟南方人基本上是两个种族,无法协调别做小学题了,来个初中的
进入WaterWorld版参与讨论
l*3
发帖数: 2279
371
请注意:
"a认为a是蓝眼睛,同时a认为b认为b是蓝眼睛,同时a认为b认为c认为c是蓝眼睛,同时
a认为b认为c认为d认为d是蓝眼睛" 和 "a认为a是蓝眼睛, b认为b是蓝眼睛, c认为c是
蓝眼睛, d认为d是蓝眼睛" 是两回事.
所以我之前说你表述不准确.
另外, "a认为a是蓝眼睛,同时a认为b认为b是蓝眼睛,同时a认为b认为c认为c是蓝眼睛
,同时a认为b认为c认为d认为d是蓝眼睛" 和 "岛上有红眼睛" 并不矛盾, 而是和 " <
岛上有红眼睛> 是common sense" 矛盾. (这个矛盾还要建立在原题的其他条件上, 比
如 " <每个人都足够聪明, 有common sense> 是common sense" )
------
我的问题是, 请具体指出 "a认为a是蓝眼睛,同时a认为b认为b是蓝眼睛,同时a认为b
认为c认为c是蓝眼睛,同时a认为b认为c认为d认为d是蓝眼睛 且<岛上有红眼睛> 是
common sense " 这句话为什么错了? 其表述中为什么自相矛盾?

c

【在 D*a 的大作中提到】
: 你的问题是两个啊,第一个是“每个人都认为自己是蓝眼睛" 在旅行者刚刚宣布 "有红
: 眼" 的时候有什么错误?
: 第二个是,如果有错, 则第一天晚上岂不是有人自杀?
: 我回答了第二个。
: 回答第一个问题。a认为自己是蓝眼睛,同时a认为b认为b是蓝眼睛,同时a认为b认为c
: 认为c是蓝眼睛,同时a认为b认为c认为d认为d是蓝眼睛,跟“岛上有红眼睛”矛盾啊。

l*3
发帖数: 2279
372
翻篇, 看这个题:
你的名字叫做 X, 你可能是红眼或者蓝眼, 你和其他99个人在一个古怪的岛上, 是这个
岛上的岛民. 你看到其他99个人中, 有3个人是红眼, 他们的名字分别是A,B,C, 其余人
是蓝眼.
这个岛上有三条规矩:
1. 不准以 "照镜子" 等类似方式直接观察自己眼睛的颜色.
2. 所有100个岛民互相之间严禁以任何方式讨论和眼睛颜色有关的事情.
3. 如果某人知道自己是红眼, 则这个人必须在当晚自杀.
有一天, 来了一个神秘旅行者, 这个旅行者是公然的权威人物, 他说了一句话: "从现
在起, 所有岛民严格按规矩行事"
在旅行者说完这话的第三天晚上过后, 你发现没有人自杀.
请问你是否自杀?

【在 h*x 的大作中提到】
: 呵呵,好。 我是A,我看到了两个红眼睛,旅行者说了计时开始,我不自杀,因为我不
: 知道那两个红眼睛知不知道对方知道这个岛上有没有红眼睛。

h*x
发帖数: 674
373
这个回答是我错了。我太纠结你的前提了。
A应该自杀。
如果岛上是三个红眼睛,A就自杀对了。
如果岛上是两个红眼睛,旅行者只说计时开始,不满足我说的前提,不存在。这时候旅
行者必须说岛上有红眼睛。

【在 p**s 的大作中提到】
: 唉,你又错了,这个岛上有ABC三个红眼,你之前说的是n>2,旅行者只要宣布计时开始
: ,第三天三个人就会自杀,怎么现在A又不自杀了?

p*****y
发帖数: 1982
374
旅行者说,我一瞎子你让我说啥?

【在 h*x 的大作中提到】
: 这个回答是我错了。我太纠结你的前提了。
: A应该自杀。
: 如果岛上是三个红眼睛,A就自杀对了。
: 如果岛上是两个红眼睛,旅行者只说计时开始,不满足我说的前提,不存在。这时候旅
: 行者必须说岛上有红眼睛。

p**s
发帖数: 2707
375
好,再重新再来一次,完全是一道新题,跟前面的讨论没有任何关系,
假如你是A,在这个有三条规定的小岛上,你看到BC是红眼,其他97个蓝眼,然后有这
么一天,来了个旅行者,是个瞎子,不管你有什么前提,他压根就不知道岛上有没有红
眼,有几个红眼,然后瞎子旅行者宣布计时开始,第一天没人自杀,第二天也没人自杀
,你作为A,第三天自杀还是不自杀?

【在 h*x 的大作中提到】
: 这个回答是我错了。我太纠结你的前提了。
: A应该自杀。
: 如果岛上是三个红眼睛,A就自杀对了。
: 如果岛上是两个红眼睛,旅行者只说计时开始,不满足我说的前提,不存在。这时候旅
: 行者必须说岛上有红眼睛。

l*3
发帖数: 2279
376
好奇问一句, 如果规则本身就是公然权威的东西, 一直在被遵守 (这也是原题的条件),
那旅行者到底带来了什么?

【在 h*x 的大作中提到】
: 这个回答是我错了。我太纠结你的前提了。
: A应该自杀。
: 如果岛上是三个红眼睛,A就自杀对了。
: 如果岛上是两个红眼睛,旅行者只说计时开始,不满足我说的前提,不存在。这时候旅
: 行者必须说岛上有红眼睛。

h*x
发帖数: 674
377
我纠正了上面的说法。 如果A看到两个红眼睛,第二天晚上还没人自杀,那他不管怎样
都自杀。
因为如果A是蓝眼睛的话,旅行者必须说岛上有红眼睛的人,那么第二天晚上那两个红
眼睛已经自杀了,论不到A在第三天纠结。
如果你非要说旅行者什么都不知道,就只说了计时开始,那不满足我前面说的条件,在
岛上只有两个红眼睛的时候就会发生错误的自杀事件啊。
旅行者只有在岛上有三个以上红眼睛的时候,可以只说计时开始,那么A的自杀就不是
错杀。

【在 p**s 的大作中提到】
: 顺便确认一下,
: 如果岛上有3个红眼(n>2),那么,旅行者宣布计时开始,不用其他废话,这3个人在
: 第三天都会自杀,虽然不是原话,但是这是你前面讨论中的观点,对不对?

h*x
发帖数: 674
378
那旅行者就是瞎子,他只说了计时开始,A错误地自杀了。这个情况不是我假设的,我
假设的是岛上知道有三个红眼睛的人,旅行者才可以说计时开始。

【在 p*****y 的大作中提到】
: 旅行者说,我一瞎子你让我说啥?
h*x
发帖数: 674
379
自杀。

【在 p**s 的大作中提到】
: 好,再重新再来一次,完全是一道新题,跟前面的讨论没有任何关系,
: 假如你是A,在这个有三条规定的小岛上,你看到BC是红眼,其他97个蓝眼,然后有这
: 么一天,来了个旅行者,是个瞎子,不管你有什么前提,他压根就不知道岛上有没有红
: 眼,有几个红眼,然后瞎子旅行者宣布计时开始,第一天没人自杀,第二天也没人自杀
: ,你作为A,第三天自杀还是不自杀?

h*x
发帖数: 674
380
对啊,你们早该考虑这个问题了,我说旅行者定义了第一天嘛。

),

【在 l*3 的大作中提到】
: 好奇问一句, 如果规则本身就是公然权威的东西, 一直在被遵守 (这也是原题的条件),
: 那旅行者到底带来了什么?

相关主题
问个奥数题每周1评:[2] 方舟子 的 16点
问个中文标点符号的嵌套问题陕西出现红眼兽 (转载)
大部分人不懂反证法在自己国家岛上杀死敌人侵略者应该不算犯罪,是英雄
进入WaterWorld版参与讨论
p**s
发帖数: 2707
381
这个我可以替他回答,带来的是T=0,计时开始,当然我认为这个是错误的,不是说T=0
错了,而是这个信息不足以改变岛上的自杀进程。就像我也可以说,给岛上多带来一个
人,这个也是事实,但是不是答案。

),

【在 l*3 的大作中提到】
: 好奇问一句, 如果规则本身就是公然权威的东西, 一直在被遵守 (这也是原题的条件),
: 那旅行者到底带来了什么?

p*****y
发帖数: 1982
382
你错了,你是个蓝眼睛的

【在 h*x 的大作中提到】
: 自杀。
p**s
发帖数: 2707
383
好了,蓝眼A已经被你自杀了5次。

【在 h*x 的大作中提到】
: 自杀。
D*a
发帖数: 6830
384
因为“"a认为a是蓝眼睛,同时a认为b认为b是蓝眼睛,同时a认为b
认为c认为c是蓝眼睛,同时a认为b认为c认为d认为d是蓝眼睛”成立的条件就是a认为b
看见2个红眼,同时a认为b认为c看见一个红眼,同时a认为b认为c认为d看见0个红眼。
这样就跟a知道b知道c知道d知道有红眼矛盾了。

【在 l*3 的大作中提到】
: 请注意:
: "a认为a是蓝眼睛,同时a认为b认为b是蓝眼睛,同时a认为b认为c认为c是蓝眼睛,同时
: a认为b认为c认为d认为d是蓝眼睛" 和 "a认为a是蓝眼睛, b认为b是蓝眼睛, c认为c是
: 蓝眼睛, d认为d是蓝眼睛" 是两回事.
: 所以我之前说你表述不准确.
: 另外, "a认为a是蓝眼睛,同时a认为b认为b是蓝眼睛,同时a认为b认为c认为c是蓝眼睛
: ,同时a认为b认为c认为d认为d是蓝眼睛" 和 "岛上有红眼睛" 并不矛盾, 而是和 " <
: 岛上有红眼睛> 是common sense" 矛盾. (这个矛盾还要建立在原题的其他条件上, 比
: 如 " <每个人都足够聪明, 有common sense> 是common sense" )
: ------

l*3
发帖数: 2279
385
靠, 难道旅行者来之前, 没有第一天?
按你的逻辑, 如果规则一直在被遵守, 早在旅行者来之前, 红眼都自杀光了.

【在 h*x 的大作中提到】
: 对啊,你们早该考虑这个问题了,我说旅行者定义了第一天嘛。
:
: ),

p**s
发帖数: 2707
386
:)

【在 p*****y 的大作中提到】
: 你错了,你是个蓝眼睛的
h*x
发帖数: 674
387
我替plus回了吧,
plus:你又错了,A是蓝眼睛。
hyx:关我什么事,这个错误又不是我造成的,是你让旅行者在岛上只有两个人的前提
下只说计时开始的。我说的是只有当岛上的红眼睛是大于2时,旅行者才只能说记时开
始。

【在 h*x 的大作中提到】
: 自杀。
h*x
发帖数: 674
388
这不是前两页讨论的内容嘛,说明plus假设错误呀,不能这么假设。 我说了,只有当
岛上的红眼睛大于2时,旅行者才能只说计时开始。
不然,如果只有两个红眼睛的时候,旅行者要说一下岛上有红眼睛的。

【在 p*****y 的大作中提到】
: 你错了,你是个蓝眼睛的
h*x
发帖数: 674
389
对啊,没有针对归纳法的第一天,第二天。
就是一个periodic boundary condition。每一天都跟前一天没区别,每一天都跟后一
天没区别,所以不会有人自杀。

【在 l*3 的大作中提到】
: 靠, 难道旅行者来之前, 没有第一天?
: 按你的逻辑, 如果规则一直在被遵守, 早在旅行者来之前, 红眼都自杀光了.

l*3
发帖数: 2279
390
那我们考虑这么一个问题:
你是X, 你和其他99个人 (三个红眼A,B,C, 其余人是蓝眼) 突然有一天 (称为第一天)
被囚禁到了一个小岛上, 你们这100个人都非常聪明 (且 "你们都很聪明" 是common
sense)
这个岛上有三个公示的规矩, 所有人都遵守 ( "遵守规矩" 是common sense). 规矩如
下:
1. 不准以 "照镜子" 等类似方式直接观察自己眼睛的颜色.
2. 所有100个岛民互相之间严禁以任何方式讨论和眼睛颜色有关的事情.
3. 如果某人知道自己是红眼, 则这个人必须在当晚自杀.
在你们登岛的第三天晚上过后, 你发现没有人自杀.
请问你是否自杀?

【在 h*x 的大作中提到】
: 对啊,你们早该考虑这个问题了,我说旅行者定义了第一天嘛。
:
: ),

相关主题
动物园举办【我和红眼蝉】活动 (转载)人不能蒙着眼睛说瞎话
[合集] 内贾德第一个在钓岛上声援中国 一席话太解气了!神啊,我老公眼睛都直了,我这个恨啊!
靠~ 受不了,宇宙大得让我头晕[讨论]携带iPad进入中国内地要交税 (ZT)
进入WaterWorld版参与讨论
l*3
发帖数: 2279
391
楼上的题, 和之前有旅行者, 是一样的吧? 从现在开始我们避免讨论旅行者, ok?

【在 h*x 的大作中提到】
: 对啊,你们早该考虑这个问题了,我说旅行者定义了第一天嘛。
:
: ),

p*****y
发帖数: 1982
392

)

【在 l*3 的大作中提到】
: 那我们考虑这么一个问题:
: 你是X, 你和其他99个人 (三个红眼A,B,C, 其余人是蓝眼) 突然有一天 (称为第一天)
: 被囚禁到了一个小岛上, 你们这100个人都非常聪明 (且 "你们都很聪明" 是common
: sense)
: 这个岛上有三个公示的规矩, 所有人都遵守 ( "遵守规矩" 是common sense). 规矩如
: 下:
: 1. 不准以 "照镜子" 等类似方式直接观察自己眼睛的颜色.
: 2. 所有100个岛民互相之间严禁以任何方式讨论和眼睛颜色有关的事情.
: 3. 如果某人知道自己是红眼, 则这个人必须在当晚自杀.
: 在你们登岛的第三天晚上过后, 你发现没有人自杀.

l*3
发帖数: 2279
393
你的意思是 "a认为b认为c认为d看见0个红眼" 和 "a知道b知道c知道d知道有红眼" 矛
盾?
我觉得这两句话并不矛盾. 这两句话为什么矛盾呢?
比如在a认为b认为c认为d认为d是红眼的情况下, 就不矛盾.

b

【在 D*a 的大作中提到】
: 因为“"a认为a是蓝眼睛,同时a认为b认为b是蓝眼睛,同时a认为b
: 认为c认为c是蓝眼睛,同时a认为b认为c认为d认为d是蓝眼睛”成立的条件就是a认为b
: 看见2个红眼,同时a认为b认为c看见一个红眼,同时a认为b认为c认为d看见0个红眼。
: 这样就跟a知道b知道c知道d知道有红眼矛盾了。

h*x
发帖数: 674
394
我是X,我看到了三个红眼ABC,我们去了某岛。
在进岛的第一天,所有人都看见了这三个规则。
第三个晚上还是没有人自杀,我会在第四个晚上自杀。

)

【在 l*3 的大作中提到】
: 那我们考虑这么一个问题:
: 你是X, 你和其他99个人 (三个红眼A,B,C, 其余人是蓝眼) 突然有一天 (称为第一天)
: 被囚禁到了一个小岛上, 你们这100个人都非常聪明 (且 "你们都很聪明" 是common
: sense)
: 这个岛上有三个公示的规矩, 所有人都遵守 ( "遵守规矩" 是common sense). 规矩如
: 下:
: 1. 不准以 "照镜子" 等类似方式直接观察自己眼睛的颜色.
: 2. 所有100个岛民互相之间严禁以任何方式讨论和眼睛颜色有关的事情.
: 3. 如果某人知道自己是红眼, 则这个人必须在当晚自杀.
: 在你们登岛的第三天晚上过后, 你发现没有人自杀.

l*3
发帖数: 2279
395
不, 让X说出凭什么他要让ABC在第三天晚上自杀? 凭什么ABC要在第三天晚上确信自己
是红眼人?

【在 p*****y 的大作中提到】
:
: )

h*x
发帖数: 674
396
这个帖子就我跟ptolemy最惨了,他不停地自宫,我不停地自杀。。。lol
l*3
发帖数: 2279
397
很好, 之前的翻篇, 我们就继续讨论这个问题.
请问你自杀的理由是什么?
--------
for the record, 附问题如下:
你是X, 你和其他99个人 (三个红眼A,B,C, 其余人是蓝眼) 突然有一天 (称为第一天)
被囚禁到了一个小岛上, 你们这100个人都非常聪明 (且 "你们都很聪明" 是common
sense)
这个岛上有三个公示的规矩, 所有人都遵守 ( "遵守规矩" 是common sense). 规矩如
下:
1. 不准以 "照镜子" 等类似方式直接观察自己眼睛的颜色.
2. 所有100个岛民互相之间严禁以任何方式讨论和眼睛颜色有关的事情.
3. 如果某人知道自己是红眼, 则这个人必须在当晚自杀.
在你们登岛的第三天晚上过后, 你发现没有人自杀.
请问你是否自杀?

【在 h*x 的大作中提到】
: 我是X,我看到了三个红眼ABC,我们去了某岛。
: 在进岛的第一天,所有人都看见了这三个规则。
: 第三个晚上还是没有人自杀,我会在第四个晚上自杀。
:
: )

h*x
发帖数: 674
398
因为我会归纳法呀,我知道如果第三天晚上没人自杀,说明ABC也看到了三个红眼人,
也就是我,那我就在第四天晚上乖乖自杀吧。

【在 l*3 的大作中提到】
: 不, 让X说出凭什么他要让ABC在第三天晚上自杀? 凭什么ABC要在第三天晚上确信自己
: 是红眼人?

l*3
发帖数: 2279
399
换个问法, 你自杀的理由是不是如下?:
"如果我(X)是蓝眼, 那么红眼A,B,C应该在第三天晚上自杀"
回答 "是" 或者 "不是", 谢谢.
--------
for the record, 附问题如下:
你是X, 你和其他99个人 (三个红眼A,B,C, 其余人是蓝眼) 突然有一天 (称为第一天)
被囚禁到了一个小岛上, 你们这100个人都非常聪明 (且 "你们都很聪明" 是common
sense)
这个岛上有三个公示的规矩, 所有人都遵守 ( "遵守规矩" 是common sense). 规矩如
下:
1. 不准以 "照镜子" 等类似方式直接观察自己眼睛的颜色.
2. 所有100个岛民互相之间严禁以任何方式讨论和眼睛颜色有关的事情.
3. 如果某人知道自己是红眼, 则这个人必须在当晚自杀.
在你们登岛的第三天晚上过后, 你发现没有人自杀.
请问你是否自杀?

【在 h*x 的大作中提到】
: 我是X,我看到了三个红眼ABC,我们去了某岛。
: 在进岛的第一天,所有人都看见了这三个规则。
: 第三个晚上还是没有人自杀,我会在第四个晚上自杀。
:
: )

h*x
发帖数: 674
400
是。

)

【在 l*3 的大作中提到】
: 换个问法, 你自杀的理由是不是如下?:
: "如果我(X)是蓝眼, 那么红眼A,B,C应该在第三天晚上自杀"
: 回答 "是" 或者 "不是", 谢谢.
: --------
: for the record, 附问题如下:
: 你是X, 你和其他99个人 (三个红眼A,B,C, 其余人是蓝眼) 突然有一天 (称为第一天)
: 被囚禁到了一个小岛上, 你们这100个人都非常聪明 (且 "你们都很聪明" 是common
: sense)
: 这个岛上有三个公示的规矩, 所有人都遵守 ( "遵守规矩" 是common sense). 规矩如
: 下:

相关主题
[合集] [转] 红蓝眼睛的逻辑陷阱中国北方人跟南方人基本上是两个种族,无法协调
再贴几张一个外嫁后生的孩子的照片大家承认吧蓝眼睛就是比粽眼睛好看 (转载)
娶妻三次, 三个女人致其性无能葛军,男,秒杀了52万江苏考生。。来做最后两题吧
进入WaterWorld版参与讨论
D*a
发帖数: 6830
401
我说的不严谨了,a认为b认为c认为d看见0个红眼,可以是a认为b认为c认为d认为自己
是蓝眼,也可以是a认为b认为c认为d认为自己是红眼,然后加上前面我没有粘贴复制的
a认为..., 如果是a认为b认为c认为d认为自己是蓝眼,就矛盾了,如果是a认为b认为c
认为d认为自己是红眼,就不矛盾。...蓝眼,就是那个第5种情况,...红眼,就是那个
第4种情况.
你也太循循善诱了...就不能直接说哪儿错了吗.

【在 l*3 的大作中提到】
: 你的意思是 "a认为b认为c认为d看见0个红眼" 和 "a知道b知道c知道d知道有红眼" 矛
: 盾?
: 我觉得这两句话并不矛盾. 这两句话为什么矛盾呢?
: 比如在a认为b认为c认为d认为d是红眼的情况下, 就不矛盾.
:
: b

l*3
发帖数: 2279
402
好, 那你的理由就是: "如果我(X)是蓝眼, 那么A,B,C应该在第三天晚上自杀"
那我请问你:
如果你是蓝眼, A,B,C在第三天晚上自杀的动机是什么? 以A举例说明, 谢谢.
--------
for the record, 附问题如下:
你是X, 你和其他99个人 (三个红眼A,B,C, 其余人是蓝眼) 突然有一天 (称为第一天)
被囚禁到了一个小岛上, 你们这100个人都非常聪明 (且 "你们都很聪明" 是common
sense)
这个岛上有三个公示的规矩, 所有人都遵守 ( "遵守规矩" 是common sense). 规矩如
下:
1. 不准以 "照镜子" 等类似方式直接观察自己眼睛的颜色.
2. 所有100个岛民互相之间严禁以任何方式讨论和眼睛颜色有关的事情.
3. 如果某人知道自己是红眼, 则这个人必须在当晚自杀.
在你们登岛的第三天晚上过后, 你发现没有人自杀.
请问你是否自杀?

【在 h*x 的大作中提到】
: 因为我会归纳法呀,我知道如果第三天晚上没人自杀,说明ABC也看到了三个红眼人,
: 也就是我,那我就在第四天晚上乖乖自杀吧。

h*x
发帖数: 674
403
plus呢? 做了个错误的假设就溜了。。。让我白死了。。。

【在 h*x 的大作中提到】
: 我替plus回了吧,
: plus:你又错了,A是蓝眼睛。
: hyx:关我什么事,这个错误又不是我造成的,是你让旅行者在岛上只有两个人的前提
: 下只说计时开始的。我说的是只有当岛上的红眼睛是大于2时,旅行者才只能说记时开
: 始。

h*x
发帖数: 674
404
A如果只看到两个红眼睛,而第二天晚上又没人自杀,那A就知道其他红眼睛也看到了2
个红眼睛,其中一个就是自己,就知道了自己也是红眼睛,就应该在第三天晚上自杀。

)

【在 l*3 的大作中提到】
: 好, 那你的理由就是: "如果我(X)是蓝眼, 那么A,B,C应该在第三天晚上自杀"
: 那我请问你:
: 如果你是蓝眼, A,B,C在第三天晚上自杀的动机是什么? 以A举例说明, 谢谢.
: --------
: for the record, 附问题如下:
: 你是X, 你和其他99个人 (三个红眼A,B,C, 其余人是蓝眼) 突然有一天 (称为第一天)
: 被囚禁到了一个小岛上, 你们这100个人都非常聪明 (且 "你们都很聪明" 是common
: sense)
: 这个岛上有三个公示的规矩, 所有人都遵守 ( "遵守规矩" 是common sense). 规矩如
: 下:

l*3
发帖数: 2279
405
"如果是a认为b认为c认为d认为自己是蓝眼,就矛盾了"
这里还是没说清楚, 没有指出 "到底矛盾在哪了"
其实单这一句话 (即 "a认为b认为c认为d认为自己是蓝眼") 并没有问题.
我直说吧, 我是这个意思:
"a认为a是蓝眼, 且a认为b认为b是蓝眼, 且a认为b认为c认为c是蓝眼, 且a认为b认为c
认为d认为d是蓝眼" 这句话错在:
a认为a是蓝眼 且a认为b认为b是蓝眼 => a认为b认为 "a是蓝眼, 且b是蓝眼"
a认为b认为 "a是蓝眼, 且b是蓝眼" 且 a认为b认为c认为c是蓝眼 => a认为b认为c认为
"a,b,c都是蓝眼"
a认为b认为c认为 "a,b,c都是蓝眼" 且 a认为b认为c认为d认为d是蓝眼 => a认为b认
为c认为d认为 "a,b,c,d都是蓝眼"
a认为b认为c认为d认为 "a,b,c,d都是蓝眼" 且 a认为b认为c认为d认为 "其他96个人都
是蓝眼" => a认为b认为c认为d认为 "没有红眼"
矛盾于a认为b认为c认为d认为 "有红眼"
-----
也就是说, 错在 "a眼中的b眼中的c眼中的d的逻辑有问题", 这个是不允许的, 因为 "
所有人都很聪明, 逻辑没问题" 是一个common sense.

c

【在 D*a 的大作中提到】
: 我说的不严谨了,a认为b认为c认为d看见0个红眼,可以是a认为b认为c认为d认为自己
: 是蓝眼,也可以是a认为b认为c认为d认为自己是红眼,然后加上前面我没有粘贴复制的
: a认为..., 如果是a认为b认为c认为d认为自己是蓝眼,就矛盾了,如果是a认为b认为c
: 认为d认为自己是红眼,就不矛盾。...蓝眼,就是那个第5种情况,...红眼,就是那个
: 第4种情况.
: 你也太循循善诱了...就不能直接说哪儿错了吗.

l*3
发帖数: 2279
406
你的意思是, A只看到两个红眼睛后, 必然有如下想法:
<如果真的只有两个红眼睛, 那他俩肯定会在第二天晚上自杀>
是不是?

2

【在 h*x 的大作中提到】
: A如果只看到两个红眼睛,而第二天晚上又没人自杀,那A就知道其他红眼睛也看到了2
: 个红眼睛,其中一个就是自己,就知道了自己也是红眼睛,就应该在第三天晚上自杀。
:
: )

h*x
发帖数: 674
407
我再重申一次我的观点,对于一楼的原题,在旅行者来之前,大家都知道这个岛上有红
眼人,红眼人知道其他的红眼人至少看到3个红眼人,蓝眼人知道红眼人至少看到4个红
眼人。
旅行者说了“这个岛上有红眼人”,这句话没有改变大家对红眼人的任何认知。红眼人
还是知道其他的红眼人至少看到3个红眼人,蓝眼人还是知道红眼人至少看到4个红眼人
。改变的是,既然这个岛上有红眼人,你们就应该执行这个岛规。于是大家从旅行者说
那句话开始算第一天,使得题目的“当天夜里”这个时间定义有了具体的意义。所以旅
行者带来的新信息是定义了第一天。
l*3
发帖数: 2279
408
或者, 你把这句话里的 "如果...那么.." 的逻辑关系详细的解释一下:
个红眼睛>
A是怎么 "知道其他红眼睛也看到了2个红眼睛" 的?

2

【在 h*x 的大作中提到】
: A如果只看到两个红眼睛,而第二天晚上又没人自杀,那A就知道其他红眼睛也看到了2
: 个红眼睛,其中一个就是自己,就知道了自己也是红眼睛,就应该在第三天晚上自杀。
:
: )

l*3
发帖数: 2279
409
现在不说旅行者, 就说前几楼的问题.

【在 h*x 的大作中提到】
: 我再重申一次我的观点,对于一楼的原题,在旅行者来之前,大家都知道这个岛上有红
: 眼人,红眼人知道其他的红眼人至少看到3个红眼人,蓝眼人知道红眼人至少看到4个红
: 眼人。
: 旅行者说了“这个岛上有红眼人”,这句话没有改变大家对红眼人的任何认知。红眼人
: 还是知道其他的红眼人至少看到3个红眼人,蓝眼人还是知道红眼人至少看到4个红眼人
: 。改变的是,既然这个岛上有红眼人,你们就应该执行这个岛规。于是大家从旅行者说
: 那句话开始算第一天,使得题目的“当天夜里”这个时间定义有了具体的意义。所以旅
: 行者带来的新信息是定义了第一天。

h*x
发帖数: 674
410
是的。

【在 l*3 的大作中提到】
: 你的意思是, A只看到两个红眼睛后, 必然有如下想法:
: <如果真的只有两个红眼睛, 那他俩肯定会在第二天晚上自杀>
: 是不是?
:
: 2

相关主题
葛军,男,秒杀了52万江苏考生。。来做最后两题吧问个中文标点符号的嵌套问题
别做小学题了,来个初中的大部分人不懂反证法
问个奥数题每周1评:[2] 方舟子 的 16点
进入WaterWorld版参与讨论
h*x
发帖数: 674
411
我刚等的无聊,码点字贝。

【在 l*3 的大作中提到】
: 现在不说旅行者, 就说前几楼的问题.
l*3
发帖数: 2279
412
为什么 "如果只有两个红眼睛, 那么这两个红眼睛会在第二天晚上自杀" ?
A的这一断言是怎么得出来的?
或者准确的说, X眼中的A的这一断言是根据什么得出来的?

【在 h*x 的大作中提到】
: 是的。
h*x
发帖数: 674
413
如果第二天晚上没有人死的话,那其他两个红眼睛就看到了两个或两个以上的红眼睛。

了2

【在 l*3 的大作中提到】
: 或者, 你把这句话里的 "如果...那么.." 的逻辑关系详细的解释一下:
: : 个红眼睛>
: A是怎么 "知道其他红眼睛也看到了2个红眼睛" 的?
:
: 2

h*x
发帖数: 674
414
A的断言跟X的断言是两码事。
X只能通过A有没有在第三个晚上自杀来判断A看到了2个红眼睛还是3个红眼睛。
X觉得,如果A只看到了2个红眼睛,他会在第三天自杀。在X看到,不存在第二天有人自
杀的情况。因为X已经知道ABC三个都是红眼睛。
对于X而言,没有“如果只有两个红眼睛”的情况,只有“如果A以为只有两个红眼睛”
的情况,然后X想,如果A以为只有两个红眼睛,那就是我是蓝眼睛,那A就会在发现第
二天没人自杀的情况下知道自己也是红眼睛,那A就会在第三天自杀。但如果A以为有三
个红眼睛,那A无论如何不会在第三个晚上自杀,那就是说明我也是红眼睛。

【在 l*3 的大作中提到】
: 为什么 "如果只有两个红眼睛, 那么这两个红眼睛会在第二天晚上自杀" ?
: A的这一断言是怎么得出来的?
: 或者准确的说, X眼中的A的这一断言是根据什么得出来的?

l*3
发帖数: 2279
415
为什么 "如果只有两个红眼睛, 那么这两个红眼睛会在第二天晚上自杀" ?
A的这一断言是怎么得出来的?
或者准确的说, X眼中的A的这一断言是根据什么得出来的?

【在 h*x 的大作中提到】
: 是的。
D*a
发帖数: 6830
416
这个很清楚了,同意

c

【在 l*3 的大作中提到】
: "如果是a认为b认为c认为d认为自己是蓝眼,就矛盾了"
: 这里还是没说清楚, 没有指出 "到底矛盾在哪了"
: 其实单这一句话 (即 "a认为b认为c认为d认为自己是蓝眼") 并没有问题.
: 我直说吧, 我是这个意思:
: "a认为a是蓝眼, 且a认为b认为b是蓝眼, 且a认为b认为c认为c是蓝眼, 且a认为b认为c
: 认为d认为d是蓝眼" 这句话错在:
: a认为a是蓝眼 且a认为b认为b是蓝眼 => a认为b认为 "a是蓝眼, 且b是蓝眼"
: a认为b认为 "a是蓝眼, 且b是蓝眼" 且 a认为b认为c认为c是蓝眼 => a认为b认为c认为
: "a,b,c都是蓝眼"
: a认为b认为c认为 "a,b,c都是蓝眼" 且 a认为b认为c认为d认为d是蓝眼 => a认为b认

h*x
发帖数: 674
417
而A的断言取决于他看到X的眼睛是什么颜色。 这个信息在A那里是确定的,但在X这里
是不确定的。
所以X的断言和A的断言不一样。

【在 h*x 的大作中提到】
: A的断言跟X的断言是两码事。
: X只能通过A有没有在第三个晚上自杀来判断A看到了2个红眼睛还是3个红眼睛。
: X觉得,如果A只看到了2个红眼睛,他会在第三天自杀。在X看到,不存在第二天有人自
: 杀的情况。因为X已经知道ABC三个都是红眼睛。
: 对于X而言,没有“如果只有两个红眼睛”的情况,只有“如果A以为只有两个红眼睛”
: 的情况,然后X想,如果A以为只有两个红眼睛,那就是我是蓝眼睛,那A就会在发现第
: 二天没人自杀的情况下知道自己也是红眼睛,那A就会在第三天自杀。但如果A以为有三
: 个红眼睛,那A无论如何不会在第三个晚上自杀,那就是说明我也是红眼睛。

l*3
发帖数: 2279
418
你搞清楚你的逻辑推导顺序, 你说:
"X只能通过A有没有在第三个晚上自杀来判断A看到了2个红眼睛还是3个红眼睛。"
就等于X断言了A "如果只看到两个红眼睛, 并且这两个红眼睛没有在第二天晚上自杀,
那么A就会在第三天晚上自杀"
X(也就是你) 断言A会这么做, 是因为 X断言了A断言了 "如果只有两个红眼睛, 那么
这两个红眼睛会在第二天晚上自杀"
对于X而言, 确实不需要讨论 "如果只有两个红眼睛" 的情况. 但是X眼中的A是需要考
察这种情况的.
也就是说

我现在问的是, 你(也就是X)凭什么认为A会如此断言? A如此断言的依据是什么?

【在 h*x 的大作中提到】
: A的断言跟X的断言是两码事。
: X只能通过A有没有在第三个晚上自杀来判断A看到了2个红眼睛还是3个红眼睛。
: X觉得,如果A只看到了2个红眼睛,他会在第三天自杀。在X看到,不存在第二天有人自
: 杀的情况。因为X已经知道ABC三个都是红眼睛。
: 对于X而言,没有“如果只有两个红眼睛”的情况,只有“如果A以为只有两个红眼睛”
: 的情况,然后X想,如果A以为只有两个红眼睛,那就是我是蓝眼睛,那A就会在发现第
: 二天没人自杀的情况下知道自己也是红眼睛,那A就会在第三天自杀。但如果A以为有三
: 个红眼睛,那A无论如何不会在第三个晚上自杀,那就是说明我也是红眼睛。

h*x
发帖数: 674
419
空隙时间再说下这个时间的概念吧,时间是有方向的大家都同意吧,这个方向是由“事
件”定义的,“事件”之后是通向未来的光錐,“事件”之前是通向以前的光錐。原帖
中,旅行者的那句话就是一个定义时间的“事件”。在他说完之后,岛上的人知道现在
是开始执行规则的时候了。不然他们不知道什么时候开始执行呀。
p*****y
发帖数: 1982
420
又绕进去了,唉,hyx不允许断言嵌套。

,

【在 l*3 的大作中提到】
: 你搞清楚你的逻辑推导顺序, 你说:
: "X只能通过A有没有在第三个晚上自杀来判断A看到了2个红眼睛还是3个红眼睛。"
: 就等于X断言了A "如果只看到两个红眼睛, 并且这两个红眼睛没有在第二天晚上自杀,
: 那么A就会在第三天晚上自杀"
: X(也就是你) 断言A会这么做, 是因为 X断言了A断言了 "如果只有两个红眼睛, 那么
: 这两个红眼睛会在第二天晚上自杀"
: 对于X而言, 确实不需要讨论 "如果只有两个红眼睛" 的情况. 但是X眼中的A是需要考
: 察这种情况的.
: 也就是说
:

相关主题
陕西出现红眼兽 (转载)[合集] 内贾德第一个在钓岛上声援中国 一席话太解气了!
在自己国家岛上杀死敌人侵略者应该不算犯罪,是英雄靠~ 受不了,宇宙大得让我头晕[讨论]
动物园举办【我和红眼蝉】活动 (转载)人不能蒙着眼睛说瞎话
进入WaterWorld版参与讨论
h*x
发帖数: 674
421
就是归纳法呀。 我相信A是会归纳法的。
所以我相信他知道如果只有两个红眼睛,这两个红眼睛的人会一起在第二天晚上自杀的。

,

【在 l*3 的大作中提到】
: 你搞清楚你的逻辑推导顺序, 你说:
: "X只能通过A有没有在第三个晚上自杀来判断A看到了2个红眼睛还是3个红眼睛。"
: 就等于X断言了A "如果只看到两个红眼睛, 并且这两个红眼睛没有在第二天晚上自杀,
: 那么A就会在第三天晚上自杀"
: X(也就是你) 断言A会这么做, 是因为 X断言了A断言了 "如果只有两个红眼睛, 那么
: 这两个红眼睛会在第二天晚上自杀"
: 对于X而言, 确实不需要讨论 "如果只有两个红眼睛" 的情况. 但是X眼中的A是需要考
: 察这种情况的.
: 也就是说
:

h*x
发帖数: 674
422
你也太小看我了,嵌套就嵌套贝。 我前面不是还用你嵌套思路在281楼回了你的假设,
你就再没回我了。

【在 p*****y 的大作中提到】
: 又绕进去了,唉,hyx不允许断言嵌套。
:
: ,

l*3
发帖数: 2279
423
"如果只有两个红眼睛, 那么这两个红眼睛会在第二天晚上自杀"
请你把这个归纳证明一下.

的。

【在 h*x 的大作中提到】
: 就是归纳法呀。 我相信A是会归纳法的。
: 所以我相信他知道如果只有两个红眼睛,这两个红眼睛的人会一起在第二天晚上自杀的。
:
: ,

p*****y
发帖数: 1982
424
因为我打不过你,就只好旁观了。
我猜你用了循环论证,但是我不确定。我逻辑课没学好,一学期就去了两次,大作业也
没交,都不知道怎么过的

【在 h*x 的大作中提到】
: 你也太小看我了,嵌套就嵌套贝。 我前面不是还用你嵌套思路在281楼回了你的假设,
: 你就再没回我了。

l*3
发帖数: 2279
425
马上就完了, 我已经循循善诱到最后几步了.

【在 p*****y 的大作中提到】
: 因为我打不过你,就只好旁观了。
: 我猜你用了循环论证,但是我不确定。我逻辑课没学好,一学期就去了两次,大作业也
: 没交,都不知道怎么过的

p*****y
发帖数: 1982
426
没看出来哦

【在 l*3 的大作中提到】
: 马上就完了, 我已经循循善诱到最后几步了.
h*x
发帖数: 674
427
如果只有一个红眼睛,当这个红眼睛知道岛上有红眼睛的时候,他看到的都是蓝眼睛,
他立刻就知道自己就是红眼睛,就在第一天晚上自杀了。
如果有两个红眼睛,这两个红眼睛都会看到一个红眼睛,但他们不知道自己是不是红眼
睛,但是在第一天夜里没人自杀以后,他们就知道对方也看到了红眼睛,也就是自己,
然后就一起在第二天夜里自杀了。

【在 l*3 的大作中提到】
: 马上就完了, 我已经循循善诱到最后几步了.
l*3
发帖数: 2279
428
引用-----
如果只有一个红眼睛,当这个红眼睛知道岛上有红眼睛的时候,他看到的都是蓝眼睛,
他立刻就知道自己就是红眼睛,就在第一天晚上自杀了。
---结束引用
请问这一个红眼睛是如何知道 "岛上有红眼睛的" ? 谁告诉他的?

【在 h*x 的大作中提到】
: 如果只有一个红眼睛,当这个红眼睛知道岛上有红眼睛的时候,他看到的都是蓝眼睛,
: 他立刻就知道自己就是红眼睛,就在第一天晚上自杀了。
: 如果有两个红眼睛,这两个红眼睛都会看到一个红眼睛,但他们不知道自己是不是红眼
: 睛,但是在第一天夜里没人自杀以后,他们就知道对方也看到了红眼睛,也就是自己,
: 然后就一起在第二天夜里自杀了。

h*x
发帖数: 674
429
打不过你还不承认我对? 那么清晰明了的思路,不知道你纠结啥。
什么循环论证呀,根本就没循环,用的是归纳法的两层嵌套。
你们老在yy别人怎么想,我不yy,我直接观测别人怎么做。

【在 p*****y 的大作中提到】
: 因为我打不过你,就只好旁观了。
: 我猜你用了循环论证,但是我不确定。我逻辑课没学好,一学期就去了两次,大作业也
: 没交,都不知道怎么过的

p*****y
发帖数: 1982
430
打不过你得意思不是我错了,是我知道自己对但是不知道怎么说服你。

【在 h*x 的大作中提到】
: 打不过你还不承认我对? 那么清晰明了的思路,不知道你纠结啥。
: 什么循环论证呀,根本就没循环,用的是归纳法的两层嵌套。
: 你们老在yy别人怎么想,我不yy,我直接观测别人怎么做。

相关主题
神啊,我老公眼睛都直了,我这个恨啊!再贴几张一个外嫁后生的孩子的照片
携带iPad进入中国内地要交税 (ZT)娶妻三次, 三个女人致其性无能
[合集] [转] 红蓝眼睛的逻辑陷阱中国北方人跟南方人基本上是两个种族,无法协调
进入WaterWorld版参与讨论
h*x
发帖数: 674
431
就知道你要到这块。
如果只有一个红眼睛,是他们做的归纳法, 是每个人在脑子里完成的。
但是现在! 没有这个如果,X看到了3个红眼睛,X知道A至少看到了2个红眼睛。 所以
不要用如果几个红眼睛了同学, 请用看到了几个红眼睛。
你觉得这个岛上会有人有疑问,岛上存在着另一个人,并且那个人不知道有红眼睛吗?
如果没有人有疑问,就没有你这个问题。

【在 l*3 的大作中提到】
: 引用-----
: 如果只有一个红眼睛,当这个红眼睛知道岛上有红眼睛的时候,他看到的都是蓝眼睛,
: 他立刻就知道自己就是红眼睛,就在第一天晚上自杀了。
: ---结束引用
: 请问这一个红眼睛是如何知道 "岛上有红眼睛的" ? 谁告诉他的?

l*3
发帖数: 2279
432
引用-----
如果只有一个红眼睛,当这个红眼睛知道岛上有红眼睛的时候,他看到的都是蓝眼睛,
他立刻就知道自己就是红眼睛,就在第一天晚上自杀了。
---结束引用
如果岛上只有一个红眼睛, 那请问这一个红眼睛是如何知道 "岛上有红眼睛的" ? 谁告
诉他的?

【在 h*x 的大作中提到】
: 如果只有一个红眼睛,当这个红眼睛知道岛上有红眼睛的时候,他看到的都是蓝眼睛,
: 他立刻就知道自己就是红眼睛,就在第一天晚上自杀了。
: 如果有两个红眼睛,这两个红眼睛都会看到一个红眼睛,但他们不知道自己是不是红眼
: 睛,但是在第一天夜里没人自杀以后,他们就知道对方也看到了红眼睛,也就是自己,
: 然后就一起在第二天夜里自杀了。

h*x
发帖数: 674
433
你觉得你对,你回答不出我问你的问题。
我觉得我对,我回答的出你的问题。
如果有第三个人,他会觉得我们谁对呀?

【在 p*****y 的大作中提到】
: 打不过你得意思不是我错了,是我知道自己对但是不知道怎么说服你。
l*3
发帖数: 2279
434
你的前提是什么? 是不是 "岛上只有一个红眼睛" ?

【在 h*x 的大作中提到】
: 就知道你要到这块。
: 如果只有一个红眼睛,是他们做的归纳法, 是每个人在脑子里完成的。
: 但是现在! 没有这个如果,X看到了3个红眼睛,X知道A至少看到了2个红眼睛。 所以
: 不要用如果几个红眼睛了同学, 请用看到了几个红眼睛。
: 你觉得这个岛上会有人有疑问,岛上存在着另一个人,并且那个人不知道有红眼睛吗?
: 如果没有人有疑问,就没有你这个问题。

l*3
发帖数: 2279
435
你连你的前提都不承认, 你下的结论谁看?

【在 h*x 的大作中提到】
: 就知道你要到这块。
: 如果只有一个红眼睛,是他们做的归纳法, 是每个人在脑子里完成的。
: 但是现在! 没有这个如果,X看到了3个红眼睛,X知道A至少看到了2个红眼睛。 所以
: 不要用如果几个红眼睛了同学, 请用看到了几个红眼睛。
: 你觉得这个岛上会有人有疑问,岛上存在着另一个人,并且那个人不知道有红眼睛吗?
: 如果没有人有疑问,就没有你这个问题。

l*3
发帖数: 2279
436
1. 岛上的人都看到有红眼睛, 这个没问题.
2. X的眼中的A的眼中的B的眼中的C的眼中是可能没有红眼睛的.
你搞不懂 2 表述的意思, 老以为 1 就能说明 2 不对, 这就是你的逻辑问题.

【在 h*x 的大作中提到】
: 就知道你要到这块。
: 如果只有一个红眼睛,是他们做的归纳法, 是每个人在脑子里完成的。
: 但是现在! 没有这个如果,X看到了3个红眼睛,X知道A至少看到了2个红眼睛。 所以
: 不要用如果几个红眼睛了同学, 请用看到了几个红眼睛。
: 你觉得这个岛上会有人有疑问,岛上存在着另一个人,并且那个人不知道有红眼睛吗?
: 如果没有人有疑问,就没有你这个问题。

p*****y
发帖数: 1982
437
我回答不出你的问题是因为你的问题不合逻辑。
你回答了我的问题,但是你得回答的逻辑是错的。
第三个人要说觉得你对,说明他没有学过逻辑学,哈哈

【在 h*x 的大作中提到】
: 你觉得你对,你回答不出我问你的问题。
: 我觉得我对,我回答的出你的问题。
: 如果有第三个人,他会觉得我们谁对呀?

h*x
发帖数: 674
438
“岛上只有一个红眼睛”是我做归纳法的第一步,不是前提,前提跟第一步不是一个概
念,前提是不变的,步骤是可以往下退的。
等我把归纳法昨晚以后,我一看,哟,三个红眼睛,那这个时候我的前提就是“我看到
了三个红眼睛,这三个红眼睛也都知道岛上有红眼睛的人”。

【在 l*3 的大作中提到】
: 你的前提是什么? 是不是 "岛上只有一个红眼睛" ?
l*3
发帖数: 2279
439
这么给你说吧:
X认为X是蓝眼睛, 且X认为A认为A是蓝眼睛 <=> X认为 (X是蓝眼睛, 且A认为A是蓝眼睛
) <=> X认为 (X 是蓝眼睛, 且A认为 (X是蓝眼睛, A是蓝眼睛) )
你能看懂这个等价关系不? 你要能看懂的话, 估计你就知道你的问题在哪了.
你把基本嵌套的视角中的 "事实" 带入了高层嵌套中的人的 "认识" 中, 而忽略了: 高
层嵌套中的人的认知, 可能并不如低层嵌套中的人的认知全面.

【在 h*x 的大作中提到】
: 你觉得你对,你回答不出我问你的问题。
: 我觉得我对,我回答的出你的问题。
: 如果有第三个人,他会觉得我们谁对呀?

l*3
发帖数: 2279
440
你归纳法的第一步是什么? "如果岛上只有一个红眼睛, 那么这个红眼睛会在地一天晚
上自杀" ?
这第一步都是错的, 那你后面全是错的.
谁管你归纳法做完后又如何如何.

【在 h*x 的大作中提到】
: “岛上只有一个红眼睛”是我做归纳法的第一步,不是前提,前提跟第一步不是一个概
: 念,前提是不变的,步骤是可以往下退的。
: 等我把归纳法昨晚以后,我一看,哟,三个红眼睛,那这个时候我的前提就是“我看到
: 了三个红眼睛,这三个红眼睛也都知道岛上有红眼睛的人”。

相关主题
中国北方人跟南方人基本上是两个种族,无法协调别做小学题了,来个初中的
大家承认吧蓝眼睛就是比粽眼睛好看 (转载)问个奥数题
葛军,男,秒杀了52万江苏考生。。来做最后两题吧问个中文标点符号的嵌套问题
进入WaterWorld版参与讨论
h*x
发帖数: 674
441
可是2不对呀,X眼中{A可以看到BC,B可以看到AC,C可以看到AB},他们没有一个人的
眼中会认为可能没有红眼睛。
这是事实。你不要把你推导过程中的假设跟事实搞混了。

【在 l*3 的大作中提到】
: 1. 岛上的人都看到有红眼睛, 这个没问题.
: 2. X的眼中的A的眼中的B的眼中的C的眼中是可能没有红眼睛的.
: 你搞不懂 2 表述的意思, 老以为 1 就能说明 2 不对, 这就是你的逻辑问题.

h*x
发帖数: 674
442
切。。

【在 p*****y 的大作中提到】
: 我回答不出你的问题是因为你的问题不合逻辑。
: 你回答了我的问题,但是你得回答的逻辑是错的。
: 第三个人要说觉得你对,说明他没有学过逻辑学,哈哈

l*3
发帖数: 2279
443
仔细看我2说的是什么, 我2说的可不是X眼中的A眼中看不到红眼睛.
我说的是 "X眼中的A眼中的B眼中的C眼中可能看不到红眼睛"

【在 h*x 的大作中提到】
: 可是2不对呀,X眼中{A可以看到BC,B可以看到AC,C可以看到AB},他们没有一个人的
: 眼中会认为可能没有红眼睛。
: 这是事实。你不要把你推导过程中的假设跟事实搞混了。

h*x
发帖数: 674
444
我431楼不说了吗,如果这个红眼睛知道岛上有红眼睛的话,他会在第一天晚上自杀。
然后你问我这不需要有人告诉他岛上有红眼睛吗?
然后我说,这个他在哪? 在归纳法的第一步,不在岛上,岛上没有人不知道有红眼睛。
然后你说我不承认自己前提,
然后我说“只有一个红眼睛”是我做归纳法的第一步,我在岛上的前提是“我看到了三
个红眼睛,并且知道他们也看到了红眼睛”。

【在 l*3 的大作中提到】
: 你归纳法的第一步是什么? "如果岛上只有一个红眼睛, 那么这个红眼睛会在地一天晚
: 上自杀" ?
: 这第一步都是错的, 那你后面全是错的.
: 谁管你归纳法做完后又如何如何.

p*****y
发帖数: 1982
445
你最后那个前提跟归纳法的第一步冲突了。

睛。

【在 h*x 的大作中提到】
: 我431楼不说了吗,如果这个红眼睛知道岛上有红眼睛的话,他会在第一天晚上自杀。
: 然后你问我这不需要有人告诉他岛上有红眼睛吗?
: 然后我说,这个他在哪? 在归纳法的第一步,不在岛上,岛上没有人不知道有红眼睛。
: 然后你说我不承认自己前提,
: 然后我说“只有一个红眼睛”是我做归纳法的第一步,我在岛上的前提是“我看到了三
: 个红眼睛,并且知道他们也看到了红眼睛”。

h*x
发帖数: 674
446
我看的懂啊,我也承认啊。
用你第三个吧,X认为 (X 是蓝眼睛, 且A认为 (X是蓝眼睛, A是蓝眼睛) ),这个很好。
第一天
X认为 (X 是蓝眼睛, 且A认为 (X是蓝眼睛, A是蓝眼睛) )
第二天
X认为 (X 是蓝眼睛, 且A认为 (X是蓝眼睛, A是蓝眼睛) )
第三天
X认为 (X 是蓝眼睛, 且A发现自己不是蓝眼睛,因为前两天没人自杀)
第三天晚上过了午夜
X发现自己是红眼睛,因为第三天晚上也没人自杀
第四天
ABCX一起自杀

【在 l*3 的大作中提到】
: 这么给你说吧:
: X认为X是蓝眼睛, 且X认为A认为A是蓝眼睛 <=> X认为 (X是蓝眼睛, 且A认为A是蓝眼睛
: ) <=> X认为 (X 是蓝眼睛, 且A认为 (X是蓝眼睛, A是蓝眼睛) )
: 你能看懂这个等价关系不? 你要能看懂的话, 估计你就知道你的问题在哪了.
: 你把基本嵌套的视角中的 "事实" 带入了高层嵌套中的人的 "认识" 中, 而忽略了: 高
: 层嵌套中的人的认知, 可能并不如低层嵌套中的人的认知全面.

l*3
发帖数: 2279
447
你这前提足够吗?
你现在讨论的 "只有一个红眼睛" 是 "X (也就是你) 认为A认为B认为C是红眼睛" 的这
种情形吧?
请问为什么 "X认为A认为B认为C认为岛上有红眼睛" ?
你只说了 "两层认为" , 也就是 "我看到了三个红眼睛,并且知道他们也看到了红眼睛
", 这至多只能说明 "X认为A认为有红眼睛"
根本不能说明 "X认为A认为B认为C认为岛上有红眼睛"
你是如何论证 "X认为A认为B认为C认为岛上有红眼睛" 的?
或者说, 你凭什么认为A认为B认为C认为岛上有红眼睛?

睛。

【在 h*x 的大作中提到】
: 我431楼不说了吗,如果这个红眼睛知道岛上有红眼睛的话,他会在第一天晚上自杀。
: 然后你问我这不需要有人告诉他岛上有红眼睛吗?
: 然后我说,这个他在哪? 在归纳法的第一步,不在岛上,岛上没有人不知道有红眼睛。
: 然后你说我不承认自己前提,
: 然后我说“只有一个红眼睛”是我做归纳法的第一步,我在岛上的前提是“我看到了三
: 个红眼睛,并且知道他们也看到了红眼睛”。

l*3
发帖数: 2279
448
如果一开始, 是这样的:
X认为
{
X是蓝眼睛,
且A认为
{
X和A是蓝眼睛,
且B认为
{
X,A,B是蓝眼睛,
且C认为
{
X,A,B,C是蓝眼睛,
}
}
}
}
其他人同理. 结果一直没人自杀, 因为大家都无法排除这种情况.
你如何解释这个?

好。

【在 h*x 的大作中提到】
: 我看的懂啊,我也承认啊。
: 用你第三个吧,X认为 (X 是蓝眼睛, 且A认为 (X是蓝眼睛, A是蓝眼睛) ),这个很好。
: 第一天
: X认为 (X 是蓝眼睛, 且A认为 (X是蓝眼睛, A是蓝眼睛) )
: 第二天
: X认为 (X 是蓝眼睛, 且A认为 (X是蓝眼睛, A是蓝眼睛) )
: 第三天
: X认为 (X 是蓝眼睛, 且A发现自己不是蓝眼睛,因为前两天没人自杀)
: 第三天晚上过了午夜
: X发现自己是红眼睛,因为第三天晚上也没人自杀

h*x
发帖数: 674
449
同学。。。。。。。我们又绕到老话题了。
不冲突。
最后那个前提是事实,是这个岛上的客观事实,但不妨碍我假设一些岛上不存在的情况
来做我的归纳法,我假设的情况是岛上不存在的,但没有关系,岛上的前提条件和我假
设的前提条件是一样的,即每个人都知道岛上有红眼睛的存在。在我的假设中,是别人
告诉了岛上的人岛上有红眼睛,在客观事实中,是岛上的人看到了红眼睛。虽然实现前
提条件的途径不同,前提条件是一样的。 所以我可以把我的假设应用到岛上。

【在 p*****y 的大作中提到】
: 你最后那个前提跟归纳法的第一步冲突了。
:
: 睛。

h*x
发帖数: 674
450
前提是岛上每一个人都知道有红眼睛,
在我的归纳法中,存在“只有一个红眼睛”的情况,这个红眼睛能知道岛上有红眼睛是
别人告诉他的。
在现实中,不存在“只有一个红眼睛”的情况,每个红眼睛都看的到其他的红眼睛,也
看到其他好眼睛看到其他红眼睛,所以仍然是岛上每一个人都知道有红眼睛这个前提。
"X认为A认为B认为C认为岛上有红眼睛" 是因为X知道A可以看到B看到C,C看到B,所以"
X认为A认为B认为C认为岛上有红眼睛" ,并且"X认为A认为C认为B认为岛上有红眼睛”

【在 l*3 的大作中提到】
: 你这前提足够吗?
: 你现在讨论的 "只有一个红眼睛" 是 "X (也就是你) 认为A认为B认为C是红眼睛" 的这
: 种情形吧?
: 请问为什么 "X认为A认为B认为C认为岛上有红眼睛" ?
: 你只说了 "两层认为" , 也就是 "我看到了三个红眼睛,并且知道他们也看到了红眼睛
: ", 这至多只能说明 "X认为A认为有红眼睛"
: 根本不能说明 "X认为A认为B认为C认为岛上有红眼睛"
: 你是如何论证 "X认为A认为B认为C认为岛上有红眼睛" 的?
: 或者说, 你凭什么认为A认为B认为C认为岛上有红眼睛?
:

相关主题
大部分人不懂反证法在自己国家岛上杀死敌人侵略者应该不算犯罪,是英雄
每周1评:[2] 方舟子 的 16点动物园举办【我和红眼蝉】活动 (转载)
陕西出现红眼兽 (转载)[合集] 内贾德第一个在钓岛上声援中国 一席话太解气了!
进入WaterWorld版参与讨论
l*3
发帖数: 2279
451
为了帮助你理解, 我再说两句:
"C认为X,A,B,C都是蓝眼睛" ≠ "B认为C认为X,A,B,C都是蓝眼睛" ≠ "A认为B认为C认
为X,A,B,C都是蓝眼睛" ≠ "X认为A认为B认为C认为X,A,B,C都是蓝眼睛"
以上的最后一个断言, 即 "X认为A认为B认为C认为X,A,B,C都是蓝眼睛" 是不可排除的
(如果 "存在红眼" 不被公开宣称为common sense的话)

好。

【在 h*x 的大作中提到】
: 我看的懂啊,我也承认啊。
: 用你第三个吧,X认为 (X 是蓝眼睛, 且A认为 (X是蓝眼睛, A是蓝眼睛) ),这个很好。
: 第一天
: X认为 (X 是蓝眼睛, 且A认为 (X是蓝眼睛, A是蓝眼睛) )
: 第二天
: X认为 (X 是蓝眼睛, 且A认为 (X是蓝眼睛, A是蓝眼睛) )
: 第三天
: X认为 (X 是蓝眼睛, 且A发现自己不是蓝眼睛,因为前两天没人自杀)
: 第三天晚上过了午夜
: X发现自己是红眼睛,因为第三天晚上也没人自杀

h*x
发帖数: 674
452
一开始是这样的:
X认为
{
X是蓝眼睛,
{且A认为X和A是蓝眼睛},{且B认为X和B是蓝眼睛},{且C认为X和C是蓝眼睛}
}
X不会认为B认为A是蓝眼睛的,B不存在A的脑海中,B存在这个岛上。
归纳法中不也是几个人同时知道他们是不是红眼睛,而不是一个个地知道。

【在 l*3 的大作中提到】
: 如果一开始, 是这样的:
: X认为
: {
: X是蓝眼睛,
: 且A认为
: {
: X和A是蓝眼睛,
: 且B认为
: {
: X,A,B是蓝眼睛,

h*x
发帖数: 674
453
我知道你们的意思啊,我要是不知道你们的意思怎么可能回答出你们前面的每一个问题。
算了吧,咱不说了吧? 就这样吧?



【在 l*3 的大作中提到】
: 为了帮助你理解, 我再说两句:
: "C认为X,A,B,C都是蓝眼睛" ≠ "B认为C认为X,A,B,C都是蓝眼睛" ≠ "A认为B认为C认
: 为X,A,B,C都是蓝眼睛" ≠ "X认为A认为B认为C认为X,A,B,C都是蓝眼睛"
: 以上的最后一个断言, 即 "X认为A认为B认为C认为X,A,B,C都是蓝眼睛" 是不可排除的
: (如果 "存在红眼" 不被公开宣称为common sense的话)
:
: 好。

p*****y
发帖数: 1982
454
我刚才说你不让嵌套的你干嘛不承认,这样让我很没面子

【在 h*x 的大作中提到】
: 一开始是这样的:
: X认为
: {
: X是蓝眼睛,
: {且A认为X和A是蓝眼睛},{且B认为X和B是蓝眼睛},{且C认为X和C是蓝眼睛}
: }
: X不会认为B认为A是蓝眼睛的,B不存在A的脑海中,B存在这个岛上。
: 归纳法中不也是几个人同时知道他们是不是红眼睛,而不是一个个地知道。

p*****y
发帖数: 1982
455
嗯,散了散了

题。

【在 h*x 的大作中提到】
: 我知道你们的意思啊,我要是不知道你们的意思怎么可能回答出你们前面的每一个问题。
: 算了吧,咱不说了吧? 就这样吧?
:
: 的

l*3
发帖数: 2279
456
引用-----
这个红眼睛能知道岛上有红眼睛是别人告诉他的。
---结束引用
题目里很明确的说明了: 岛内人不允许以任何形式互相讨论关于眼睛颜色的事情. 你这
归纳法就错了.
引用------
"X认为A认为B认为C认为岛上有红眼睛" 是因为X知道A可以看到B看到C,C看到B,所以"
X认为A认为B认为C认为岛上有红眼睛"
---结束引用
"C看到B是红眼" 这件事情, B是不自知的, 如果你的嵌套成立, 你等于直接承认了 "B
自知是红眼", 既然如此, B直接当晚自杀好了, 等那么久干什么?
我修改一下用词, 之前的问法不够准确:
你(X)凭什么认为A认为B认为C认为岛上一定有红眼睛 (注意 "一定" 二字)?
你如何排除这种情况? :


以"


【在 h*x 的大作中提到】
: 前提是岛上每一个人都知道有红眼睛,
: 在我的归纳法中,存在“只有一个红眼睛”的情况,这个红眼睛能知道岛上有红眼睛是
: 别人告诉他的。
: 在现实中,不存在“只有一个红眼睛”的情况,每个红眼睛都看的到其他的红眼睛,也
: 看到其他好眼睛看到其他红眼睛,所以仍然是岛上每一个人都知道有红眼睛这个前提。
: "X认为A认为B认为C认为岛上有红眼睛" 是因为X知道A可以看到B看到C,C看到B,所以"
: X认为A认为B认为C认为岛上有红眼睛" ,并且"X认为A认为C认为B认为岛上有红眼睛”

l*3
发帖数: 2279
457
你这就是狗屎逻辑了.
我回答了你的每一个问题, 那是不是说我也懂你意思? 而且你是错的?

题。

【在 h*x 的大作中提到】
: 我知道你们的意思啊,我要是不知道你们的意思怎么可能回答出你们前面的每一个问题。
: 算了吧,咱不说了吧? 就这样吧?
:
: 的

l*3
发帖数: 2279
458
这怎么又开始说 "不存在嵌套" 了?
那你刚叨叨半天说的都是啥?

【在 h*x 的大作中提到】
: 一开始是这样的:
: X认为
: {
: X是蓝眼睛,
: {且A认为X和A是蓝眼睛},{且B认为X和B是蓝眼睛},{且C认为X和C是蓝眼睛}
: }
: X不会认为B认为A是蓝眼睛的,B不存在A的脑海中,B存在这个岛上。
: 归纳法中不也是几个人同时知道他们是不是红眼睛,而不是一个个地知道。

h*x
发帖数: 674
459
嘿嘿,不好意思,我收回:)
我嵌套了两层。我觉得可以在假设中嵌套多层,但面对岛上的实际问题,只要嵌套两层。
刚才他还在问我X认为A认为怎样怎样,这个是两层,所以我说ok,嵌套呀。
但是再往后我们在前提条件上发生了分歧。 哎,算了,就这样了吧?

【在 p*****y 的大作中提到】
: 我刚才说你不让嵌套的你干嘛不承认,这样让我很没面子
h*x
发帖数: 674
460
你不要说脏话好不好,最烦你这点。
你回答了我什么问题呀,我说个n和k,你都不让我用。 你用的各种符号,我哪个不是
随着你用的?

【在 l*3 的大作中提到】
: 你这就是狗屎逻辑了.
: 我回答了你的每一个问题, 那是不是说我也懂你意思? 而且你是错的?
:
: 题。

相关主题
靠~ 受不了,宇宙大得让我头晕[讨论]携带iPad进入中国内地要交税 (ZT)
人不能蒙着眼睛说瞎话[合集] [转] 红蓝眼睛的逻辑陷阱
神啊,我老公眼睛都直了,我这个恨啊!再贴几张一个外嫁后生的孩子的照片
进入WaterWorld版参与讨论
l*3
发帖数: 2279
461
OK, 你既然不理解正确的做法, 那我也没有办法.
但是我可以这样指出你的错误:
你的归纳法第一步就错了, 原题明确说了 "岛上的人不允许互相讨论关于眼睛颜色的问
题"
而你归纳法中用了 "岛上的人可以互相告诉 "有没有红色眼睛" 这回事", 所以错了.
你要想证明你对, 就请用其他方法绕开这个漏洞.

【在 h*x 的大作中提到】
: 一开始是这样的:
: X认为
: {
: X是蓝眼睛,
: {且A认为X和A是蓝眼睛},{且B认为X和B是蓝眼睛},{且C认为X和C是蓝眼睛}
: }
: X不会认为B认为A是蓝眼睛的,B不存在A的脑海中,B存在这个岛上。
: 归纳法中不也是几个人同时知道他们是不是红眼睛,而不是一个个地知道。

l*3
发帖数: 2279
462
我回答了你什么问题?
那我还反问你回答了我什么问题呢.
这种主观性的判断最好别做, 因为在另一方看来, 你犯的是同样的错误.
我现在就这么指出你的错误所在:
你的归纳法第一步就错了, 原题明确说了 "岛上的人不允许互相讨论关于眼睛颜色的问
题"
而你归纳法中用了 "岛上的人可以互相告诉 "有没有红色眼睛" 这回事", 所以错了.
你要想证明你对, 就请用其他方法绕开这个漏洞.

【在 h*x 的大作中提到】
: 你不要说脏话好不好,最烦你这点。
: 你回答了我什么问题呀,我说个n和k,你都不让我用。 你用的各种符号,我哪个不是
: 随着你用的?

l*3
发帖数: 2279
463
说脏话不文明, 我道歉.
不过在我看来你还是错的.

【在 h*x 的大作中提到】
: 你不要说脏话好不好,最烦你这点。
: 你回答了我什么问题呀,我说个n和k,你都不让我用。 你用的各种符号,我哪个不是
: 随着你用的?

l*3
发帖数: 2279
464
你的归纳法第一步就错了, 原题明确说了 "岛上的人不允许互相讨论关于眼睛颜色的问
题"
而你归纳法中用了 "岛上的人可以互相告诉 "有没有红色眼睛" 这回事", 所以错了.
你要想证明你对, 就请用其他方法绕开这个漏洞, 并完整的把证明过程叙述出来.

【在 h*x 的大作中提到】
: 你不要说脏话好不好,最烦你这点。
: 你回答了我什么问题呀,我说个n和k,你都不让我用。 你用的各种符号,我哪个不是
: 随着你用的?

h*x
发帖数: 674
465
恩,我归纳法中用了。 这有啥错的?
你管我怎么让他们知道的,只要他们知道就行了。
就好比我们都知道水到100摄氏度就沸腾,现在水100摄氏度了,我问你会不会沸腾,你
肯定说会啊。 你管我怎么把水弄到100摄氏度的,我可以用火烧,也可以用电炉,反正
弄到了贝。

【在 l*3 的大作中提到】
: 我回答了你什么问题?
: 那我还反问你回答了我什么问题呢.
: 这种主观性的判断最好别做, 因为在另一方看来, 你犯的是同样的错误.
: 我现在就这么指出你的错误所在:
: 你的归纳法第一步就错了, 原题明确说了 "岛上的人不允许互相讨论关于眼睛颜色的问
: 题"
: 而你归纳法中用了 "岛上的人可以互相告诉 "有没有红色眼睛" 这回事", 所以错了.
: 你要想证明你对, 就请用其他方法绕开这个漏洞.

h*x
发帖数: 674
466
就是嘛。 但是认为是一回事,怎么做又是一回事啊。
你现在肯定想说我逻辑不对,我也想说你逻辑不对,如果我们都说对方逻辑不对,大家
都不开心,不如都不说啊。
好歹也灌了这么长时间的水了,没有功劳也有苦劳。

【在 l*3 的大作中提到】
: 说脏话不文明, 我道歉.
: 不过在我看来你还是错的.

l*3
发帖数: 2279
467
现在原题的条件是 "大气压有500kpa", 在这个条件下水到100度不沸腾.
结果你还要说水100度沸腾, 并且以为我在质疑你到底是用火烧的还是用电炉, 这不是
很无聊吗?
我只是在告诉你, 500kpa下, 水到100度不会飞腾, 你却洋洋自得的对我说 "哈哈, 我
是用电炉让他沸腾的, 不是用火烧的"

【在 h*x 的大作中提到】
: 恩,我归纳法中用了。 这有啥错的?
: 你管我怎么让他们知道的,只要他们知道就行了。
: 就好比我们都知道水到100摄氏度就沸腾,现在水100摄氏度了,我问你会不会沸腾,你
: 肯定说会啊。 你管我怎么把水弄到100摄氏度的,我可以用火烧,也可以用电炉,反正
: 弄到了贝。

h*x
发帖数: 674
468
我不觉得这个错。这个是我假设,这有什么的。
我的假设还n=1怎样怎样,n=2怎样怎样呢,那n=1不是和n=2自相矛盾吗,怎么能同时存
在于一个假设中呢?
我觉得只要我假设的前提条件,和现实生活的前提条件一样,我就可以用我的假设。

【在 l*3 的大作中提到】
: 你的归纳法第一步就错了, 原题明确说了 "岛上的人不允许互相讨论关于眼睛颜色的问
: 题"
: 而你归纳法中用了 "岛上的人可以互相告诉 "有没有红色眼睛" 这回事", 所以错了.
: 你要想证明你对, 就请用其他方法绕开这个漏洞, 并完整的把证明过程叙述出来.

c****p
发帖数: 6474
469
3个人也不会自杀吧。
任何一个红眼睛看到另外两个红眼睛都没法判断自己是不是红眼睛。
因为只有两个红眼睛的时候这俩红眼睛也不会自杀。
所以第三个红眼睛看到另外两个在第二天没自杀,并不严格说明自己也是红眼睛。

【在 h*x 的大作中提到】
: 如果岛上的红眼睛的人数大于2,就可以。
: 因为大家都知道有红眼睛了。也知道别人都知道有红眼睛了。
: 然后只要根据自己看到的红眼睛的人数在相应的第几天等就可以了。

l*3
发帖数: 2279
470
谁告诉你n=1和n=2在同一个假设中了?
归纳法可没这么说过.

【在 h*x 的大作中提到】
: 我不觉得这个错。这个是我假设,这有什么的。
: 我的假设还n=1怎样怎样,n=2怎样怎样呢,那n=1不是和n=2自相矛盾吗,怎么能同时存
: 在于一个假设中呢?
: 我觉得只要我假设的前提条件,和现实生活的前提条件一样,我就可以用我的假设。

相关主题
再贴几张一个外嫁后生的孩子的照片大家承认吧蓝眼睛就是比粽眼睛好看 (转载)
娶妻三次, 三个女人致其性无能葛军,男,秒杀了52万江苏考生。。来做最后两题吧
中国北方人跟南方人基本上是两个种族,无法协调别做小学题了,来个初中的
进入WaterWorld版参与讨论
h*x
发帖数: 674
471
好呀,条件是“大气压有500kpa”,我说假设用实验仪器A我们可以做到这个气压。
结果现在用仪器B了,用哪个仪器重要吗? 让条件满足“大气压有500kpa”不就行了?

【在 l*3 的大作中提到】
: 现在原题的条件是 "大气压有500kpa", 在这个条件下水到100度不沸腾.
: 结果你还要说水100度沸腾, 并且以为我在质疑你到底是用火烧的还是用电炉, 这不是
: 很无聊吗?
: 我只是在告诉你, 500kpa下, 水到100度不会飞腾, 你却洋洋自得的对我说 "哈哈, 我
: 是用电炉让他沸腾的, 不是用火烧的"

h*x
发帖数: 674
472
整个归纳法就是假设n等于任何整数的情况!
你说n能不能是1? 能不能是2?

【在 l*3 的大作中提到】
: 谁告诉你n=1和n=2在同一个假设中了?
: 归纳法可没这么说过.

l*3
发帖数: 2279
473
你从一个偷换概念的行为跳到另一个偷换概念的行为, 没啥意思.
-------
总之, 你在你的证明中, 用到了一条, 也就是:
岛上的人可以互相讨论关于 "是否存在红眼" 的事情.
只要你用到这个, 证明就错了, 不管你是在什么地方用的, 只要你用了, 就是错, 证明
的规范要求就是如此的. 你如果觉得你能给出正确严谨的证明, 那你一定可以避开使用
上面那条.

【在 h*x 的大作中提到】
: 好呀,条件是“大气压有500kpa”,我说假设用实验仪器A我们可以做到这个气压。
: 结果现在用仪器B了,用哪个仪器重要吗? 让条件满足“大气压有500kpa”不就行了?

l*3
发帖数: 2279
474
你这就是不懂归纳法.
归纳法说的是: 如果n=1, 那么如何如何. 如果n=2 那么如何如何..
归纳法从来不说 "如果n=1且n=2, 那么如何如何"
你不明白什么叫 "出现在同一个假设中"

【在 h*x 的大作中提到】
: 整个归纳法就是假设n等于任何整数的情况!
: 你说n能不能是1? 能不能是2?

h*x
发帖数: 674
475
所以说算了,别讨论了。。。我们谁也说服不了谁,虽然我们都觉得自己无比正确,对
方应该被自己说服,但是就接受现状吧。。。反正我是接受了。。。

【在 l*3 的大作中提到】
: 你从一个偷换概念的行为跳到另一个偷换概念的行为, 没啥意思.
: -------
: 总之, 你在你的证明中, 用到了一条, 也就是:
: 岛上的人可以互相讨论关于 "是否存在红眼" 的事情.
: 只要你用到这个, 证明就错了, 不管你是在什么地方用的, 只要你用了, 就是错, 证明
: 的规范要求就是如此的. 你如果觉得你能给出正确严谨的证明, 那你一定可以避开使用
: 上面那条.

l*3
发帖数: 2279
476
你仔细想想你的证明过程.
不管你其他部分说了什么, 只要你在证明中说过类似 <因为岛上的人可以互相告诉 "自
己是否看到红眼" > 这样的话, 那就是个错误的证明.

【在 h*x 的大作中提到】
: 整个归纳法就是假设n等于任何整数的情况!
: 你说n能不能是1? 能不能是2?

h*x
发帖数: 674
477
哎,我想了,我说出来又是新的一轮想。咱不说了吧。

【在 l*3 的大作中提到】
: 你仔细想想你的证明过程.
: 不管你其他部分说了什么, 只要你在证明中说过类似 <因为岛上的人可以互相告诉 "自
: 己是否看到红眼" > 这样的话, 那就是个错误的证明.

l*3
发帖数: 2279
478
你的证明中是否用到了 <岛上的人可以互相讨论关于 "眼睛颜色" 的事情> 这一条件?
用到就是错, 没用到就还有可能对.
这还有什么好多说的? 我觉得这么简单的事情, 没必要上升到什么 "观念不同" "公说
公有理婆说婆有理" "每个人都有没个人的看法" 这一类的大道理上去吧?

【在 h*x 的大作中提到】
: 所以说算了,别讨论了。。。我们谁也说服不了谁,虽然我们都觉得自己无比正确,对
: 方应该被自己说服,但是就接受现状吧。。。反正我是接受了。。。

l*3
发帖数: 2279
479
我就问你, 现在有一个问题, 他有一些前提条件, 记作q1,q2等等.
然后有一个人要做这个问题, 他给出了一个结果和证明, 他在证明中用到了 "q1不成立
" 这一命题. 那这证明还对不对?

【在 h*x 的大作中提到】
: 哎,我想了,我说出来又是新的一轮想。咱不说了吧。
h*x
发帖数: 674
480
哈哈哈,好吧,我错了,不说了,你早点休息吧~~

?

【在 l*3 的大作中提到】
: 你的证明中是否用到了 <岛上的人可以互相讨论关于 "眼睛颜色" 的事情> 这一条件?
: 用到就是错, 没用到就还有可能对.
: 这还有什么好多说的? 我觉得这么简单的事情, 没必要上升到什么 "观念不同" "公说
: 公有理婆说婆有理" "每个人都有没个人的看法" 这一类的大道理上去吧?

相关主题
问个奥数题每周1评:[2] 方舟子 的 16点
问个中文标点符号的嵌套问题陕西出现红眼兽 (转载)
大部分人不懂反证法在自己国家岛上杀死敌人侵略者应该不算犯罪,是英雄
进入WaterWorld版参与讨论
p*****y
发帖数: 1982
481
你们怎么都还没走
那我来给你出个题。这是个新题,跟前面的讨论没有关系。
真命题1: 如果P1,那么P2,记作P1=>P2
真命题2: 如果P2且P3,那么P4,记作(P2 and P3)=>P4
真命题3: 如果P1,那么非P3,记作P1 => !P3
现已知P1为真,且没有其他条件,能否确定P4的值(真或假)?
请回答,确定或者不确定。

【在 h*x 的大作中提到】
: 恩,我归纳法中用了。 这有啥错的?
: 你管我怎么让他们知道的,只要他们知道就行了。
: 就好比我们都知道水到100摄氏度就沸腾,现在水100摄氏度了,我问你会不会沸腾,你
: 肯定说会啊。 你管我怎么把水弄到100摄氏度的,我可以用火烧,也可以用电炉,反正
: 弄到了贝。

l*3
发帖数: 2279
482
你这态度太不诚恳了.
根本没有从内心里意识到你的错误.
我就问你一个, 你什么专业的?
告诉我, 我就不继续讨论了.

【在 h*x 的大作中提到】
: 哈哈哈,好吧,我错了,不说了,你早点休息吧~~
:
: ?

h*x
发帖数: 674
483
不能啊。 P1 => !P3
就不存在 (p2 and p3),就推不出p4啊。

【在 p*****y 的大作中提到】
: 你们怎么都还没走
: 那我来给你出个题。这是个新题,跟前面的讨论没有关系。
: 真命题1: 如果P1,那么P2,记作P1=>P2
: 真命题2: 如果P2且P3,那么P4,记作(P2 and P3)=>P4
: 真命题3: 如果P1,那么非P3,记作P1 => !P3
: 现已知P1为真,且没有其他条件,能否确定P4的值(真或假)?
: 请回答,确定或者不确定。

p*****y
发帖数: 1982
484
嗯,我就觉得你啥都知道,一直在戏耍我们。

【在 h*x 的大作中提到】
: 不能啊。 P1 => !P3
: 就不存在 (p2 and p3),就推不出p4啊。

h*x
发帖数: 674
485
哈哈,就不告诉你! 我干嘛暴露我自己啊。
我知道你会看到我答案,但我不知道其他还有谁会看到我回复。
万一我在其他地方再露点信息,被人肉了怎么办。

【在 l*3 的大作中提到】
: 你这态度太不诚恳了.
: 根本没有从内心里意识到你的错误.
: 我就问你一个, 你什么专业的?
: 告诉我, 我就不继续讨论了.

h*x
发帖数: 674
486
怎么可能,我有那么无聊吗。。。我前面每个回复都用心回复的。

【在 p*****y 的大作中提到】
: 嗯,我就觉得你啥都知道,一直在戏耍我们。
p*****y
发帖数: 1982
487
那我还是上次那个题,三个真命题不变,这次已知P1和P3均为真值,能否确定P4的值?
为什么?

【在 h*x 的大作中提到】
: 怎么可能,我有那么无聊吗。。。我前面每个回复都用心回复的。
l*3
发帖数: 2279
488
说个专业都会暴露?
那你私信我呗.谢

【在 h*x 的大作中提到】
: 哈哈,就不告诉你! 我干嘛暴露我自己啊。
: 我知道你会看到我答案,但我不知道其他还有谁会看到我回复。
: 万一我在其他地方再露点信息,被人肉了怎么办。

h*x
发帖数: 674
489
真命题3不是 如果P1,那么非P3 吗?
怎么会出现P1和P3均为真值啊?

【在 p*****y 的大作中提到】
: 那我还是上次那个题,三个真命题不变,这次已知P1和P3均为真值,能否确定P4的值?
: 为什么?

h*x
发帖数: 674
490
不要。我又不认识你。
再说,我告诉你我的专业我有什么好处?
如果你说我告诉你我的专业你就不讨论了,那我可以单方面停止讨论啊,这个如果对我
而言没用,嘿嘿~~

【在 l*3 的大作中提到】
: 说个专业都会暴露?
: 那你私信我呗.谢

相关主题
动物园举办【我和红眼蝉】活动 (转载)人不能蒙着眼睛说瞎话
[合集] 内贾德第一个在钓岛上声援中国 一席话太解气了!神啊,我老公眼睛都直了,我这个恨啊!
靠~ 受不了,宇宙大得让我头晕[讨论]携带iPad进入中国内地要交税 (ZT)
进入WaterWorld版参与讨论
l*3
发帖数: 2279
491
凝聚态物理

【在 h*x 的大作中提到】
: 不要。我又不认识你。
: 再说,我告诉你我的专业我有什么好处?
: 如果你说我告诉你我的专业你就不讨论了,那我可以单方面停止讨论啊,这个如果对我
: 而言没用,嘿嘿~~

h*x
发帖数: 674
492
你随便猜,反正我不置可否 :p

【在 l*3 的大作中提到】
: 凝聚态物理
p*****y
发帖数: 1982
493
那我换个问法,
这是一个老题。一个人X在进行推理,条件是那三个真命题。
问题1:X决定把(P1为真)作为推理的前提条件,他还能否同时假设P3为真?
问题2:如果X在推理中用P3为真作条件,他还需要补充什么条件可以得到P4为真?

【在 h*x 的大作中提到】
: 真命题3不是 如果P1,那么非P3 吗?
: 怎么会出现P1和P3均为真值啊?

h*x
发帖数: 674
494
答问题1: 不能啊,因为这个假设跟真命题3矛盾。
答问题2: 补充p2也是真。

【在 p*****y 的大作中提到】
: 那我换个问法,
: 这是一个老题。一个人X在进行推理,条件是那三个真命题。
: 问题1:X决定把(P1为真)作为推理的前提条件,他还能否同时假设P3为真?
: 问题2:如果X在推理中用P3为真作条件,他还需要补充什么条件可以得到P4为真?

p*****y
发帖数: 1982
495
恭喜你,都对了。注意问题2,如果补充P1为真是无效的,虽然可以间接得到P2为真,
但是P1跟P3互斥了
我说的是你那个归纳法的第一步,(此题在哪里我也不知道,就是你是X,看见ABC三个
红眼的那个题,但是不知道自己是不是红眼,扔到岛上算第一天,没有旅行者)

【在 h*x 的大作中提到】
: 答问题1: 不能啊,因为这个假设跟真命题3矛盾。
: 答问题2: 补充p2也是真。

h*x
发帖数: 674
496
就知道你要绕到这个问题上。
那p1p2p3p4分别是我归纳法第一步的什么?

【在 p*****y 的大作中提到】
: 恭喜你,都对了。注意问题2,如果补充P1为真是无效的,虽然可以间接得到P2为真,
: 但是P1跟P3互斥了
: 我说的是你那个归纳法的第一步,(此题在哪里我也不知道,就是你是X,看见ABC三个
: 红眼的那个题,但是不知道自己是不是红眼,扔到岛上算第一天,没有旅行者)

p*****y
发帖数: 1982
497
P3是归纳法第一步"岛上只有一个红眼人"
P2是“该红眼人知道岛上有红眼人”,这两个可以得到P4“该红眼人知道自己是红眼人”
P1是“该红眼人看见岛上有红眼人”
P1可以得到P2,但是P1和P3是互斥的,因为P3是岛上只有一个红眼人,P3决定了这个人
只能看见蓝眼人不能看见红眼人。
虽然X知道每个人都看见岛上有红眼人,但是这个事实不能用作推理。
有效补充使P2成立的方法是 旅行者说“岛上有红眼人”

【在 h*x 的大作中提到】
: 就知道你要绕到这个问题上。
: 那p1p2p3p4分别是我归纳法第一步的什么?

h*x
发帖数: 674
498
我还没看完,但看到p1和p3互斥那有点问题,
你原来的真命题是,如果p1真,那么非p3, 但没有如果p3真,那么非p1呀。
这才是互斥。
我自己纠正,这个没有问题。只要p1真,就非p3,那p3真,就非p1, 逆否命题成立。

人”

【在 p*****y 的大作中提到】
: P3是归纳法第一步"岛上只有一个红眼人"
: P2是“该红眼人知道岛上有红眼人”,这两个可以得到P4“该红眼人知道自己是红眼人”
: P1是“该红眼人看见岛上有红眼人”
: P1可以得到P2,但是P1和P3是互斥的,因为P3是岛上只有一个红眼人,P3决定了这个人
: 只能看见蓝眼人不能看见红眼人。
: 虽然X知道每个人都看见岛上有红眼人,但是这个事实不能用作推理。
: 有效补充使P2成立的方法是 旅行者说“岛上有红眼人”

p*****y
发帖数: 1982
499
嗯,搞错了,前面把自己绕进去了。

【在 h*x 的大作中提到】
: 我还没看完,但看到p1和p3互斥那有点问题,
: 你原来的真命题是,如果p1真,那么非p3, 但没有如果p3真,那么非p1呀。
: 这才是互斥。
: 我自己纠正,这个没有问题。只要p1真,就非p3,那p3真,就非p1, 逆否命题成立。
:
: 人”

p*****y
发帖数: 1982
500
哦,那没搞错,那就是太困了

【在 h*x 的大作中提到】
: 我还没看完,但看到p1和p3互斥那有点问题,
: 你原来的真命题是,如果p1真,那么非p3, 但没有如果p3真,那么非p1呀。
: 这才是互斥。
: 我自己纠正,这个没有问题。只要p1真,就非p3,那p3真,就非p1, 逆否命题成立。
:
: 人”

1 2 下页 末页 (共2页)
进入WaterWorld版参与讨论
相关主题
靠~ 受不了,宇宙大得让我头晕[讨论]大家承认吧蓝眼睛就是比粽眼睛好看 (转载)
人不能蒙着眼睛说瞎话葛军,男,秒杀了52万江苏考生。。来做最后两题吧
神啊,我老公眼睛都直了,我这个恨啊!别做小学题了,来个初中的
携带iPad进入中国内地要交税 (ZT)问个奥数题
[合集] [转] 红蓝眼睛的逻辑陷阱问个中文标点符号的嵌套问题
再贴几张一个外嫁后生的孩子的照片大部分人不懂反证法
娶妻三次, 三个女人致其性无能每周1评:[2] 方舟子 的 16点
中国北方人跟南方人基本上是两个种族,无法协调陕西出现红眼兽 (转载)
相关话题的讨论汇总
话题: 红眼话题: 眼睛话题: 岛上话题: 自杀话题: 知道